PSYCHIATRY ROTATION (Grady consult)

Lakukan tugas rumah & ujian kamu dengan baik sekarang menggunakan Quizwiz!

A 65-year-old woman with a long-standing history of poorly controlled diabetes mellitus presents to your office complaining of sleep disturbances, lack of interest in her hobbies, poor concentration, lack of energy, low motivation, and depressed mood on more days than not throughout the past month or so. She also states that she has had a burning sensation in her feet for many years. She has never had prior psychiatric treatment. Which of the following medications has been shown to benefit both of her conditions responsible for her complaints? A. duloxetine B.fluoxetine C. mirtazapine D. paroxetine E. phenelzine

The correct answer is: A Duloxetine is an anti-depressant which belongs to the SNRI class, being a dual serotonin and norepinephrine reuptake inhibitor). It is commonly prescribed for major depressive disorder, diabetic peripheral neuropathy, fibromyalgia, generalized anxiety disorder, chronic musculoskeletal pains, etc. Duloxetine is the only medication listed in the answer choices that is FDA approved for control of pain symptoms in diabetic neuropathy. Duloxetine boosts the neurotransmitters of serotonin, norepinephrine, and dopamine. It also works on the reuptake pumps. Duloxetine has been found to be very effective in the control of pain symptoms, and in someone with depression and pain issues, it targets both somatic and psychiatric symptoms in one medication. Onset of therapeutic action is usually not immediate, but is often delayed 2-4 weeks for depression. It can reduce neuropathic pain within a week, but onset can take longer. If it is not working adequately within 6-8 weeks, dosage can be increased. Some notable side effects include: nausea, diarrhea, decreased appetite, dry mouth, constipation, sweating, sexual dysfunction, urinary retention, etc. Answer B: Fluoxetine is an anti-depressant and belongs to the SSRI class of medications (selective serotonin reuptake inhibitors). It is commonly prescribed for major depressive disorder, obsessive-compulsive disorder, panic disorder, bulimia nervosa, PTSD, pre-menstrual dysphoric disorder, etc. It blocks the serotonin reuptake pump (serotonin transporter) and increases serotonergic neurotransmission, boosting serotonin neurotransmission. Fluoxetine also has antagonist properties at 5HT2C receptors, which could increase norepinephrine and dopamine neurotransmission. Fluoxetine has been known to be effective in the treatment of anxiety. Onset of therapeutic action is usually not immediate, but is often delayed 2-4 weeks for depression. If it is not working adequately within 6-8 weeks, dosage can be increased. Some notable side effects include: nausea, diarrhea, decreased appetite, dry mouth, constipation, agitation, insomnia, sexual dysfunction, sweating, urinary retention, etc. The long half-life of fluoxetine provides for decreased withdrawal symptoms following any abrupt discontinuation of medication. Answer C: Mirtazapine is an alpha-2-antagonist and a dual serotonin and norepinephrine agent. It works by blocking alpha-2 adrenergic presynaptic receptors, thereby increasing norepinephrine neurotransmission. It also blocks alpha-2 adrenergic presynaptic receptors on serotonin neurons, thereby increasing serotonergic neurotransmission. It also blocks 5HT2A, 5HT2C, and 5HT3 serotonin receptors and blocks H1 histamine receptors. Onset of therapeutic action is usually not immediate, but is often delayed 2-4 weeks for depression. If it is not working adequately within 6-8 weeks, dosage can be increased. Mirtazapine greatly helps for anxiety and insomnia that is associated with depression. It is very efficacious at sedation and should be taken at nighttime. Notable side effects include dry mouth, constipation, increased appetite and weight gain (due to H1 antagonism and 5HT2C antagonism), sedation (due to H1 antagonism), dizziness, urinary changes, hypotension, etc. Answer D: Paroxetine is an anti-depressant and belongs to the SSRI class of medications (selective serotonin reuptake inhibitors). It is commonly prescribed for major depressive disorder, obsessive-compulsive disorder, panic disorder, social anxiety disorder, post-traumatic stress disorder, generalized anxiety disorder, and pre-menstrual dysphoric disorder. It blocks the serotonin reuptake pump and increases serotonergic neurotransmission, boosting serotonin neurotransmission. Paroxetine also has mild anticholinergic actions. Paroxetine has been known to be very effective in the treatment of anxiety. Onset of therapeutic action is usually not immediate, but is often delayed 2-4 weeks for depression. If it is not working adequately within 6-8 weeks, dosage can be increased. Some notable side effects include: nausea, diarrhea, decreased appetite, dry mouth, constipation, agitation, insomnia, sexual dysfunction, urinary retention, etc. Paroxetine produces the highest incidence of discontinuation syndrome out of the SSRIs due to its relatively short half-life and anticholinergic activity. Answer E: Phenelzine belongs to the MAOI class (monoamine oxidase inhibitor) and is FDA approved for atypical depression, and is often used for treatment-resistant depression and treatment-resistant panic disorder. It works by irreversibly blocking MAO from breaking down norepinephrine, serotonin, and dopamine. This presumably boosts neurotransmission of these neurotransmitters. Onset of therapeutic action is usually not immediate, but is often delayed 2-4 weeks for depression. If it is not working adequately within 6-8 weeks, dosage can be increased. Notable side effects include dizziness, sedation, headache, sleep disturbance, fatigue, weakness, tremor, constipation, dry mouth, nausea, and weight gain. If a patient is on a MAOI, he/she has to avoid tyramine containing food to prevent occurrence of hypertensive crisis. MAOIs can also cause a possibly fatal serotonin syndrome when combined with medications that block serotonin reuptake, so it is advisable to wait 5 weeks after discontinuing an SSRI before starting the MAOI. Bottom Line: Duloxetine is an SNRI that can be used in the treatment of depression and diabetic neuropathy.

A 46-year-old female with a history of disorganized schizophrenia presents to the hospital via ambulance with altered mental status. She was found by her neighbors in the apartment complex hallway. History reveals that she has been off of her medication and had recently started taking haloperidol. Vitals are as follows: Temperature 38.9ºC (102oF) Blood pressure 182/109 mmHg Heart rate 120/min Respiratory rate 18/min Observation demonstrates severe muscle rigidity and diaphoresis. Question 1 of 3 in this set This patient is likely suffering from A. acute psychotic episode B. anticholinergic toxicity C. malignant hyperthermia D. neuroleptic malignant syndrome E. serotonin syndrome

The correct answer is: D Neuroleptic malignant syndrome (NMS) is an idiosyncratic and life threatening complication of neuroleptic medication(s) that is an emergency and can be fatal if not treated skillfully in a timely manner. As indicated by the name, it is most commonly associated with neuroleptic medications such as haloperidol, chlorpromazine, risperidone, etc. NMS is defined as the development of severe muscular rigidity, an elevated temperature (hyperthermia) in the setting of recent neuroleptic use, and two or more of the following symptoms: profuse diaphoresis, tremors, dysphagia, incontinence, changes in level of consciousness, mental status changes, mutism, elevated or labile blood pressure, tachycardia, tachypnea, leukocytosis, and laboratory evidence of muscular injury (elevated CPK). The symptoms should not be better accounted for by substance use or a general medical condition or a neurological condition. In this case scenario, the patient has elevated blood pressure, tachycardia, hyperthermia, tachypnea, severe muscular rigidity, and diaphoresis, and has recently started using neuroleptic medication, pointing towards the diagnosis of NMS. Answer A: An acute psychotic episode is unlikely to present with muscular rigidity or abnormal vital signs as described in the case scenario. Answer B: Anticholinergic medication toxicity presents with flushing, mydriasis, bladder retention, delirium, and hyperthermia. It is more common in the setting of anticholinergic overdose or exposure. Anticholinergic medication toxicity does NOT present with diaphoresis. The diagnosis of anticholinergic medication toxicity is further clarified by the administration of physostigmine, which will temporarily reverse signs and symptoms of anticholinergic excess. Answer C: Malignant hyperthermia is associated with the administration of anesthesia. It is manifest by the characteristic histologic appearance of skeletal muscle fibers. Although malignant hyperthermia has a very similar clinical presentation to NMS, it is more common in the setting of inhaled halogen anesthetic use, such as halothane. Symptoms include hyperthermia, muscular rigidity, and hyporeflexia. Answer E: Serotonin syndrome is classically known as a triad of mental status changes, autonomic hyperactivity, and neuromuscular abnormalities that occurs in the setting of serotonergic agents. Serotonin syndrome occurs more commonly in the setting of a drug interaction between a serotonergic agent and a monoamine oxidase inhibitor (MAOI). It can also result from the use of multiple medications with serotonergic actions or from overdoses involving serotonergic agents. Other symptoms include: shivering, restlessness, diaphoresis, tremor, hyperreflexia, myoclonus, and ataxia. It is also associated with the use of selective serotonin reuptake inhibitors. Tremor is a more frequently found peripheral motor finding. Fever is less frequent and less profound. Bottom Line: Neuroleptic malignant syndrome is an emergency that is associated with neuroleptic medication use, such as haloperidol, risperidone, chlorpromazine, etc. Symptoms include severe muscular rigidity, hyperthermia, altered mental status, tachycardia, and hypertension. Neuroleptic malignant syndrome (NMS) most commonly presents with markedly elevated levels (more than 1000 IU/L) of creatine kinase, making it a useful test to order (CPK) in a patient with NMS. Muscle cell damage from severe muscular rigidity during an acute episode can result in elevated serum CPK. Additionally, the damaged muscle cells can also release myoglobin, resulting in myoglobinuria and rhabdomyolysis. The treatment for neuroleptic malignant syndrome (NMS) largely involves cessation of neuroleptic medications (dopamine antagonists), starting dopamine agonist and muscle relaxant treatment, and supportive care measures such as IV fluids, antiarrhythmics to correct any potential arrhythmias that may arise from the syndrome, correction of electrolyte abnormalities, antihypertensive agents to correct malignant hypertension, and anticoagulation to prevent thrombosis. The initial management should incorporate transfer to a hospital where supportive care and intensive, adequate hemodynamic monitoring can be initiated. Supportive care includes adequate IV fluids and active cooling. Electrolyte balance and renal function have to be monitored closely given the increased risk of renal failure associated with NMS. The two agents that are useful in the treatment of NMS are dantrolene and bromocriptine. Dantrolene is a muscle relaxant that is effective in relieving the muscular rigidity, hyperthermia, and hypermetabolism. Bromocriptine is a centrally acting dopamine agonist that is given to restore lost dopaminergic tone during NMS.

Personality Disorders

A pervasive pattern of eccentric behavior and social and interpersonal deficits beginning in early adulthood characterize schizotypal personality disorder. Five or more of the following characteristics must be present: inappropriate or constricted affect, lack of close friends, excessive social anxiety, odd beliefs or magical thinking, ideas of reference, unusual perceptual disturbances such as bodily illusions, odd thinking and speech, suspiciousness or paranoid ideation and behavior or appearance that is odd or peculiar. This item, however, surrounds the personality disorder's associated impairment in personality functioning rather than the pathologic traits. Those with schizotypal personality disorder have impairments in personality functioning which manifest by: 1. Impairments in self functioning: Confused boundaries between self and others; distorted self-concept; emotional expression often not congruent with context or internal experience. Unrealistic or incoherent goals; no clear set of internal standards. 2. Impairments in interpersonal functioning: Pronounced difficulty understanding impact of own behaviors on others; frequent misinterpretations of others? motivations and behaviors. Marked impairments in developing close relationships, associated with mistrust and anxiety. Answer A: Patients with avoidant personality disorder exhibit avoidance and fear of interpersonal contact due to feelings of inadequacy and fear of embarrassment. A diagnosis of avoidant personality disorder is made when a pervasive pattern exists and is exhibited by four or more of the following: avoidance of occupational activities that involve interpersonal contact due to fear of criticism or rejection, unwillingness to become involved with others unless certain of being liked, restraint within intimate relationships due to fear of ridicule, preoccupation with being criticized or rejected in social situations, feelings of inadequacy cause inhibited behavior in new interpersonal situations, views of self as inept, unappealing or inferior and reluctance to take personal risks or engage in new activities due to possibility of embarrassment. Answer B: Borderline personality disorder is marked by unstable affect and self-image as indicated by five or more of the following: a pattern of intense and unstable interpersonal relationships, frantic efforts to avoid abandonment, persistently unstable sense of self, impulsive behaviors in two or more areas (sex, spending, substance abuse, reckless driving, binge eating), recurrent suicidal behaviors/threats/gestures or self-mutilation, affect instability and marked mood reactivity, chronic feelings of emptiness, difficulty controlling anger and transient, stress-related paranoia or dissociation. Answer C: Histrionic patients are described as dramatic, flamboyant and attention seeking. Five or more of the following are present: inappropriately seductive or provocative behavior, discomfort in situations in which he/she is not the center of attention, rapidly shifting or shallow expression of emotions, consistent use of physical appearance to draw attention to self, speech that is excessively impressionistic and lacks detail, exaggerated expression of emotion or theatricality, suggestibility or being easily influenced by others or circumstances and consideration of relationships to be more intimate than they really are. Answer D: Schizoid personality disorder is characterized by social withdrawal, a lack of desire for interpersonal relationships and reduced range of affect. Four or more of the following are present: neither desires nor enjoys close relationships including family involvement, chooses solitary activities, has little or no interest in sexual experiences with others, takes pleasure in few activities, lacks close friends, appears indifferent to praise or criticism from others and is emotionally flat, cold or detached. Bottom Line: Personality disorders are pervasive and maladaptive patterns of relating to others that develop by early adulthood and cause impairment in multiple areas. These patterns of beliefs and behaviors are not better accounted for by another mental disorder, the direct effect of a substance or caused by a medical condition. These stable and inflexible behaviors are chronic and best treated with psychotherapy.

Treatment of Mild-moderate Serotonin Syndrome?

Cessation of the offending agents, benzodiazepines, IV hydration/supportive and observation.

typical vs atypical antipsychotics MOA

First generation/typical/neuroleptics- D2 antagonists Second Generation/Atypical: 5HT2A antagonists, fast D2 dissociation

Dystonia

acute onset of sustained involuntary muscular contractions or Spasms impairment of voluntary muscular control commonly in head, face, neck Consider onset when differentiating dystonia and tardive dyskinesia

5HT2A antagonism

atypical anti-psy

Seroquel (quetiapine)

atypical antipsychotic

TMS (transcranial magnetic stimulation)

pulsating magnetic coil isplaced over prefrontal regions of the brain to treat depression with minimal side effects

Opiate withdrawal symptoms

range from mild, flulike discomfort to severe muscle pain, stomach cramps, diarrhea, and unpleasant mood pupils dilated, yawning, hyperactive bowel sounds, piloerection lacrimation. losing fluids

cyp450 enzyme metabolism of clozapine

cyp1a2

An overdose of which antihypertensive mimics opiate intoxication?

Clonidine

What are 3 W's of Personality Disorder Clusters?

Cluster A: Weird Cluster B: Wild Cluster C: Worried

First Generation Antipsychotics are __________ antagonists.

D2

Displacement vs Projection

Displacement: person A has feelings about person B, redirects these feelings onto person/object C Projection: person A has feelings about person B, believes that person B is the one with those feelings

Tx for mania during pregnancy

ECT

Prozac (fluoxetine)

SSRI

Akathesia

inability to remain still; motor restlessness and anxiety

Prominent horizontal nystagmus

thiamine deficiency

When dopamine is in excess in the nigrostriatal pathway it can cause ______________________, such as __________________, ___________________, _______________.

hyperkinetic movements such as tics, chorea, dyskinesias

Bipolar Type 1 (rapidly cycling)

4 episodes within 12 months

CIWA scale

10-item assessment tool that is used for measuring the SEVERITY of symptoms of alcohol withdrawal

Delusional disorder criteria

A. Nonbizarre delusions for 1+month B. Criterion A for schizophrenia has never been met (TH and OH allowed if related to delusional theme) C. Apart from the delusions, functioning is not markedly impaired and behavior not obviously odd or bizarre D. If mood episodes concurrent with delusions, total duration brief relative to duration of delusional periods E. Not due to direct effects of substance or GMC

Lithium toxicity

Narrow Therapeutic Window the likelihood of lithium intoxication is increased when lithium excretion is impaired ●Underlying renal insufficiency ●Effective volume depletion ●Elderly patients (low glomerular filtration rate) Drug- Interactions that increase serum lithium ●Thiazide diuretics ●Non-steroidal anti-inflammatory drugs except aspirin ●Angiotensin converting enzyme inhibitors ●Antibiotics tetracycline and metronidazole

DDX list for depressive patient.

bipolar 1&2 MDD

DDX for Psychotic, Depressed and Anxious chief complaints.

bipolar disorder hypothyroidism, hyperthyroidism, ________due to physiological condition ________ due to substance abuse

Atypical anti-psychotics

clozapine, risperidone, olanzapine, aripiprazole, quetiapine, ziprasidone

Negative sx of schizophrenia

negative symptoms are conceptualized as an absence or diminution of normal processes. Negative symptoms may be primary or secondary. ●flat affect ●anhedonia ●alogia (no speech) ●poor attention ●avolition (apathy)

Lamictal (lamotrigine)

Anticonvulsant Mood Stabilizer

Carbamazepine (Tegretol)

Anticonvulsant- seizures Mania- mixed/acute trigeminal neuralgia

Gabapentin (Neurontin)

Anticonvulsant/Antineuralgic

persistent complex bereavement disorder

Intense symptoms of depression and guilt that persist beyond 12 months following the death of a loved one is indicative of persistent complex bereavement disorder. Not only do symptoms persist beyond 12 months, but they are generally severe in nature and interfere with daily functioning. According to the DSM V, persistent complex bereavement disorder, also called complicated grief disorder Normal grief is limited to 12 months following the loss of a loved one. Feelings of pervasive hopelessness, helplessness, worthlessness, guilt, lack of pleasure, and suicidal ideation are present in patients with depression or complicated grief, but are not components of normal grief.

True or False: the Us preventative services task force recommends against routine depression screening without in-office supportive staff (trained nurses, CBT, psychiatrist)

TRUE The US Preventive Services Task Force recommends that adults should only be screened for depression when there is staff-assisted care and support available to make an accurate diagnosis, offer effective treatment and provide follow-up care.

CD4 cells function

also called T-helper cells. They help to identify, attack, and destroy specific bacteria, fungi, and viruses that cause infections. CD4 cells are also a major target for HIV, which binds to the surface of CD4 cells, enters them, and either replicates immediately, killing the cells in the process, or remains in a resting state, replicating later.

Miosis is ____ (constriction/dilation) of the pupil, mediated by _____ (parasympathetic/sympathetic) stimulation.

constriction/

PMDD (premenstrual dysphoric disorder)

group of symptoms similar to but more severe than PMS, including severe mood disturbances (increase dose one week before menstrual cycle)

Haloperidol (Haldol)

typical antipsychotic

Disruptive Mood Dysregulation Disorder (DMDD)

3 outbursts per week 12 months of A-D present criteria No period longer than 3 months without symptoms

Agitation medications

Ativan Haldol (IV and PO) (B52- with Benadryl)

What is the most common form of child abuse in the us? a. physical b. sexual c. emotional d. neglect

d. Neglect is the most common type of child maltreatment in the United States (78.5%) and is caregiver failure to meet basic nutritional, medical, educational, and emotional needs of a child.

What types of patient should a primary care provider refer to psychiatry?

depressed that's failed 1-2 trials of antidepressants bipolar depressed suicidal depressed depressed with psychotic features

Opiod intoxication triad tx?

●miosis ●Respiratory depression ●CNS depression TX: naloxone (narcan)

MAOIs (monoamine oxidase inhibitors)

-antidepressants that inhibit the action of an enzyme (monoamine oxidase) that is responsible for breakdown of NE and 5-HT -can have fatal side effects if foods rich in amino acid TYRAMINE are consumed -thus, not used unless other classes of medication have failed -Depression with ATYPICAL features respond well to MAOIs

Zoloft (sertraline)

SSRI antidepressant/anxiolytic

A 40-year-old female presents to the clinic complaining of anxiety and work fatigue. History reveals that she often worries that her husband will leave her for a prettier woman. Past medical history is significant for hepatitis C infection with cirrhosis. She is started on paroxetine and returns in six weeks without relief of her symptomology. Which of the following is the most appropriate pharmacologic management? A. augment with lorazepam B. increase paroxetine dose C. switch to clonazepam D.switch to pregabalin E. switch to sertraline

The correct answer is: B This patient is suffering from generalized anxiety disorder (GAD), characterized by persistent worrying along with psychological and physical symptoms that interfere with daily functioning of an individual as demonstrated by fatigue and insomnia in this patient. First line treatment for GAD is based on pharmacologic and/or psychological therapy with cognitive behavioral therapy depending on availability of resources and patient preference. Regarding pharmacologic therapy, guidelines support selective serotonin receptor inhibitor SSRI (paroxetine) or serotonin norepinephrine reuptake inhibitor SNRI (venlafaxine, duloxetine) as first line therapies for which take about 6-8 weeks to show full effect. Benzodiazepines and tricycle anti-depressants (TCA) like amitriptyline are used as second line therapy for patients who do not respond to SSRI or SNRI or in addition to them. Some studies have shown efficacy with pregabalin but it is not a first line therapy for GAD. Answer A: Lorazepam can be added, but not before trying to increase the dose of Paroxetine. Answer B: Paroxetine is a selective serotonin reuptake inhibitor (SSRI) which is first line therapy for generalized anxiety disorder. Typically it is started as low dose therapy and if a patient has even a partial response to it, the dose can be slowly increased. However, if the patient has not shown any signs of improvement after six to eight weeks on a therapeutic dose, the medication can be tapered off and another class of medication can be tried instead. Answer C: Benzodiazepines are not intended for first line or long-term therapy, and not enough time has passed to evaluate whether the current paroxetine dose is efficacious in this patient. Clonazepam is a benzodiazepine which helps curb anxiety by potentiating the effects of endogenous GABA. Benzodiazepines are very useful for acute symptom relief during the period before an SSRI takes effect or in addition to an SSRI for partial responders. Side effects include dependence especially in someone with a history of drug or alcohol abuse. Other side effects include amnesia, drowsiness and impaired psychomotor performance. However, clonazepam has a long half-life (t 1/2 of 30-40 hours) making it a less favorable answer choice in a patient with known liver cirrhosis due to higher potential of drug toxicity. Answer D: Pregabalin works by inhibiting excitatory neurotransmitter release including norepinephrine, serotonin and substance P which is well documented to help patients with chronic pain syndromes. However, exact mechanism of efficacy regarding GAD is unknown. In some trials, pregabalin has been shown to reduce anxiety symptoms in patients with GAD compared to placebo. Side effects include sedation and dizziness. Although it is sometimes better tolerated than benzodiazepines, it would be inappropriate to change from SSRI to pregabalin without waiting for at least 6-8 weeks for full drug response. Answer E: Increasing the dose of Paroxetine should be tried first before switching medications. Bottom Line: Always increase the dose of a medication before adding or switching to other medications.

A 43-year-old female presents to the emergency department with altered mental status. Her boyfriend states that she awoke this morning in her usual state of health but over the course of the morning began acting confused and agitated. She has a history of depression and recently began a new medication, the name of which he cannot recall. She has no history of suicide attempts and no other medical history. Vitals reveal a temperature of 39.7ºC (103.4ºF), pulse of 143/min, respirations 22/min, and blood pressure 158/88 mm Hg. Major swings in her pulse and blood pressure are noted during the exam. Neurological exam is significant for hyperreflexia, sustained clonus, and muscular rigidity. Labs reveal WBC of 13,000 and an elevated creatine phosphokinase. Which of the following medications or combinations of medications most likely led to this patient's current condition? A. chlorpromazine and fluoxetine B. citalopram and paroxetine C.haloperidol D. imipramine and sertraline E. selegiline and fluoxetine

The correct answer is: E The patient is experiencing serotonin syndrome, which is characterized by hyperpyrexia, autonomic instability, neuromuscular excitability, and altered mental status (e.g. confusion, agitation, hallucination). Muscular rigidity, warm flushed skin, mydriasis, and diaphoresis can also be present as well as leukocytosis and elevated CPK on laboratory analysis. It can easily be confused with neuroleptic malignant syndrome (NMS), which similarly presents with altered mental status, fever, and identical lab findings; however, NMS has a slower onset of symptoms (occurring over days, not hours) and is associated with bradykinesia and "lead pipe" neuromuscular rigidity, not hyperreflexia and clonus. The combination of the monoamine oxidase-inhibitor (MAOI) selegiline with any other serotonergic agent, fluoxetine, is most likely to cause serotonin syndrome. Answer A: Chlorpromazine is a first generation antipsychotic and poses an increased risk for NMS, not serotonin syndrome. When used in combination with fluoxetine, a selective serotonin reuptake inhibitor (SSRI), there is no increased risk for SS. Answer B: The combination of two SSRIs, citalopram and paroxetine, is not as likely to cause serotonin syndrome as the combination of an SSRI and MAO-I; therefore, this is not the best answer. Answer C: Haloperidol is an antipsychotic that poses an increased risk for NMS, not serotonin syndrome. Answer D: The combination of a tricyclic antidepressant (imipramine) and an SSRI (sertraline) can possibly lead to serotonin syndrome; however, an MAO-I in combination with another serotonergic drug is a more likely culprit. Bottom Line: MAO-Is pose the greatest risk for serotonin syndrome when taken in combination with other serotonergic medications, herbal supplements, or tyramine and tryptophan-containing drinks and foods. Serotonin syndrome is characterized by hyperpyrexia, altered mental status, neuromuscular excitability and autonomic instability.

Mirtazapine (Remeron) MOA

a2 antagonist

Antisocial personality disorder (APD)

Disregard for societal norms and moral values, little empathy, lack conscience, willing to harm or manipulate others for gain -over-represented in prison populations "SOCIOPATHS" -has nothing to do with avoiding social situations

True or False: Patients with Schizotypal disorder do not want to have interpersonal relationships.

False: they want to. IN CONTRAST to patients with schizoid that want to AVOID forming relationships with other people. A person who appears detached, sheltered, lacks empathy, and does not show their emotions most likely has schizoid personality disorder.

A 20-year-old woman presents to her primary care physician because of excessive daytime sleepiness that started several months ago. She sleeps approximately ten hours nightly but awakens unrefreshed and typically takes an equally unrefreshing nap in the afternoon. Sometimes when she awakens she is confused, disoriented and clumsy. She does not experience unusually vivid dreams or sleep paralysis nor does she fall asleep without warning. She takes no medications, and her family history is unremarkable. She smokes approximately one half pack of cigarettes daily and occasionally drinks a beer or two with friends. On examination she appears tired but well. She is neurologically intact and the remainder of the physical examination is unremarkable. Polysomnogram shows sleep latency to be five minutes with normal REM sleep latency. Which of the following is the most likely diagnosis? A. chronic fatigue syndrome B. circadian rhythm sleep disorder C. hypersomnolence disorder D.narcolepsy E. REM sleep behavior disorder

The correct answer is: C This patient most likely has hypersomnolence disorder, previously known as primary or secondary insomnia in DSM-IV. Diagnostic criteria (abridged) include the following: A. Excessive sleepiness despite at least 7 hours of sleep with at least one of the following: recurrent sleep or lapses of sleep during day, prolonged main sleep episode of >9 hours that is nonrestorative, difficulty being fully awake after abrupt awakening. B. Occurs at least 3x/week for at least 3 months. C. Accompanied by distress or impairment of cognitive, social, occupational, or other important areas of functioning. Answer A: Patients with chronic fatigue syndrome will also have excessive fatigue that is not alleviated by rest; however, these patients will typically complain of a variety of somatic complaints (headache, myalgias, arthralgias, and low-grade fever) as well. Answer B: Circadian rhythm sleep disorder is seen in patients who are unable to sleep during the night but are able to sleep during the day. Answer D: This patient has no characteristic features of narcolepsy such as short REM sleep latency, cataplexy, hypnagogic hallucinations, sleep paralysis. DSM-5 distinguishes narcolepsy - now known to be associated with hypocretin deficiency - from other forms of hypersomnolence (hypersomnolence disorder). Answer E: Patients with REM sleep behavior disorder are not normally paralyzed during REM sleep and tend to act out their dreams. Bottom Line: Primary hypersomnia is characterized by excessive daytime sleepiness in the face of more than adequate nighttime sleep and daytime napping. Often patients awake sleep-drunk with confusion, disorientation, and clumsiness.

A 32-year-old female presents with nausea, bloating, and abdominal pain. She reports a several year history of frequent headaches and chronic low back pain. Since late adolescence she complains of irregular menses and dysmenorrhea. History reveals she previously had neuropathic changes in her lower extremities that have since resolved. The most likely diagnosis is A. factitious disorder B. functional neurological symptom disorder C. malingering D. somatic symptom disorder E.somatic symptom disorder pain predominant

The correct answer is: D Somatization Disorder is now known as somatic symptom disorder and is defined as multiple physical complaints that result in treatment being sought or significant impairment in functioning, with no medical condition or substance fully explaining the symptoms. The main feature of this disorder is a concern with physical symptoms that are attributed to a nonpsychiatric disease. This concern can manifest as one or more somatic symptoms that result in excessive thoughts, feelings, or behaviors related to those symptoms and that are distressing or result in significant disruption of daily life. This unusual constellation of symptoms must not be intentionally produced (as in factitious disorder or malingering) or explained by a known general medical condition. Answer A: In factitious disorder, the patient intentionally produces symptoms and this behavior is motivated by secondary gain in the form of assuming the sick role. Primary gain (monetary incentive, improved quality of life, escape from legal responsibilities) is not the motivation for these patients as it is in cases of malingering. While the question stem does not specifically address patient motivation for presentation, it does meet criteria for somatization disorder based on the presence of pain, gastrointestinal, sexual and pseudoneurological complaints. However, in order to definitively diagnose somatization disorder both factitious disorder and malingering must be ruled out. Answer B: Conversion disorder, now known as functional neurological symptom disorder in DSM-5, is defined by symptoms involving voluntary motor or sensory function that suggest neurological illness, are not intentionally produced but cannot be explained by a medical condition, substance effect or cultural behavior or experience. There are associated psychological factors associated with the symptoms that are often preceded by stress. While this patient does have a history of lower extremity neuropathic changes, the existence of multiple other pain, gastrointestinal and sexual symptoms makes a diagnosis of somatization disorder more appropriate. Answer C: Malingering describes intentionally feigned signs or symptoms of physical or psychological illness that are motivated by primary gain (external incentives such as financial compensation, housing, avoiding responsibilities or obtaining drugs). It should be suspected when there is significant discrepancy between the patient's complaints and objective findings (on physical exam and diagnostic testing), lack of cooperation with diagnosis and treatment, medicolegal context of presentation such as referral by an attorney and in patient's with antisocial personality disorder. Malingering must be ruled out in order for diagnosis of factitious disorder or a somatoform disorder to be made. Answer E: Pain disorder, now known as somatic symptom disorder with predominantly pain symptoms, is diagnosed when the chief complain is significant pain causing distress or impairment, occurs in association with psychological factors and yet is not better explained by another mental illness. A general medical condition can be present that accounts for a portion of the pain but psychological factors are judged to play an important role in the onset, severity, exacerbation or maintenance of the pain. Bottom Line: When a patient presents with somatic complaints that are associated with stress or psychological factors, somatic symptom disorder should be investigated. However, the differential diagnosis must always include known general medical condition (with thorough investigation of symptoms via physical exam and diagnostic testing), substance effect (either medication, toxin, or illicit substance induced illness), factitious disorder (feigning of symptoms for secondary gain) and malingering. COMBANK Insight : In DSM-5, somatoform disorders are now referred to as somatic symptom and related disorders. The DSM-5 classification reduces the number of these disorders and subcategories to avoid problematic overlap. Diagnoses of somatization disorder, hypochondriasis, pain disorder, and undifferentiated somatoform disorder have been removed. Individuals previously diagnosed with somatization disorder will usually have symptoms that meet DSM-5 criteria for somatic symptom disorder, but only if they have the maladaptive thoughts, feelings, and behaviors that define the disorder, in addition to their somatic symptoms. Because the distinction between somatization disorder and undifferentiated somatoform disorder was arbitrary, they are merged in DSM-5 under somatic symptom disorder. Individuals previously diagnosed with hypochondriasis who have high health anxiety but no somatic symptoms would receive a DSM-5 diagnosis of illness anxiety disorder (unless their health anxiety was better explained by a primary anxiety disorder, such as generalized anxiety disorder). Some individuals with chronic pain would be appropriately diagnosed as having somatic symptom disorder, with predominant pain. For others psychological factors affecting other medical conditions or an adjustment disorder may be more appropriate.

A 12-year-old male presents with repetitive eye blinking, facial grimacing, shoulder shrugging, and head jerking that waxes and wanes throughout the day.

Tourette's disorder manifests with an onset before the age of 18 years and presents with both motor and vocal tics at some time during the illness, although not necessarily at the same time. A tic is a sudden, rapid, recurrent, non-rhythmic, stereotypic motor movement or vocalization. The tics occur many times during the day nearly every day or intermittently throughout a period of more than one year. During this period, there is never a tic-free period of more than three consecutive months. Tourette's disorder is believed to be the result of dysfunction in the basal ganglia and related to structures in the midbrain and cortex. Twin studies, adoption studies, and segregation analysis studies all support a genetic etiology for Tourette's disorder, but results for different families varies. Research is still ongoing.

Vagal Nerve Stimulation (VNS)

an emerging biomedical treatment for depression that involves electrically stimulating the vagus nerve with a small battery-powered implant

Risperdal (risperidone)

atypical antipsychotic MOA: fewer extrapyramidal adverse effects than Haldol. Binds to dopamine D2-receptor with 20 times lower affinity than for 5-HT2-receptor. Improves negative symptoms of psychoses and reduces incidence of adverse extrapyramidal effects.

Tardive dyskinesia (TD)

chronic/late onset of irreversible neurologic side effect of antipsychotic medications; characterized by abnormal, involuntary movements such as lip smacking, tongue protrusion, chewing, blinking, grimacing, and choreiform movements of the limbs and feet commonly occurs after yrs of antipsychotic rx use, can also occur if patient stops taking their meds

A CD4 count of <200 cells/microL indicates the ________________________and high susceptibility to opportunistic infections, such as Pneumocystis pneumonia.

clinical stage of AIDS At CD4 counts less than 200 cells/mm3, the immune system can no longer keep opportunistic infections in check. At low CD4 counts, a healthcare practitioner may recommend starting prophylactic treatment for opportunistic infections such as Pneumocystis carinii (jiroveci) pneumonia (PCP) or candidiasis (thrush).

Haldol (haloperidol)

typical antipsychotic MOA:

Ativan (lorazepam)

Benzodiazepine Antianxiety; C-IV

Delirium DSM-5 diagnostic criteria

●Disturbance in attention (reduced ability to direct, focus, sustain, and shift attention) and awareness. ●The disturbance develops over a short period of time (usually hours to days), represents a change from baseline, and tends to fluctuate during the course of the day. ● Disturbance in cognition (memory deficit, disorientation, language, visuospatial ability, or perception) ● Sx not better explained by another preexisting, evolving or established neurocognitive disorder, and do not occur in the context of a severely reduced level of arousal, such as coma ●Evidence from the history, physical examination, or laboratory findings is present that indicates the disturbance is caused by a direct physiologic consequence of a general medical condition, an intoxicating substance, medication use, or more than one cause.

Symptom domains of schizophrenia

●Positive symptoms ●Negative symptoms ●Cognitive impairment ●Mood and anxiety symptoms

A 19-year-old college student presents to her primary care physician complaining of a depressed mood since failing an examination three weeks ago. She has been having multiple awakenings during the night and complains of being unable to go back to sleep. She has been having difficulty concentrating on her studies, and often finds herself thinking that studying is futile, as she will likely fail her next exam regardless. She still enjoys spending time with her friends, but admits to feeling anxious and guilty that she is not studying. She finds herself often thinking about the exam, and wishes to forget about it. She is requesting something to help improve both her mood and concentration. Which of the following is the most likely diagnosis? A. adjustment disorder with depressed mood B. generalized anxiety disorder C. major depressive disorder D.normal reaction to stressors E. persistent depressive disorder

The correct answer is: A Adjustment disorder is a reaction to an event that is more extreme than normal, causes significant impairment in social or occupational (academic) function. The following are the criteria for adjustment disorder: - Emotional response to a stressor within 3 months of inciting event - Clinically significant symptoms as a result - Symptoms do not persist longer than 6 months after stressor has resolved There are five different subtypes of adjustment disorder, each of which are characterized by a specific set of moods and/or behavior: 1) with depressed mood, 2) with anxiety, 3) with mixed anxiety and depressed mood, 4) with disturbance of conduct, 5) with mixed disturbance of emotions and conduct. This patient has depressive symptoms in excess of what would be typically be expected in response to failing an examination. However, she does not meet criteria for major depression and the distress was triggered by a particular stressor (failing the examination), and therefore she can accurately be characterized as adjustment disorder with depressed mood. Answer B: Generalized anxiety disorder requires excessive and uncontrollable anxiety for at least six months, and consists of symptoms of restlessness, fatigue, difficulty concentrating, irritability, muscle tension, and sleep disorder. While this patient has some of the above symptoms, they have only been present for three weeks and are in response to a specific stressor. Answer C: Major depressive disorder requires the presence of 5 of the following criteria to be diagnosed most or all of the time over a 2-week period: depressed mood, anhedonia, change in weight or appetite, sleep pattern changes, psychomotor retardation, fatigue or loss of energy, feelings of worthlessness or guilt, decreased concentration, and either passive or active suicidal ideation. This patient only reports four of the criteria (sadness, insomnia, difficulty concentrating, and guilt) and does not meet the criteria for depression. Answer D: Her reaction is more extreme than is considered normal for the event triggering it, and it is affecting her ability to function as a student. Answer E: What was referred to as dysthymia in DSM-IV now falls under the category of persistent depressive disorder, which includes both chronic major depressive disorder and the previous dysthymic disorder. By definition, dysthymic disorder requires at least two years of persistent symptoms of depressed mood and this patient's symptoms have only been present for the last three weeks, thereby ruling out dysthymia. Bottom Line: An adjustment disorder is a reaction to an event, or stressor, that is more extreme than normal, causes impairment in social or occupational (academic) function, and does not meet criteria for another Axis I disorder.

A 20-year-old man with a history of depression overdoses on his antidepressant and presents to the emergency department by ambulance. On physical examination, his temperature is 38.5°C (101.4°F), heart rate is 110/min, respiratory rate is 15/min, and blood pressure is 100/70 mm Hg. He is sedated and disoriented. His skin is flushed and dry, and his pupils are dilated and reactive to light. There are reduced bowel sounds on auscultation. Which of the following is the best way to determine the severity of his overdose? A. electrocardiography B. prescription refill history C. presence of seizures D.serum drug level E. urine drug level

The correct answer is: A This patient's clinical presentation and physical examination are consistent with tricyclic antidepressant (TCA) toxicity. TCAs inhibit the reuptake of norepinephrine and serotonin, increasing their availability at the synapse. They can be lethal in overdose and it is very important to assess a patient's suicide risk prior to prescribing these medications, as a 1-week supply can be lethal, which is why TCAs are rarely used as first-line agents anymore. TCA toxicity has a number of manifestations including the 3Cs: Convulsions, Coma, and Cardiotoxicity and anti-HAM: (H) anti-histaminic: sedation (A) anti-adrenergic: cardiovascular side effects, such as tachycardia, arrhythmias, and orthostatic hypotension (M) anti-muscarinic: dry mouth, urinary retention, constipation, tachycardia, blurred vision. This patient is manifesting signs of both anti-histaminic and anti-muscarinic (or anticholinergic) properties: sedation, tachycardia, dry skin, mydriasis, and decreased bowel sounds. It is therefore important to immediately do a 12-lead ECG to rule out cardiotoxicity as TCAs block sodium channels in the heart, leading to prolongation of the QRS interval (>100 msec) and wide-complex tachyarrhthmyias. The best treatment of TCA overdose is intravenous sodium bicarbonate. Answer B: Although a pill container can give you an idea of what medication he took it is inaccurate to use as a determinant for overdose severity. Answer C: Seizures may occur with TCA overdose; however, ECG changes are the most life-threatening manifestation. Answer D: Serum drug levels are useful, but ECG changes are the best measure of severity of overdose effect. Answer E: Urine drug screens are not useful for judging severity since they may not correlate with clinical symptoms.

A 45-year-old male presents with recurrent heart palpitations, restlessness, irritable mood, and difficulty sleeping. He reports problems both falling and staying asleep. History reveals that he drinks several sodas and smokes a half pack of cigarettes per day. Observation reveals the patient constantly paces and has difficulty sitting still. Physical examination reveals a heart rate of 105/min and is otherwise normal. Which of the following is the most appropriate next step in management? A. alprazolam B. cognitive behavioral therapy C. lifestyle modifications D.propranolol E. zolpidem

The correct answer is: C The case describes a patient experiencing somatic and psychiatric complaints directly related to the effects of excessive caffeine and nicotine consumption. Caffeine intoxication is characterized by insomnia, restlessness, agitation, muscle twitching and diuresis. Nicotine intoxication causes insomnia, anxiety and restlessness as well. Excessive caffeine and nicotine intoxication can both result in cardiac arrhythmia. The sleep disorder described would thus be classified as substance-induced sleep disorder. Answer A: Alprazolam is a short acting benzodiazepine approved for the treatment of anxiety and panic disorder. While this patient is experiencing symptoms of anxiety including psychomotor agitation, irritability and insomnia, these symptoms are better accounted for by the physiological effects of the caffeine and nicotine he is consuming. As a result, he cannot be diagnosed with a formal anxiety disorder unless his symptoms persist despite discontinuation of these substances. Benzodiazepines must be carefully prescribed in light of their potential for physical dependence and will not adequately treat the patient's symptoms if the offending cause (caffeine and nicotine) is not addressed. Answer B: CBT is appropriate for many mental disorders, however, this patient most likely has a substance use disorder such as recurrent intoxications at this time. Lifestyle modifications should be pursued initially before more time-consuming CBT is initiated. Answer D: Propranolol is a beta blocker sometimes used for panic disorder. It may be appropriate for this patient, but lifestyle modifications and diagnostic studies such as an EKG should be completed initially. Answer E: Zolpidem is a non-benzodiazepine sedative-hypnotic that interacts with a subtype of the benzodiazepine receptor and is indicated for the short-term treatment of primary insomnia. In order to diagnose primary insomnia the sleep disorder must not be due to the direct physiological effects of a substance (in this case caffeine and nicotine). The evaluation of insomnia must always include investigation for primary psychiatric or physical illness prior to use of pharmacotherapy as risk of dependency increases with duration of treatment. Insomnia is often a symptom of a disorder rather than a disorder itself. Bottom Line: In order to diagnose any of anxiety disorder, mood disorder or sleep disorder the Diagnostic and Statistical Manual of Mental Disorders (DSM-IV-TR) specifies that the symptoms not be better accounted for by the direct physiological effects of a substance. Anxiety, insomnia and psychomotor agitation are common symptoms of caffeine and nicotine intoxication. COMBANK Insight : Substance use disorders have been restructured for consistency and clarity in DSM-5. The categories of substance abuse and substance dependence have been eliminated and replaced with an overarching new category of substance use disorders - with the specific substance used defining the specific disorders. "Dependence" has been easily confused with the term "addiction" when, in fact, the tolerance and withdrawal that previously defined dependence are actually very normal responses to prescribed medications that affect the CNS and do not necessarily indicate the presence of an addiction.

A 29-year-old female complains that she has been "down in the dumps." History reveals that she has poor self-image and has felt "like a complete failure" for the past five years and that she is worthless. Having felt this dejected for so many years, she can only identify brief periods lasting up to a couple of weeks when her mood has temporarily brightened in the past. She states her family members describe her as chronically pessimistic and indecisive. The most likely diagnosis is A. adjustment disorder B. bipolar type I disorder C. bipolar type II disorder D.cyclothymic disorder E. persistent depressive disorder

The correct answer is: E Dysthymic disorder, now known as persistent depressive disorder (dysthymia) in DSM-5, represents a consolidation of chronic major depressive disorder and dysthymic disorder. Criteria (abridged) include the following: A. Depressed mood for most of the day, for more days than not, as indicated by either subjective account or observation by others, for at least 2 years. B. Presence, while depressed, of two (or more) of the following: poor appetite or overeating, insomnia or hypersomnia, low energy or fatigue, low self-esteem, poor concentration or difficulty making decisions, feelings of hopelessness. C. Individual never has been without symptoms for more than 2 months at a time. D. Criteria for a major depressive disorder may be continuously present for 2 years. Answer A: Adjustment disorders describe the development of emotional or behavioral symptoms within three months of the onset of an identifiable stressor. This stress-related distress or significant impairment is not better accounted for by another Axis I disorder and it does not persist beyond six months of termination of the stressor. The symptoms are characterized as depressed mood, anxiety, mixed anxiety and depressed mood or mixed disturbance of emotions and conduct. The question stem does not allude to a specific stressor and rather describes a chronic pattern of clinical depression rather than an acute episode. Adjustment disorders have been re-conceptualized as a heterogeneous array of stress-response syndromes that occur after exposure to a distressing (traumatic or non-traumatic) event, rather than a residual category for individuals who exhibit clinically significant distress but whose symptoms do not meet criteria for a more discrete disorder (as in DSM-IV). Answer B: A history of at least one manic episode is required for a diagnosis of bipolar I disorder to be made. A manic episode is characterized by expansive, elevated or irritable mood lasting at least one week or to a degree requiring hospitalization and is associated with three or more of the following (four if mood is irritable): decreased need for sleep, distractibility, increase in goal-directed activity, grandiosity, racing thoughts or flight of ideas, pressured speech and excessive risk-taking behavior (such as gambling, foolish financial investments or indiscriminate sexual activity). While the presence of major depression is an important part of bipolar disorder and often dominates the clinical picture, there must be evidence of a current or past manic episode as to differentiate from major depression or dysthymia. The question stem provides no evidence of current or past mania but rather a chronic depressed state best characterized as dysthymic disorder. The DSM-IV diagnosis of bipolar I disorder, mixed episodes - requiring that the individual simultaneously meet full criteria for both mania and major depressive episode - is replaced with a new specifier "with mixed features." Answer C: A history of at least one current or past hypomanic episode is required for a diagnosis of bipolar II disorder to be made. Hypomania is similar to mania in that both share the same clinical symptoms (elevated, expansive or irritable mood combined with decreased need for sleep, distractibility, increased goal-directed activity, grandiosity, racing thoughts, pressured speech and excessive risk-taking behavior) but differ in duration and severity - the minimum duration for a hypomanic episode is four days and the severity of the symptoms does not necessitate hospitalization. The question stem does not mention current or past symptoms consistent with hypomania but rather a history of chronic depression consistent with dysthymic disorder. Answer D: Cyclothymic disorder is a chronic mood disorder with (abridged) criteria that include: A. Numerous periods with hypomanic symptoms that do not meet hypomanic episode criteria for at least 2 years AND with numerous periods of depressive symptoms that do not meet criteria for major depressive episode. B. Individual has not been without symptoms for more than 2 months at a time. Bottom Line: Dysthymic disorder is now known as persistent depressive disorder and can be diagnosed in those with depression for at least 2 years. COMBANK Insight : ?What was referred to as dysthymia in DSM-IV now falls under the category of persistent depressive disorder in DSM-5, which includes both chronic major depressive disorder and the previous dysthymic disorder.

A 30-year-old woman with a history of depression, anxiety, and one previous suicide attempt is brought to the emergency department by her husband. He states that he found her on the floor of their bathroom with several empty pill bottles. He is unsure what medications were in the bottles. She is unresponsive to all but deep pain. Her temperature is 38.5°C (101.3°F), pulse is 150/min, respirations are 6/min, and blood pressure is 90/60 mm Hg. Physical examination reveals that her skin is dry and flushed. An electrocardiogram is obtained displaying a wide-complex tachycardia. After securing her airway and ensuring IV access, which of the following is the best next step? A. amiodarone B. CT of the head C. defibrillation D.flumazenil E. sodium bicarbonate

The correct answer is: E This patient has symptoms of tricyclic antidepressant (TCA) overdose (wide complex tachycardia, hyperthermia, hypotension, flushed and dry skin, hypoventilation) and should be treated with bicarbonate as soon as IV access is obtained. TCAs inhibit the reuptake of norepinephrine and serotonin and their side effects include antihistamine properties (sedation), antiadrenergic properties (cardiovascular effects, orthostatic hypotension, tachycardia), and antimuscarinic effects (dry mouth, constipation, urinary retention, blurred vision). TCAs are lethal in overdose with the major complications being convulsions, coma, and cardiotoxicity. On ECG, the hallmark of TCA toxicity is a widened QRS of greater than 100 msec), with the degree of QRS widening correlating to the severity of TCA overdose. This patient should receive sodium bicarbonate with a goal blood pH of 7.50 -7.55. Alkalinization of the serum uncouples Na+ from the myocardium and also enhances drug elimination by the kidney. Sodium bicarbonate helps overcome the Na+ blockade, which helps to correct hypotension and prevent arrhythmias, and the alkalinization of the serum helps reverse the toxic effects of the TCA. Answer A: Amiodarone and other anti-arrhythmics (procainamide, quinidine, disopyramide, moricizine, flecainide, and propafenone) are contraindicated in TCA overdose because of the risk of QT prolongation and torsades de pointes. Answer B: This patient is unstable and should be given sodium bicarbonate immediately. Head CT is inappropriate at this time, as head trauma would not typically cause her current symptoms. Answer C: There is no indication for defibrillation at this time. The abnormalities on ECG should resolve after administration of sodium bicarbonate. Answer D: Flumazenil is generally contraindicated in patients with TCA overdose because it may precipitate seizures. It is used to treat benzodiazepine overdose, and should only be used in patient's who are benzodiazepine naive, as it may precipitate life-threatening seizures in patients who are addicted to benzodiazepines. Bottom Line: TCA overdose presents with wide complex tachycardia, hyperthermia, hypotension, flushed and dry skin, and hypoventilation. It is treated with sodium bicarbonate.

A 35-year-old woman with a history of depression controlled with psychotherapy is started on citalopram because of worsening mood symptoms and passive suicidal ideation. Two weeks later she begins to feel much better, waking later in the morning. The following week she decides to take on several new projects at work and then quits her job after finishing them because her "talents are wasted there." She tries calling the Secretary of State because she believes she has developed the perfect diplomatic formula to achieve world peace. She is arrested trying to sneak into the State Department and then admitted to the psychiatric ward for treatment. When she arrives on the ward she is sleeping two or less hours nightly and insists that she be let go because seeing the Secretary of State "is a matter of life or death for the entire world." On mental status examination, her speech is loud and pressured and her thought processes are tangential. Urine drug screen is negative. Her symptoms have been present for the past nine days. Which of the following is the most likely diagnosis? A. bipolar I disorder B.bipolar II disorder C. major depressive disorder with psychotic features D. schizoaffective disorder E. schizophrenia

The correct answer is: A Bipolar disorders are mood disorders that present with episodes of both depression and either mania (bipolar 1) or hypomania (bipolar 2). Mania is characterized by elevated, expansive, or irritable mood that is different than patient's usual mood state. True manic episodes, which are necessary for a diagnosis of Bipolar 1, require a continuous changed in mood for at least 1 week, with at least three of the following: inflated self-esteem, decreased need for sleep, increased speech, racing thoughts, psychomotor agitation, and excessive involvement in pleasurable activities. This patient's grandiosity, euphoria, lack of need for sleep, and tangentiality and length of episode (9 days) illustrate an acute manic episode, which, by itself defines bipolar I disorder. In this case starting citalopram likely triggered her mania, as patient's with bipolar disorder must be started on mood stabilizer, which can later be used in conjunction. The DSM-5 criteria for a manic episode requires the following: A. A distinct period of abnormally and persistently elevated, expansive, or irritable mood and abnormally and persistently increased activity or energy, lasting at least 1 week and present for most of the day, nearly every day. B. During this period three of the following symptoms have persisted representing a noticeable change from usual behavior: inflated self-esteem or grandiosity, decreased need for sleep, more talkative, flight of ideas, distractibility, increased goal-directed activity, or excessive involvement in activities that have a high potential for painful consequences. C. The episode is sufficiently severe to cause marked impairment in social or occupational functioning or to necessitate hospitalization to prevent harm to self or others, or there are psychotic features. The DSM-5 criteria for a hypomanic episode requires the following: A. A distinct period of abnormally and persistently elevated, expansive, or irritable mood and abnormally and persistently increased activity or energy, lasting at least 4 consecutive days and present for most of the day, nearly every day. B. During this period three of the following symptoms have persisted representing a noticeable change from usual behavior: inflated self-esteem or grandiosity, decreased need for sleep, more talkative, flight of ideas, distractibility, increased goal-directed activity, or excessive involvement in activities that have a high potential for painful consequences. E. The episode is NOT severe enough to cause marked impairment in social or occupational functioning. Answer B: In bipolar II disorder patients will experience hypomania rather than true mania. Hypomania differs from mania in that it does not lead to significant impairment, lasts for 4 days, has no psychotic features, and does not require hospitalization. However, as described above, this patient meets criteria for true mania. Answer C: Mania excludes the diagnosis of major depressive disorder. She no longer has symptoms of major depressive disorder after treatment with a selective serotonin inhibitor (SSRI), eliminating this diagnosis. Answer D: Patients with schizoaffective disorder must have episodes meeting criteria for a mood disorder (depression and/or mania) and have a period of at least two weeks in duration consistent with psychosis without evidence of a mood disorder. Though this patient's current symptoms may be mistaken for psychosis associated with schizophrenia, her loud pressured speech and tangentiality are more consistent with a manic episode. Also, her symptoms started following treatment with an SSRI, which is also more consistent with a manic episode. Answer E: As described above, this patients symptoms are more consistent with mania, not psychosis. Bottom Line: Bipolar I disorder is characterized by mania: acute episodes of grandiosity, little need for sleep, pressure speech, and tangential thought. Almost all patients spend much more time depressed than manic. COMBANK Insight : Diagnostic criteria for bipolar disorders now include both changes in mood and changes in activity or energy. The DSM-IV diagnosis of bipolar I disorder, mixed episodes - requiring that the individual simultaneously meet full criteria for both mania and major depressive episode - is replaced with a new specifier "with mixed features." Particular conditions can now be diagnosed under other specified bipolar and related disorder, including categorization for individuals with a past history of a major depressive disorder whose symptoms meet all criteria for hypomania except the duration criterion is not met (i.e., the episode lasts only 2 or 3 days instead of the required 4 consecutive days or more). A second condition constituting another specified bipolar and related disorder variant is that too few symptoms of hypomania are present to meet criteria for the full bipolar II syndrome, although the duration, at least 4 consecutive days, is sufficient. Patients with bipolar disorder should be treated with either a mood stabilizer, or a combination of both a mood stabilizer and an atypical or second generation antipsychotic. Lithium and valproic acid are generally the first line mood stabilizers, with carbamazepine and oxcarbazepine as second-line choices. Antidepressants can also be added to a mood stabilizer if depression does not respond to a mood stabilizer alone. Supportive or insight-oriented therapy can also help after the patients have been stabilized. Answers A & B & D: Antidepressants should be used with care in bipolar disorder, as they may precipitate mania. However, can be added on as a second agent when patients have acute depressive episodes. They should not be used in acute mania as they can exacerbate symptoms. Atypical antipsychotics, such as olanzapine, may be used for acute manic episodes. Haloperidol is frequently used in the hospital setting for treatment of acute agitation that may be present in manic episodes. Answer C: Lamotrigine can be used as maintenance therapy, not acute mania. Bottom Line: Valproic acid, lithium, and atypical antipsychotics are FDA approved first-line treatments for acute mania.

A 2-year-old male presents with his mother who complains the child is not eating or drinking enough for the past two weeks. The mother is afraid he may be losing weight and feels there is something wrong because he has become less active lately. Past medical history is negative for any medical problems except for a low birth weight although he was born at full-term. Vitals are within normal limits. A plotted growth chart reveals a below average weight and height for his age. Physical examination reveals hypotonia, microcephaly, wide-set eyes, round-shaped face, and low-set ears. While being examined, the patient cries with a high-pitched crying sound. The most likely diagnosis is A. Cri-du-chat syndrome B. Down syndrome C. fetal alcohol syndrome D.fragile X syndrome E. Prader-Willi syndrome

The correct answer is: A Most cases of Cri-du-chat (cat cry) syndrome are due to a partial deletion of the short arm of chromosome 5 at 5p15.3, which is the critical area that results in the high-pitched cat-like crying. The deleted chromosome is usually of paternal origin in more than 80% of cases. Few cases are due to parental translocation of 5p. Patients have an early-onset of the cat-like crying, and this may resolve fairly quickly. Cat-like crying occurs due to vocal cord abnormalities. Other features found in patients include: low birth weight, failure to thrive, hypotonia, psychomotor retardation, severe intellectual disabilities, microcephaly (small-sized head), epicanthal folds (skin of the upper eyelid covering the medial, inner corner of the eye), hypertelorism (abnormally increased distance between the eyes), round-shaped face, downslanting palpebral fissures, broad nasal bridge, and low-set ears. Most children will have low weight or short height for age. Patients may also have cardiac abnormalities. In this case scenario, the patient had low birth-weight but other abnormalities did not manifest until a year or so later. He has the characteristic high-pitched, cat-like crying which is only found in Cri-du-chat syndrome. He also has failure to thrive, hypotonia, psychomotor retardation, microcephaly, round-shaped face, low-set ears, wide-set eyes, all of which are found in Cri-du-chat syndrome. Answer B: Down syndrome, also known as Trisomy 21, occurs due to an extra copy of chromosome 21. It is the most common genetic cause of mental retardation. The average IQ amongst individuals with Down syndrome is in the 25-50 range. Common physical features include: a characteristic facial appearance (upslanting), palpebral fissures, epicanthal folds (skin of the upper eyelid covering the medial, inner corner of the eye), low-set ears, a flat nasal bridge, a protruding tongue, a short neck, Brushfield spots on iris (small spots on the periphery of the iris due to aggregation of connective tissue), short stature, and a single palmar crease. Patients with Down syndrome are also more prone to develop hypothyroidism, congenital heart disease, and duodenal atresia. Alzheimer's dementia is also more prevalent amongst adults with Down syndrome. Psychiatric manifestations include: non-specific behavioral issues, anxiety, and depression. Answer C: Fetal alcohol syndrome (FAS) refers to a cluster of symptoms found in children with in-utero exposure to alcohol, and the severity of symptoms correlates with the severity of the mother's alcohol intake. Children with FAS have a characteristic facial appearance which comprises of microcephaly (small-sized head), flattened mid-face, epicanthal folds (skin of the upper eyelid covering the medial, inner corner of the eye), flattened philtrum (space between nose and upper lip), thin upper lip, micrognathia (small jaw). Infants with FAS have problems with pre- and post-natal growth deficiencies, and may also have cardiac abnormalities. Psychiatric symptoms include: mental retardation, ADHD, oppositional behaviors, depression, anxiety, and post-traumatic stress symptoms due to psychosocial stressors. Answer D: Fragile X syndrome is a maternally inherited condition due to an increased number of triplet CGG repeats in the FMR1 gene located on chromosome Xq27.3. It is the most common inherited form of mental retardation. Down syndrome, on the other hand, is the most common form non-inherited mental retardation. Although fragile X syndrome is more often described in males, it may also occur in carrier females, ranging from being asymptomatic to having severe disease. Physical features include: characteristic facial appearance of a large head with a long face, prominent jaw and forehead, and large ears. Patients also develop large-sized testes. They have connective tissue disease and hypotonia. Psychiatric symptoms include moderate to severe mental retardation, pervasive developmental disorder, ADHD, and mood lability problems. Answer E: Prader-Willi syndrome (PWS) occurs due to the absence of a critical region on paternally-inherited chromosome 15q11-q13. This genetic area undergoes the process of imprinting, which is the differential expression of the region based on whether the chromosome is of paternal or maternal origin. The facial appearance in PWS is characterized by upslanting, almond-shaped eyes, thin upper lips, and down- turned mouth. Other features include: light skin and hair coloration, small hands and feet, small external genitalia (hypogonadism), prominent low tone, and failure to thrive in infancy. PWS is also characterized by hyperphagia, which develops in early childhood and leads to obesity, but can be controlled with behavioral techniques. Patients with PWS have high pain thresholds and rarely vomit. Psychiatric symptoms seen in patients with PWS include: obsessions, compulsions, temper tantrums, skin-picking, anxiety, depression, and psychosis. Patients may also demonstrate a decreased IQ or specific learning disabilities accompanied by areas of relative strength (such as completing jigsaw puzzles). On the other hand, deletion of the same region on the maternally inherited chromosome results in Angelman syndrome, which is characterized by severe intellectual disabilities, postnatal microcephaly, seizures, frequent laughter and smiling, excitable personalities, hypermotoric behaviors, attention problems, decreased speech or absence of speech output, ataxia, etc. Bottom Line: Cri-du-chat syndrome occurs due to a partial deletion of the short arm of chromosome 5 at 5p15.3, and is characterized by high-pitched, cat-like crying, low birth weight, failure to thrive, hypotonia, psychomotor retardation, severe intellectual disabilities, microcephaly, hypertelorism and other features. COMBANK Insight : On both the COMAT and the COMLEX, answer choices will be presented in alphabetical order. Proper nouns and eponyms will be presented in upper case and other words in lower case as if the answers are completing the sentence of the final question above.

A 47-year-old male presents with a depressive episode with associated fatigue and hair loss. History reveals bipolar disorder type II, manic predominant, treated with lithium. Physical examination demonstrates dry skin, patchy hair loss of the scalp, mild bilateral ankle edema, and delayed relaxation of patellar deep tendon reflexes. Structural examination reveals tissue texture changes to T2 with an associated upper thoracic flexion hump. The most likely cause is A. alopecia areata B. hypogonadism C. hypothyroidism D.iron deficiency anemia E. vitamin B12 deficiency

The correct answer is: C Lithium is a common medication in the treatment of bipolar disorder. Common side effects of lithium include drowsiness, hair loss, weight gain, tremor, diabetes insipidus (increased thirst and urination) and decreased thyroid function. Secondary hypothyroidism can present with weight gain, edema, bradycardia, goiter and delayed deep tendon reflex relaxation. Thyroid studies should be routinely monitored for a patient on lithium. Answer A: Alopecia areata causes patchy hair loss in a usually healthy individual. Hair loss occurs in a single, well circumscribed oval patch and is non-scarring. Episodes have spontaneous onset and remission and is attributed to autoimmune origin. Answer B: Hypogonadism can refer to either decreased production of sperm or testosterone. Decreased vigor and libido and depressed mood are common symptoms of hypogonadism and occur within a few weeks of its development, but they are relatively nonspecific. Decreased muscle mass and body hair are less common, but do not occur for a year or many years. Hot flashes occur only when the degree of hypogonadism is severe. Gynecomastia, tender or not, is more likely to occur in primary than secondary hypogonadism, as is infertility. Answer D: Iron deficiency anemia has symptoms of fatigue, hair loss, constipation with sensations of numbness and burning. Edema and deep tendon reflex changes are not common in this disease. Answer E: Vitamin B12 deficiency presents with macrocytic anemia, peripheral neuropathy and ataxia. In severe vitamin B12 deficiency, the dorsal columns of the spinal cord degenerate, resulting in paresthesia, loss of vibration, and absent reflexes. Bottom Line: Secondary causes of depression in a patient with history of psychiatric diagnosis is an important inclusion in the differential diagnosis. Lithium, a common medication in treatment of bipolar disorder, can depress thyroid function resulting in secondary hypothyroidism and presenting as symptoms of depression.

A 22-year-old female presents to the emergency department with sweating, palpitations, and chest pain. Her symptoms began acutely 30 minutes ago, shortly after she finished having lunch which included beer, aged cheese, and cured meat. Past medical history is only significant for depression, for which she takes phenelzine. Vitals reveal she is afebrile with a heart rate of 120/min, respirations at 18/min, and blood pressure of 190/110 mm Hg. She appears very anxious, her pupils are dilated and minimally responsive to light, and her skin is cool to the touch. What is the most likely pathologic mechanism causing this patient's symptoms? A. blocked post-synaptic reuptake of catecholamines B. decreased stimulation of GABA receptors C. increased release of catecholamines at nerve synapses D.increased synthesis of catecholamines E. stimulation of catecholamine receptors

The correct answer is: C This patient is most likely experiencing a hyperadrenergic crisis from being on the monoamine oxidase inhibitor (MAOI), phenelzine. Patients usually present with hypertensive urgency/emergency which can lead to possible stroke or myocardial infarction. Patients taking MAOIs need to avoid tyramine-containing foods, including cured meats or fish, beer, red wine, all cheeses except cottage and cream cheeses, and overripe fruits. Many over-the-counter pain and flu medications need to be avoided as well. Hyperadrenergic crises occur because MAOIs inhibit the gastrointestinal monoamine oxidase system, which normally detoxifies tyramine containing foods. Consequently, tyramine acts as an indirect sympathomimetic, causing the increased release of endogenous catecholamines at nerve synapses which can lead to a hyperadrenergic crisis. Answer A: Tricyclic antidepressants block the reuptake of catecholamines but is not involved in this scenario. Answer B: MAOIs have no known action on the GABA system and do not induce catecholamine synthesis. Answer D: When released into the synaptic space, neurotransmitters are either reabsorbed into the proximal nerve and metabolized by monoamine oxidase (MAO) or destroyed by catechol-o-methyl transferase (COMT) in the synaptic cleft. MAOIs do not induce catecholamine synthesis but rather increase endogenous concentrations of norepinephrine, dopamine, and serotonin through inhibition of the enzyme (monoamine oxidase) responsible for the breakdown of these neurotransmitters. MAOIs inhibit breakdown of the neurotransmitters norepinephrine, dopamine, and serotonin; when ingested in excess it results in hypertension, tachycardia, tremors, seizures, and hyperthermia. Answer E: Direct sympathomimetics and dopamine agonists (bromocriptine, ropinirole) stimulate catecholamine receptors. MAOIs do not directly interact with catecholamine receptors. Bottom Line: Tyramine-containing foods cause hypertensive emergency/urgency in the setting of monoamine oxidase inhibitor (MAOI) use because tyramine is normally broken down by monoamine oxidase. Tyramine acts as an indirect sympathomimetic, causing the increased release of endogenous catecholamines at nerve synapses

Cyclothymic Disorder (Cyclothymia)

At least 2 years of sub-threshold hypomanic and depressive episodes Not symptomless for 2 months or more No full episode in first 2 years

Transference

(psychoanalysis) the process whereby emotions are passed on or displaced from one person to another

Schizophrenia diagnostic criteria

- criteria A: requires *at least 2 of the following*, and at least 1 of these should include 1-3 (first 4 are "positive symptoms"): 1. Delusions 2. Hallucinations—often auditory 3. Disorganized speech 4. Disorganized or catatonic behavior 5. Negative symptoms (affective flattening, avolition, anhedonia, asociality, alogia) - criteria B: *marked decline in functioning* in work, interpersonal relations, and self-care markedly below pre-morbid level - criteria C: lasting *> 6 months*

Bipolar 1 vs Bipolar 2

1 - manic episodes, depressive episodes common but not required for diagnosis 2 - hypomanic(4-7days) episodes, >1 major depressive episodes required

Conduct Disorder (CD) criteria DSM-5

At least 3 of the listed criteria within the past 12 months, with one criterion being within 6 months. -aggression -destruction -deceitfulness -violations of rules/laws/norms -under the age of 18 Conduct disorder is the pediatric version of antisocial personality disorder.

cocaine UDS duration

3 days

Second Generation Antipsychotics are ___________antagonists.

5HT2A -also secondary d2 antagonism also called atypical antipsychotics/serotonin-dopamine antagonists

CD4 count ranges

A normal CD4 count ranges from 500-1,200 cells/mm3 in adults and teens. A low CD4 count indicates that your immune system has been affected by HIV and/or the disease is progressing. The CDC considers people who have an HIV infection and CD4 counts below 200 cells/mm3 to have AIDS (stage III HIV infection), whether or not they have any signs or sx

A 17-year-old boy is brought to the emergency department by his friends. They were at a house party and found their friend unconscious after he ingested an unknown substance. On physical exam, vital signs are temp 96F, heart rate 50, respiratory rate of 4, and oxygen saturation 92% on room air. The boys pupils are miotic. On lung auscultation, there are crackles bilaterally. You administer supplemental oxygen. Which of the following is the most appropriate clinical intervention?

ABC's and Intravenous naloxone are an appropriate first step in managing a patient with acute heroin (opiate) overdose

What are the symptoms/physical exam findings of serotonin syndrome?

Agitation Diaphoresis Confusion Mydriasis Clonus Disorientation Difficulty concentrating Diaphoresis Hallucinations Hyperreflexia Fever Seizure Tremors SS is the overactivation of both the central and peripheral serotonin receptors as a result of high levels of serotonin

What tests might you order when admitting a psychiatric patient?

CBC TSH CMP UDS B12/folate RPR/HIV/CD4 blood alcohol level pregnancy test if appropriate head ct HX and COLLATERAL ARE ESSENTIAL IF AVAILABLE

Names of Typical Antipsychotics/ Neuroleptics

Chlorpromazine (Thorazine) Fluphenazine (Prolixin) Thioridazine (Mellaril) Haloperidol (Haldol)

Narcissistic Personality Disorder (NPD)

Cluster B A personality disorder primarily characterized by an unrealistic, inflated sense of self-importance and a lack of sensitivity to the needs of other people. "I'm so attractive, talented and smart" "my ideas are the best, I'm the best and you should support me without question" ● these pt have a fragile ego and self-esteem ●Lash out whenever they feel slighted

Drug of Abuse with no false positives when testing metabolites in urine

Cocaine

Diabetic Ketoacidosis (DKA)

Diabetic ketoacidosis is typically characterized by hyperglycemia over 250 mg/dL bicarbonate level less than 18 mEq/L pH less than 7.30, with ketonemia and ketonuria

Your patient is psychotic....List some DDX

DDX ●bipolar type 1: psychotic features ●schizophrenia ●bipolar type 2: manic episode with psychotic features ●major depressive disorder with psychotic features ●brief psychotic disorder ●schizoaffective disorder ●schizophreniform ●substance induced psychotic disorder ●psychosis due to other physiological condition ●delirium/dementia

Which dopaminergic pathway is associated with negative symptoms?

Decreased dopamine in the mesocortical pathway is postulated to be responsible for negative and depressive symptoms of schizophrenia. Nicotine releases dopamine in the mesocortical pathways alleviating negative symptoms (self-medication hypothesis).

Confusion Assessment Method (CAM)

Delirium assessment Requires features 1 and 2 and either 3 or 4: 1. Acute change in mental status and fluctuating course - Hours to Days 2. Inattention 3. Disorganized thinking or 4. Altered level of consciousness

Which of the following is untrue of the nigrostriatal pathway? ● a) The nigrostriatal pathway mediates motor movements. ●b) Blockade of dopamine D2 receptors in this pathway can lead to dystonia, parkinsonian symptoms and akathisia. ● c) Projects from the hypothalamus to the anterior pituitary. ●d) Hyperactivity of dopamine in the nigrostriatal pathway is the postulated mechanism in hyperkinetic movement disorders such as chorea, tics and dyskinesias. ●e) Long-standing D2 blockade in the nigrostriatal pathway can lead to tardive dyskinesia.

FALSE c) Projects from the hypothalamus to the anterior pituitary The nigrostriatal pathway projects from the dopaminergic neurons in the substantia nigra to the basal ganglia/striatum.

Generalized Anxiety Disorder (GAD)

Generalized anxiety disorder (GAD) is a common psychiatric disorder often seen in the primary care setting. It is characterized by excessive and persistent worrying that occurs more days than not for six or more months. Other clinical manifestations include insomnia, headaches, difficulty relaxing, and fatigue. The anxiety symptoms experienced with GAD are difficult to control and cause significant distress and impairment in activities of daily living. GAD is two times more common in women than in men and is the most common psychiatric disorder seen in the elderly.

Agitation medications in Grady

Haldol - from psychosis Ativan - other agitation (antipsychotic and benzo available) -order in both IM/PO form use the least restrictive version

Requirements to fill out a 1013.

Harm to self harm to others not able to care for themselveds + signs/sx of mental health illness

Hyperprolactinemia

Higher-than-normal prolactin levels, which may result in spontaneous breastmilk production and amenorrhea. Causes include pituitary tumors and some pharmaceuticals (commonly antipsychotics)

Delirium tremens (DTs)

In delerium tremens, patients will have gross tremors, marked visual hallucinations, profound confusion and agitation. Withdrawal symptoms can begin within 6 hours of cessation of alcohol intake and typically peaks after 24-36 hours of abstinence.

What lab findings are commonly seen in Neuroleptic Malignant Syndrome?

Increased creatine kinase (50-100% of cases) Leukocytosis (70-98% of cases)

Different Classes of Antidepressants

MAOIs: phenelzine, tranylcypromine TCAs: amitriptyline, imipramine, clomipramine SSRIs: fluoxetine, paroxetine, sertraline Others: bupropion, mirtazapine, trazodone, venlafaxine

Lithium indications/ SE/ MOA

MOA ●neuroprotective effects (structural) ●inhibits excitatory neurotransmitters such as dopamine and glutamate ●lithium induces downregulation of NMDA receptors (GABA)

A 17-year-old woman presents to your clinic with a chief complaint of sadness, loss of appetite, and the inability to sleep. She is having a hard time focusing at work and she is tired all of the time. She tells you that for the past month she has been hearing voices telling her to harm herself. She is interested in treatment. What is the most appropriate first-line therapy for this condition?

Major depressive disorder is defined as an episode of major depression lasting at least two weeks with the following symptoms: (SIGECAPS) sleep changes, interest (lack thereof), guilt, energy (lack thereof), cognition/concentration (lack thereof), appetite (increased or decreased), psychomotor (agitation or retardation), and suicide (thoughts of or attempts). Major depressive disorder with psychotic features is defined by the above mentioned symptoms plus psychotic features of delusions or hallucinations, either auditory or visual. The first-line treatment for major depressive disorder with psychotic features is an antidepressant plus an antipsychotic medication. Studies show that sertraline, an antidepressant, in combination with olanzapine, an antipsychotic, works more effectively than just sertraline or just olanzapine alone.

Dopaminergic pathways

Mesolimbic Mesocortical Nigrostriatal Tuberoinfundibular

At sub-clinical doses (7.5mg) which antidepressant can have the paradoxical effect of sedation and appetite stimulant?

Mirtazapin (romeron increased release of serotonin and norepinephrine Unlike most TCAs, it shows weak or no activity as a blocker of sodium or calcium channels, or as an anticholinergic

MOCA

Montreal Cognitive Assessment >26=normal, add point for <12th grade education

Lead pipe rigidity

Motor dysfunction secondary to a lesion of the basal ganglia where during slow PROM, constant resistance is felt throughout; common symptom of Parkinson's disease

CIWA criteria

Nausea and vomiting (0-7) Tremor (0-7) Paroxysmal sweats (0-7) Anxiety (0-7) Agitation (0-7) Tactile disturbances (0-7) Auditory disturbances (0-7) Visual disturbances (0-7) Headaches, fullness in head (0-7) Orientation and clouding of sensorium (0-4)

Neuroleptic Malignant Syndrome (NMS)

Neuroleptic malignant syndrome: life-threatening condition characterized by muscle rigidity, autonomic instability, altered mental status, and hyperthermia that occurs 1-2 weeks after initiation or dose adjustment of a dopaminergic or antipsychotic drug.

When dopamine is deficient in the _________________________pathway, it can cause Parkinsonism with tremor, rigidity, akinesia or bradykinesia.

Nigrostriatal dopamine pathway

OCD vs OCPD

OCPD does not have recurrent obsessions/compulsions OCPD is ego syntonic extreme perfectionism

Wernicke-Korsakoff syndrome

Organic brain syndrome resulting from prolonged heavy alcohol use, involving confusion, unintelligible speech, and loss of motor coordination. It may be caused by a deficiency of thiamine, a vitamin metabolized poorly by heavy drinkers.

Describe panic disorder

Panic Disorder Recurrent panic attacks:PalpitationsShortness of breathChest painDizzinessDepersonalization/derealizationNumbness or tinglingFear of dying or losing control Worry about or maladaptive behavior related to panic attacks for at least 1 month Panic attacks do not only occur in response to particular stimulus Treat with SSRI or CBT

Which cluster A personality disorder is characterized by pervasive distrust and suspicion? commonly misinterpreting social cues- people talking about them or a coworker giving them a funny face.

Paranoid Personality Disorder

Serotonergic Agents that commonly cause Serotonin Syndrome.

SSRI Citalopram (Celexa) Escitalopram (Lexapro) Fluoxetine (Prozac) Paroxetine (Paxil, Pexeva) Sertraline (Zoloft) SNRI TCAs Dextromethorphan Meperidine

List the "schizo disorders" in order from the least to greatest in terms of impaired functioning and effect on daily life.

Schizoid < schizotypal < schizophrenic < schizoaffective

Which cluster A personality disorder is characterized by social withdrawal ?

Schizoid Personality Disorder

A 10-year-old female presents with inappropriate laughing and gross fasciculations of the tongue. History reveals a recent upper respiratory infection.

Sydenham chorea is a movement disorder characterized by chorea, emotional lability, and hypotonia. It is one of the major clinical manifestations of acute rheumatic fever. It is seen in about one-third of cases of ARF, most often in school-aged children. Neurologic symptoms include chorea (distal movements of the hands, rapid irregular jerking of the face and feet), muscle weakness (characterized by "milkmaid's grip") and loss of fine motor control. The movements are continuous while the patient is awake but improve with sleep. It has been described as "St. Vitus' dance." The symptoms typically improve gradually, with a mean duration of 12 to 15 weeks.

What are classic findings in pediatric abusive head trauma?

The classic constellation of abusive head trauma includes subdural hematoma, traumatic brain injury, and retinal hemorrhages. Retinal hemorrhages are present in up to 75% of cases and have a high specificity. Bucket handle fracture Posterior rib fractures Fractures of different ages Cutaneous: bruises, bites, burns Shaken baby syndrome: retinal hemorrhages

A 39-year-old male presents with dysuria, erectile dysfunction, heartburn, and nausea. He reports chronic shoulder, back, and knee pain for the past decade and has a history of transient lower extremity weakness. Various laboratory and imaging studies performed over the years have never displayed any abnormalities. Mental status examination reveals an anxious middle-aged male who expresses concern over the possibility of being seriously ill. The most likely diagnosis is A. Capgras syndrome B. functional neurological symptom disorder C. Münchausen syndrome D.somatic symptom disorder with predominant pain E. somatic symptom disorder

The correct answer is: E Somatic symptom disorder has replaced Briquet's syndrome, which is an eponym for somatization disorder The DSM-5 diagnostic criteria include: A. One or more somatic symptoms that are distressing or result in significant disruption in daily activity. B. Excessive thoughts, feelings, or behaviors related to the somatic symptoms or associated health concerns as manifested by at least one of the following: 1. disproportionate and persistent thoughts about the seriousness of one's symptoms; 2. persistently high level of anxiety about health or symptoms. 3. excessive time and energy devote to these symptoms or health concerns. C. Although any one somatic symptom may not be continuously present, the state of being symptomatic is persistent (typically more than 6 months). Answer A: Capgras syndrome is characterized by the delusional belief that a familiar person (family member, friend, close acquaintance) has been replaced by an imposter who appears identical. A variant of Capgras Syndrome is a Fregoli delusion in which it is believed that a persecutor can assume the physical appearance of a stranger. It is rare and can be seen in patients suffering from paranoid schizophrenia, dementia, epilepsy and brain injury. Answer B: Conversion disorder, now known as Functional Neurologic Symptom Disorder in DSM-5 to emphasize the importance of the neurological examination, also falls under the category of somatic symptom disorders and describes pseudo-neurological symptoms not better accounted for by known medical illness. Diagnosis requires the presence of voluntary motor or sensory impairment that suggest neurological illness that is not intentionally produced but cannot be explained by a medical condition, substance effect or cultural behavior or experience. There are associated psychological factors linked to the symptoms, which are often preceded by stress. Answer C: Münchausen syndrome describes a form of factitious disorder also referred to as hospital addiction because these patients are motivated by the secondary gain of assuming the sick role and so subject themselves to unnecessary tests and procedures in an attempt to acquire medical diagnoses. These patients intentionally feign symptoms in order to support a particular medical diagnosis. The question stem describes somatization disorder, a type of somatoform disorder in which patients present with multiple pain, gastrointestinal, sexual and neurological complaints which are not feigned yet are not supported by a known general medical condition. Answer D: While the patient presents with multiple somatic complaints, some of which are pain-related, the overall picture is more consistent with somatic symptom disorder in which chronic complaints from multiple physiologic systems are involved. Bottom Line: Briquet's syndrome is an eponym for somatization disorder, a somatoform disorder characterized by a history multiple pain, gastrointestinal, sexual and pseudo neurological symptoms beginning before the age of 30 that are not better described by a known medical condition and are not intentionally feigned. COMBANK Insight : In DSM-5, somatoform disorders are now referred to as somatic symptom and related disorders. The DSM-5 classification reduces the number of these disorders and subcategories to avoid problematic overlap. Diagnoses of somatization disorder, hypochondriasis, pain disorder, and undifferentiated somatoform disorder have been removed. Individuals previously diagnosed with somatization disorder will usually have symptoms that meet DSM-5 criteria for somatic symptom disorder, but only if they have the maladaptive thoughts, feelings, and behaviors that define the disorder, in addition to their somatic symptoms. Because the distinction between somatization disorder and undifferentiated somatoform disorder was arbitrary, they are merged in DSM-5 under somatic symptom disorder. Individuals previously diagnosed with hypochondriasis who have high health anxiety but no somatic symptoms would receive a DSM-5 diagnosis of illness anxiety disorder (unless their health anxiety was better explained by a primary anxiety disorder, such as generalized anxiety disorder). Some individuals with chronic pain would be appropriately diagnosed as having somatic symptom disorder, with predominant pain. For others psychological factors affecting other medical conditions or an adjustment disorder may be more appropriate.

A 32-year-old woman is brought to the ED by her husband because she "has been acting strangely." She has a past medical history of depression that is well controlled with fluoxetine. He also reports she has been suffering from a cold and took over-the-counter cough medication a few hours prior to the onset of her symptoms. She is agitated and confused. Her blood pressure is 160/80 mm Hg, pulse is 140 beats per minute, and temperature is 39°C. She is flushed and her pupils are dilated. She has clonus and hyperreflexia in her lower extremities, as well as a tremor in her hands. What is the most likely diagnosis?

The patient is suffering from serotonin syndrome likely due to an interaction between her antidepressant medication, fluoxetine and dextromethorphan, a cough suppressant found in many over-the-counter cough and cold medications. Fluoxetine is a selective serotonin reuptake inhibitor (SSRI). Serotonin syndrome results from excessive serotonin accumulation in the synaptic cleft and manifests as a triad of altered mental status, autonomic instability, and neuromuscular abnormality. Serotonin syndrome often occurs as the result of a drug-drug interaction between medications that increase the amount of serotonin in the synaptic cleft, however can occur following an overdose with an SSRI.

Borderline personality Disorder was named so because____________

Their personality borders on psychotic

Schizoaffective disorder Bipolar type

This subtype applies if a manic episode is part of the presentation. Major depressive episodes may also occur.

Schizoaffective disorder Depressive type

This subtype applies if only major depressive episodes are part of the presentation.

What is neurolepsis and what class of medications are highly associated with it?

This syndrome has 3 main features: ●Psychomotor slowing ●Emotional quieting ●Affective indifference

Anti-psychotic medications

Treat psychotic d/o including schizophrenia Atypical: Risperdal, Zyprexa, Seroquel, Geodon, Invega, Santris, Abilify Typical: Thorazine, Haldol

True or False: Conduct disorder and oppositional-defiant disorder may present similarly, but individuals with oppositional-defiant disorder do not engage in destruction of property such as fire setting or graffiti. Symptoms of oppositional-defiant disorder are also less severe than those of conduct disorder.

True

Your patient says I'm anxious_______________. List some DDX

What are your DDX? ●substance induced anxiety disorder ●hyperthyroidism ●panic disorder ●generalized anxiety disorder ●Specific phobia ●OCD

A 37-year-old male with torticollis and a dystonic facial expression in which the patient appears to have an exaggerated smile. He also complains of writer's cramp and a blepharospasm.

Wilson's disease is transmitted through an autosomal recessive gene and results in an error of copper metabolism, thus leading to copper deposition. The typical age of onset is usually childhood or adolescence, though it can be diagnosed as late as 50s. Neuropsychiatric symptoms include: dystonias, chorea, ataxias, wing-beating tremors, mood disorders, psychosis, intellectual decline. Keyser-Fleischer rings may be seen in the eyes due to copper depositon, and liver dysfunction can result. The treatment for this disorder is chelation therapy with D-penicillamine and dietary copper restriction.

Delirium tremens (DTs)

a disorder involving sudden and severe mental changes or seizures caused by abruptly stopping the use of alcohol

Delirium is characterized by:

acute onset, waxing and waning of consciousness, hallucinations or illusions (visual hallucinations are common) hyper or hypoactive states are possible the presentation isn't limited to motor symptoms. consider affect changes

Tricyclic Antidepressants (TCAs)

amitriptyline nortriptyline imipramine desipramine clomipramine doxepin amoxapine

pregnant psychotic lady > safest rx?

antipsycotic- ECT is refractory

Certainty of being liked

avoidant

High risk of withdrawing from alcohol..

chronic alcohol abuse - thiamine replacement before getting glucose to prevent wernicke Korsakoff syndrome. symptom triggered benzodiazepine administration CIWA protocal and vital signs every 4 hours.

Opiod overdose triad:

coma, respiratory depression, miosis (give narcan)

Choreoathetosis--acute intoxication- meth

combination of chorea and athetosis dance like writhing movements

Miosis

constricted pupils normal pupil size 2-4mm(bright light) 4mm-8mm (dark)

mydriasis

dilation of the pupil

Narcissistic Personality Disorder (NPD)

externalizes blames exaggerated self importance entitled failures blamed on others

Dependent Personality Disorder (Cluster C)

extreme dependency in close relationship, urgent seek for another when relationships end -codependency

Delusions of schizophrenia

fixed false beliefs (resistant to change, even in the face of overwhelming contradictory evidence) ●persecution or grandeur ●present in approximately 80% of pt with schizophrenia

tuberoinfundibular pathway

galactorrhea

Intermittent Explosive Disorder (IED)

intense outbursts that could be provoke by anything

atypical depression symptoms

leadened paralysis increase appetite

Schizophreniform disorder

meets all the criteria for schizophrenia are met, but the total duration of the disorder is less than six months.

Positive symptoms are theorized to be the result of over-activity of dopamine in the ___________________ pathway.

mesolimbic

If there is no longitudinal information available, can you diagnose a personality disorder?

no, Longitudinal Information is essential to diagnose personality disorders.

Clonus

rapidly alternating involuntary contraction and relaxation of a muscle in response to sudden stretch

hyperprolactinemia , galactorrhea nd gynecomastia

risperdol most likely to cause those symptoms

Delirium treatment algorithm

supportive treatments treat underlying cause

Hallucinations (Schizophrenia)

the perception of a sensory process in the absence of an external source. ●Auditory: MC 40-80% ●Visual: often unformed (glowing orbs/flashes of color) some pts with schizophrenia describe fully formed human figures, faces, or body parts. ●Somatic: pain, sexual intercourse, ●olfactory/gustatory:

Alcohol/Benzo Withdrawal

thiamine/glucose Ativan with symptom PRN CIWA

Extrapyramidal symptoms are most commonly caused by_____________antipsychotic drugs that antagonize dopamine D2 receptors.

typical, The most common typical antipsychotics associated with EPS are haloperidol and fluphenazine.

What is the triad of symptoms found in Serotonin Syndrome?

●Altered Mental Status ●Neuromuscular Abnormalities ●Autonomic Hyperactivity

What are the cluster B personality disorders?

●Antisocial ●Borderline ●Histrionic ●Narcissistic

Histrionic Personality Disorder (Cluster B)

●Attention-seeking ●inappropriately ●overly-dramatic storytelling ●superficial relationships ● often dressed hyper-sexually/ inappropriate

Symptom domains of Personality Disorders

●Cognitive-perceptual symptoms - eg, hallucinations and paranoid ideation ●Impulsive-behavioral dyscontrol - eg, self-injury, theft, interpersonal conflict ●Affective dysregulation - eg, depressed mood, mood lability, anxiety, anger

Tuberoinfundibular pathway: Hyperprolactinemia

Dopamine acts as a prolactin-inhibiting factor, D2 blockade increases prolactin levels by promoting its release in the pituitary gland.

A 23-year-old woman is brought to the emergency department by police after she bit an officer as he was trying to restrain her. They found her attempting to disrobe in the middle of a busy intersection. In the emergency department she is put in restraints after screaming and attempting to hit a nurse who was trying to get her vital signs. Her pulse is 125/min and systolic blood pressure is 160 mmHg. Diastolic pressure could not be obtained. Physical examination reveals vertical nystagmus is noted, but neither a full neurological exam nor a mental status exam is possible. Which of the following is the best treatment for this patient's condition? A. diazepam B. haloperidol C. metoprolol D.risperidone E. urine alkalinization

The correct answer is: A This patient is suffering from acute phencyclidine (PCP) intoxication, as evidenced by aggressive behavior, nystagmus (classically vertical), sympathetic activation, and an apparent need to undress. Phencyclidine is a dissociative anesthetic that is abused primarily for its hallucinatory effects. Rapid control of psychomotor agitation is the cornerstone of successful management. Physical restraints may be necessary initially and several staff members are often needed to control patients agitated from PCP. Chemical sedation should be administered as rapidly as possible. PCP is best treated with diazepam, or with lorazepam if pre-existing liver damage is a concern. Answers B & D: Risperidone or haloperidol should be considered if treatment with benzodiazepines fails. Some clinicians prefer to avoid butyrophenones because they are purported to lower the seizure threshold and impair heat dissipation in hyperthermic patients. However, there are no high quality studies in humans to support such claims. Answer C: Metoprolol does not play a role in the management of acute PCP intoxication. Answer E: In the past urine acidification was used to increase clearance but is now no longer used because of the risk of metabolic acidosis and renal damage. Bottom Line: PCP intoxication is characterized by aggressive behavior, sympathetic activation, vertical nystagmus, and often by a need to undress. Treatment is with diazepam. COMBANK Insight : Vertical nystagmus is a quick clue to PCP intoxication but horizontal and rotary nystagmus can also be seen.

A 26-year-old female patient is referred for treatment of a possible eating disorder. History reveals that she has been binge eating at least three times per week for the past five months. After these binges, she forcefully tries to vomit the food and subsequently "walks off the calories" with four mile walks. She reports that she enjoys food but does not want to become overweight. The patient is of normal weight for her height, reports no past psychiatric history, and her past medical history is significant only for childhood asthma. Physical examination reveals mildly eroded skin over the knuckles of her hands. A basic metabolic panel is obtained and reveals the following: Sodium: 137 mEq/L Potassium: 4.0 mEq/L Chloride: 100 mEq/L Bicarbonate: 23 mEq/L BUN: 9.0 mg/dL Creatinine: 0.9 mg/dL Glucose: 100 mg/dL Which of the following is the most appropriate management at this time? A. cognitive behavioral therapy B. family-based therapy C. fluoxetine D. inpatient psychiatric admission E. IV isotonic normal saline

The correct answer is: A This scenario describes a case of bulimia nervosa based on the criteria of binge eating episodes, purging behaviors, and normal weight. Bulimia nervosa is characterized by concurrent binge eating and inappropriate compensatory behaviors at least twice per week over a period of three months or more. Binge eating episodes involve the eating of a very large amount of food in a relatively short period of time, and these episodes are characterized by a loss of control, either because the overeating feels unpreventable or unstoppable. Patients engage in compensatory behaviors to neutralize the impact of binge eating on body weight. These compensatory behaviors include: self-induced vomiting; use of laxatives, enemas, diuretics, stimulants, diet pills; restrictive patterns of eating such as meal-skipping; strenuous exercise; or any combination of these behaviors. A core feature of bulimia nervosa is that one's shape or worth critically determines one's self-image or self-esteem. Patients with bulimia nervosa are usually of normal weight or overweight, but not underweight. If all the characteristics of bulimia nervosa are present and the patient is underweight, the diagnosis is anorexia nervosa, binge-eating/purging type. Bulimia nervosa includes two sub-types: purging type in which patients regularly use self-induced vomiting, laxatives, diuretics, or enemas to purge the food they binge on, or non-purging type in which patients may engage in compensatory behaviors of excessive eating or excessive physical activity or exercise but no purging. Classic physical examination findings in the purging sub-type of bulimia nervosa include eroded tooth enamel due to frequent vomiting and eroded skin over the knuckles of the fingers due to self-induced vomiting by putting fingers in the mouth. Psychotherapy is the treatment of choice for eating disorders, and pharmacologic treatments offer adjunctive use. Psychotherapies for eating disorders cover emotional states and conflicts, interpersonal relationships, history of trauma, excessive preoccupation with body/food/weight, and maladaptive coping skills that have resulted in abnormal eating behaviors. Cognitive-behavioral therapy (CBT) is the best researched, best established psychotherapy for bulimia nervosa and for binge eating disorder. CBT is based on the interplay of cognition, mood, physiological responses, and behaviors, and CBT strives to address and amend the distorted thinking and processing that has a negative impact on patients' mood, physiology, and behaviors. CBT is found to be more effective than medications for bulimia nervosa, but has been less promising for anorexia nervosa, possibly due to the problems of ego-syntonicity of symptoms and possible cognitive deficits secondary to being underweight in anorexia nervosa. Answer B: Family-based therapy involves parents being faced with the task of re-feeding their underweight child. Research demonstrates that this has the greatest impact for the treatment of anorexia nervosa in adolescents. Answer C: Adding medications to the psychotherapy treatment regimen may be useful in some cases. No particular medication has been shown to have superior efficacy, so it is recommended that a medication be selected based on history of the patient's symptoms and response and on the medication's side-effect profile. Fluoxetine is the best studied among pharmacologic treatments effective for bulimia nervosa and is the only one that is FDA-approved for its treatment. It has been found to be generally well-tolerated among patients and the recommended dosage is to first start with 20 mg every morning and then to gradually increase the dosage to 60 mg per day. Answer D: Eating disorders can often be managed well in an outpatient setting, but sometimes, there may be a need for inpatient admission and a safe, therapeutic environment. After adequate inpatient treatment, outpatient treatment helps for continuous follow-up. Patients with the purging sub-type of bulimia nervosa may be at risk for medical complications due to electrolyte disturbances. They may have hypokalemia from vomiting or laxative abuse, and may have hypochloremic alkalosis. Those patients who vomit repeatedly may place themselves at risk for esophageal tears and gastric tears. Indications for admission to an inpatient unit include the following: serious medical risks as evident by significant electrolyte disturbances, dehydration, continuous ipecac abuse, very low body weight (<75% of expected body weight), growth arrest, psychiatric risk of danger to self or others, psychotic symptoms, escalating or severe eating disorder symptoms such as excessive purging throughout the day, or persistence of symptoms despite outpatient treatment. In this case scenario, the patient's labs are within normal limits and there is no evidence of electrolyte abnormalities or dehydration currently, and she does not pose a risk of harm to self or others at this point. Answer E: Dehydration is implicated by a BUN to creatinine ratio of greater than 15 or 20 mg/dL. The goal of IV fluid therapy is to correct existing abnormalities in volume status and/or serum electrolytes. The composition of IV fluid that is to be given is very much dependent on the type of fluid that has been lost and any concurrent electrolyte imbalances. Most patients with dehydration are treated with isotonic or 1/2 isotonic saline but the choice of IV therapy can be influenced by concurrent abnormalities in serum sodium or serum potassium or the presence of metabolic acidosis. Hypotonic solutions should be used in hypernatremia and isotonic or hypertonic saline should be used in hyponatremia. Isotonic saline and/or blood should be used in patients with blood loss. Potassium or bicarbonate may be added in patients with hypokalemia or metabolic acidosis. In this case scenario, the patient does not appear to have dehydration due to normal BUN to creatinine ratio, and she does not have any electrolyte abnormalities. She does not require IV fluid therapy at this time but she needs to be educated on healthy eating habits and may benefit from consultation with a dietitian who specializes in eating disorders. Bottom Line: CBT has been found to be the most effective psychotherapy for patients with bulimia nervosa, and has been found to be more efficacious than pharmacologic treatment.

A 24-year-old male presents to the office with the complaint of nightmares, flashbacks, and irritability for 1 week. History reveals that three weeks ago, he was involved in a motor vehicle accident where he witnessed the death of his mother and brother. He cannot remember details of the accident, and feels guilty that he survived when his family members died. The patient describes feeling numb and detached from his current surroundings and feels like he has lost contact with reality and daily obligations such as going back to work, and states he now has a fear of motor vehicles and refuses to leave his neighborhood. Physical examination reveals a well-developed male who appears anxious and fatigued. The most likely diagnosis is A. acute anxiety disorder B. acute stress disorder C. adjustment disorder D.post-traumatic disorder E. schizophreniform disorder

The correct answer is: B Acute stress disorder (ASD) involves exposure to a traumatic event and a subsequent response of intense fear, helplessness, or horror. The traumatic event involves a threat of death to self or others, or a threat to the physical integrity of self or others. Symptoms occur within 4 weeks of the traumatic event and persist for at least 2 days and up to less than 4 weeks. Either while experiencing or after experiencing the traumatic event, the individual has three or more of the following dissociative symptoms: a sense of numbing or detachment, a reduction in awareness of surroundings, derealization, depersonalization, dissociative amnesia. The traumatic event is re-experienced in the form of dreams or images, marked anxiety or hypervigilance exists, and marked avoidance of stimuli occurs. The impairments cause significant distress in social, occupational, or other important areas of functioning. In this case, the patient was exposed to a traumatic event that threatened the life or physical integrity of self or others, and the case describes a duration of symptoms that is less than one month. The primary difference between ASD and posttraumatic stress disorder (PTSD) is the duration of the symptoms and the former's emphasis on dissociative reactions to the trauma. ASD refers to symptoms manifested during the period from 2 days to 4 weeks post-trauma, whereas PTSD can only be diagnosed from 4 weeks. Answer A: Acute anxiety disorder is characterized by worry and fear about something that might happen in the future. To rule out anxiety disorders secondary to general medical or substance abuse conditions, a detailed history and review of symptoms is needed. With the patient, review the use of caffeine-containing beverages (coffees, teas, energy drinks, sodas), over-the-counter medications (aspirin with caffeine, sympathomimetics), herbal remedies, or the use of street drugs. Answer C: Adjustment disorder occurs with the development of emotional or behavioral symptoms in response to an identifiable stressor(s) occurring within 3 months of the onset of the stressor(s). These symptoms or behaviors are clinically significant as evidenced by either of the following: (1) marked distress that is in excess of what would be expected from exposure to the stressor; (2) significant impairment in social or occupational (academic) functioning. Once the stressor has terminated, the symptoms do not persist for more than an additional six months. Answer D: DSM-5 criteria for PTSD differ significantly from the DSM-IV criteria. The stressor criterion A is more explicit with regard to events that qualify as "traumatic" experiences. Also, DSM-IV Criterion A2 (subjective reaction) has been eliminated. Whereas there were three major symptom clusters in DSM-IV - re-experiencing, avoidance/ numbing, and arousal - there are now four symptom clusters in DSM-5, because the avoidance / numbing cluster is divided into two distinct clusters; avoidance and persistent negative alterations in cognitions and mood. This latter category, which retains most of the DSM-IV numbing symptoms, also includes new or re-conceptualized symptoms, such as persistent negative emotional states. The final cluster - alterations in arousal and reactivity - retains most of the DSM-IV arousal symptoms. It also includes irritable behavior or angry outbursts and reckless or self-destructive behavior. PTSD is now developmentally sensitive in that diagnostic thresholds have been lowered for children and adolescents. Furthermore, separate criteria have been added for children age 6 or younger. Answer E: Schizophreniform disorder is characterized by at least two or more of the following Criterion A symptoms, each present for a significant portion of time, continuously for less than six months: (1) delusions; (2) hallucinations; (3) disorganized speech; (4) grossly disorganized or catatonic behaviors; (5) negative symptoms. Delusions, disorganized speech, hallucinations, and agitation are considered to be positive symptoms, associated with dopamine receptors. Negative symptoms of schizophrenia are associated with muscarinic receptors and include characteristics such as flattened affect, social withdrawal, anhedonia (lack of interest), apathy, and poverty of thought. Only one of the above five criterion symptoms is required if delusions are bizarre or hallucinations consist of a voice keeping up a running commentary on the individual's behavior or thoughts, or two or more voices conversing with each other. These symptoms are distressing and affect daily functioning. Schizophreniform disorder symptoms occur for more than one month but last less than six months, whereas, brief psychotic disorder symptoms last for one month or less. On the other hand, schizophrenia is diagnosed when the symptoms occur for more than six months. Two-thirds of individuals with schizophreniform disorder progress to being diagnosed with schizophrenia. Bottom Line: Acute stress disorder is characterized by an intense fear and horror within 4 weeks of a traumatic event. Symptoms last from a minimum of 2 days to a maximum of 4 weeks. COMBANK Insight : The duration of disease presentation is very important for diagnosing psychiatric disorders. Sometimes it is the only duration of symptoms that leads you to the correct answer.

True or False: All antipsychotics reduce dopaminergic neurotransmission.

True. Schizophrenia is theorized to occur due to hyperactivity of dopamine so the medications used to treat them all reduce dopaminergic transmission

Atypical anti-psychotics that must be taken with food __________ and ____________.

Lurasidone and

Zyprexa (olanzapine)

atypical antipsychotic MOA:

What are the 4 mature defense mechanisms?

●Sublimation ●Altruism ●Suppression ●Humor (SASH)

Invega (paliperidone)

Atypical Antipsychotic It can treat schizophrenia and schizoaffective disorder. Invega sustenna (beginning) IM Invega trinza (maintenance) MOA: Paliperidone is the primary active metabolite of risperidone. The mechanism of action is unknown but it is likely to act via a similar pathway to risperidone.

DSM-5 includes 10 personality disorders grouped into three clusters based upon descriptive similarities.

Cluster A characteristics - Individuals may appear odd and eccentric •Paranoid •Schizoid •Schizotypal Cluster B characteristics - Individuals often appear dramatic, emotional, or erratic •Antisocial •Borderline •Histrionic •Narcissistic Cluster C characteristics - Individuals often appear anxious or fearful •Avoidant •Dependent •Obsessive-compulsive

_________________________ inhibits prolactin secretion into circulation.

Dopamine

A 52-year-old male presents to the emergency department by ambulance with visual hallucinations. History reveals that he was discovered in an alley and reported to be seeing "red zebras." He has a history of multiple emergency department visits due to alcohol intoxication within the past year and is presumed to be homeless. Vital signs are within normal limits. On exam he is alert, awake, and oriented to person, place and time. He reports his last drink was 12 hours ago. The most likely diagnosis is A. alcoholic hallucinosis B. alcoholic withdrawal C. delirium tremens D.LSD intoxication E. opioid withdrawal

The correct answer is: A Acute alcoholic hallucinosis occurs in an alcohol-dependent individual acutely after the cessation of drinking, and occurs during alcohol withdrawal. No delirium, tremors, or autonomic hyperactivity develops. Alcoholic hallucinosis occurs within 12 to 48 hours after cessation of chronic alcohol abuse. Symptoms include tactile, visual, and/or auditory hallucinations. Symptoms can become chronic. However, vital signs and mental orientation remain normal. The treatment is supportive therapy, which can include IV fluids and electrolyte replacement. Answer B: Minor (early) alcoholic withdrawal presents with an onset of 8-9 hours after the last drink with tremulousness, flushed face, mild anxiety, headache, GI upset, insomnia, hallucinations (in 25% of individuals), diaphoresis, grand mal seizures (rum fits), and mild disorientation. Answer C: Delirium tremens (DTs) describes late, acute withdrawal symptoms with an onset of 48-96 hours after the last drink and cessation from heavy drinking. DTs are otherwise known as alcohol withdrawal delirium. Symptoms of DTs include the following: tremors, an increase in psychomotor activity, vivid hallucinations, illusions, delusions, profound disorientation and confusion, increased autonomic activity (tachycardia, hyperhidrosis, fever, dilated pupils). Patients may also experience formication (tactile hallucinations that feel like insects crawling all over the skin). In this case scenario, the patient last drank about 12 hours ago and is still likely intoxicated. DTs describe a more acute withdrawal that occurs within 48-96 hours after the last drink, and in DTs, patients often present with abnormal vital signs of fever and tachycardia. The clinical presentation is highly suggestive for acute alcohol withdrawal delirium. Delirium tremens typically presents within 48 to 96 hours after cessation of alcohol consumption. Symptoms include tachycardia, psychomotor agitation, combativeness, hypertension, and fever. Left untreated, it can be fatal. Treatment is with intravenous benzodiazepines to control psychomotor agitation as well as supportive measures. Answer D: LSD intoxication also presents with hallucinations, a distorted sense of time, euphoria, and a sense of well-being; a perception of being a passive observer of events or being outside one's body, feelings of expansiveness, and enhanced mystical or spiritual experiences. Patients may also experience synesthesia (a blending of the senses), where users report "hearing" colors or "seeing" sounds. Other effects include fear, panic reactions, dysphoria, frightening imagery, and an overwhelming sense of dread. Frank psychosis may occur and occasionally persists for days. Answer E: Opioid withdrawal presents with tachycardia, hypertension, piloerection, nausea, vomiting. Hallucinations are not associated with opioid withdrawal. Bottom Line: Alcoholic hallucinosis occurs within 12 to 48 hours after cessation of chronic alcohol abuse. Symptoms include tactile, visual, and/or auditory hallucinations, however, vital signs and mental orientation remain normal. The treatment is supportive therapy, which can include IV fluids and electrolyte replacement.

A 32-year-old female has a long-standing history of anorexia nervosa and is very worried about the risk of her children developing the condition. As her provider, you inform the patient that A. a combination of genetic, biological, psychological, family, environmental, and social factors contribute to developing an eating disorder B. as long as the mother is not currently affected, the children are not at increased risk C. psychological factors will be the defining determinant of whether the children develop anorexia nervosa or not D.relationships with other children at school and peer pressure define the main components that will lead to the development of anorexia nervosa E. the children have a two-fold increased risk of developing anorexia nervosa if at least one parent has anorexia nervosa based on genetic factors

The correct answer is: A Anorexia nervosa is characterized as an eating disorder in which patients refuse to maintain a minimally normal weight, have intense fear of gaining weight, and have a distorted self-image of their body and weight. A combination of genetic, biological, psychological, family, environmental, and social factors contribute to developing anorexia nervosa. No one factor is the sole defining factor in the development of anorexia nervosa. Research has been undertaken to understand the biological basis of the disorder. Neurochemical studies have revealed that diminished norepinephrine turnover and activity are suggested by reduced MHPG (3-methoxy-4-hydroxyphenylglycol) levels in the urine and the CSF of some patients with anorexia nervosa. An increase in MHPG levels is associated with a decrease in depression. Endogenous opioids may contribute to the denial of hunger in patients with anorexia nervosa. Starvation results in many biochemical changes including: hypercortisolemia, amenorrhea. CT scan images of patients with anorexia nervosa have revealed enlarged sulci and ventricles in anorectic patients during starvation. Some theorists have proposed a hypothalamic-pituitary axis (neuroendocrine) dysfunction. Some studies have demonstrated dysfunctions in the three neurotransmitters of serotonin, dopamine, and norepinephrine that are implicated in regulating eating behavior in the paraventricular nucleus of the hypothalamus. Other humoral factors that may be involved include: CRF (corticotropin-releasing factor), neuropeptide Y, gonadotropin-releasing hormone, and thyroid-stimulating hormone. Social factors also play a crucial role in the development of anorexia nervosa. Patients with anorexia nervosa find much support for their practices in society's and media's overemphasis on thinness and exercise. Some patients with anorexia nervosa have close, but troubled relationships with their parents. Children and adolescents with binging and purging behaviors often come from dysfunctional families with high levels of chaos, hostility, and isolation with low levels of empathic and nurturing upbringings. Sometimes, adolescents with severe eating disorders may tend to draw attention away from very strained parental marital relationships or from very dysfunctional family homes. These family and environmental factors contribute to the development of eating disorders as these individuals may be helpless, emotionally deprived, psychologically disturbed, and their abnormal eating behaviors give them some form of control over their actions. Vocational and non-vocational interests interact with other factors to increase the risk of developing anorexia nervosa such as: young girls engaging in strict ballet classes, boys engaging in wrestling schools, young girls engaging in gymnastics, etc. Sexual orientation and sexual attractiveness also play a role in the development of eating disorders in some individuals. There have been some hypotheses surrounding psychological factors that may contribute to the development of anorexia nervosa. Anorexia nervosa has been found to be much more prevalent amongst females as compared to males. Onset is usually found to be in adolescence where there is a demand for adolescents to behave more independently and increase their social and sexual functioning. One such hypothesis denotes that underlying psychological disturbances in young women with the disorder have conflicts surrounding the transition from girlhood to womanhood. Other psychological states of mind that have been implicated in the development of the disorder include helplessness and difficulty establishing autonomy. There are no specific genetic factors that have been found to contribute to the development of the disorder. Some studies point towards higher concordance rates in monozygotic twins than in dizygotic twins. Other evidence has demonstrated that sisters of patients with anorexia nervosa are more likely to develop anorexia nervosa, but this may occur as a result of social influences more than genetic influences. Answer B: This is simply not true. One should not tell the mother to be unconcerned about the development of anorexia nervosa in her children when she is obviously concerned, but rather, one should reassure her and inform her that there are various factors that may contribute to the development of this disorder. Answer C: Psychological factors contribute to the development of eating disorders, amongst other factors discussed above, in most individuals with these disorders, but they are not the defining determinant of whether the children will develop anorexia nervosa or not. Answer D: Peer pressure and peer relationships may certainly contribute to the development of eating disorder in some individuals but they are usually not the main components in the development of the disorder in most individuals. Answer E: Current research has not found enough information to say how much of a genetic role there is in the development of anorexia nervosa in families. Bottom Line: A combination of various, multiple factors contribute to the development of anorexia nervos

A 27-year-old male on the inpatient psychiatric unit has a six minute seizure episode witnessed by the nursing staff. Upon arrival to the patient's room he appears to be in a post-ictal state and is minimally responsive. Upon initiating a physical examination the patient begins to have another tonic-clonic seizure. Question 1 of 2 in this set The most appropriate medication at this time is A. levetiracetam B. lorazepam C. phenobarbital D.phenytoin E. valproic acid

The correct answer is: B The above-mentioned patient is suffering from status epilepticus. Status epilepticus refers to an epileptic seizure that lasts more than 30 minutes or a constant or near-constant state of having seizures. Initial therapy for status epilepticus should include a benzodiazepine medication such as lorazepam, which are usually the treatment of choice when quick routes are needed for acute treatment. This is usually given in the injectable IV form. Alternative medications would include diazepam or midazolam. Answers A & E: Valproic acid and levetiracetam are used to treat seizure disorders, but they are not used in the acute management of status epilepticus. Answer C: Phenobarbital and phenytoin are also used to treat status epilepticus, but they are second-line agents and should not be used until lorazepam or diazepam has been tried first. Answer D: Phenobarbital and phenytoin are also used to treat status epilepticus, but they are second-line agents and should not be used until lorazepam has been tried first. Bottom Line: Lorazepam is the first medication that should be used to treat status epilepticus. Alternatives are diazepam and midazolam. COMBANK Insight : A patient who does not recover from the post-ictal state before seizing again is in status epilepticus and needs to be treated accordingly. When a hospitalized patient seizes, a fingerstick glucose should always be obtained as hypoglycemia is a common cause of seizures in hospital patients. Glucose and thiamine should be given if the patient is hypoglycemic. Initial clinical assessment of a patient in status epilepticus is as follows: neurological assessment, general evaluation with attention paid to respiratory and circulatory status, frequent vital signs, placement of two IV catheters, and cardiac monitoring with pulse oximetry. Patients should be given oxygen as needed. Electrolytes, glucose, toxicology, CBC, LFTs, Ca, Mg, ABG, and seizure medication therapeutic levels should be checked. As noted in the previous question, lorazepam is the first-line medication that should be used for patients in status epilepticus. Alternatives for initial therapy are other benzodiazepines such as diazepam. If a patient does not respond to lorazepam, the next medication that should be given is IV phenytoin. Phenytoin stabilizes neuronal membranes and decreases seizure activity by increasing efflux or decreasing influx of sodium ions across cell membranes in the motor cortex during generation of nerve impulses. It also prolongs effective refractory period and suppresses ventricular pacemaker automaticity as well as shortens the action potential in the heart. This patient does not yet have refractory status epilepticus, but the primary drugs used if refractory are phenobarbital, pentobarbital, midazolam, and propofol. There is no consensus about which should be used first.

A 16-year-old girl with a history of depression is brought to the emergency department after being found in her room lying next to an empty bottle of pills. She has no significant medical history aside from depression, for which she takes bupropion. She lives with her mother, who suffers from migraines and takes amitriptyline, and her father, who has hypertension and chronic lower back pain following a motor vehicle accident, for which he takes extended-release verapamil and oxycodone. On physical examination, she is somnolent and oriented to person and place but not time. Her heart rate is 40/min and regular, and her blood pressure is 67/45 mm Hg. Pupils are 4mm and equally reactive. Naloxone is administered but produces no improvement in her mental status or hemodynamic parameters. A finger stick reveals a glucose of 102 mg/dL. Which of the following is the most likely explanation for this patient's current condition? A. bupropion overdose B. calcium channel blocker overdose C.ethanol intoxication D. hypothermia E. narcotic overdose F. tricyclic antidepressant overdose

The correct answer is: B The striking features of this patient's presentation are profound hypotension and bradycardia with relatively preserved mentation. This clinical presentation is consistent with calcium channel blocker overdose, which should be suspected given her history of depression and access to verapamil. Calcium channel blockers (CCBs) produce four cardiovascular effects: (1) peripheral vasodilation (2) negative chronotropy (decreased heart rate) (3) negative inotropy (4) delayed cardiac conduction The signs and symptoms of CCB toxicity logically follow from these cardiovascular effects, resulting in profound hypotension, bradycardia and various degrees of atrioventricular block. Despite profound hypotension, mental status is often times preserved in these patients. The ingestion of extended-release formulations will result in the delayed onset of these findings and is often complicated by sudden cardiac collapse, shock, and death. The treatment for significant calcium channel blocker ingestion is supportive, as there is no antidote. Mortality is high. Answer A: Bupropion overdose presents with hypertension, tachycardia, and seizure, not hypotension and bradycardia. Answer C: Ethanol is commonly co-ingested with other drugs; however, it alone would not explain this patient's behaviors and exam findings. Answer D: Hypothermia may present with bradycardia, hypotension, and altered mental status; however, it would be unlikely that this patient had as significant exposure from the history provided. Answer E: Narcotic overdose is unlikely in this patient. In addition to altered mental status and bradycardia, miosis is typically noted. The patient's condition would be expected to improve with the administration of naloxone. Answer F: Signs and symptoms of mild to moderate tricyclic antidepressant (TCA) intoxication include confusion, agitation, and anticholinergic symptoms. Severe TCA toxicity presents with lethargy or coma, seizure, autonomic instability and characteristic EKG findings (widened QRS complex and terminal R wave in lead aVR). Bottom Line: Calcium channel blocker overdose presents with profound bradycardia, hypotension, and preserved or mildly depressed mentation. COMBANK Insight : Some items may contain more than just the standard 5 (A-E) answer choices. Do not be surprised if you encounter this on exam day.

What makes the schizotypal personality disorder different from the other Cluster A personality disorders?

Unlike the other cluster A disorders, patients with schizotypal personality disorder experience perceptual and cognitive dysfunction. Patients with schizotypal personality disorder often demonstrate odd beliefs, like magical thinking and eccentric clothing preferences.

A 53-year-old-woman is hospitalized after a suicide attempt using her psychiatric medication. She has a history of multiple previous attempts, the first when she was 16 years old, and several violent outbursts towards others. She has had multiple partners in the past several years, and her suicide attempt was due to her recent breakup with "the love of her life." She feels that her life is too unstable, and that "everyone always abandons" her. Since being in the hospital, she has fired several nurses due to mistrust of their intentions towards her. She has identified the physician on the general medical floor as the only trustworthy person on her medical team and asked him to arrange for her to be discharged home instead of to inpatient psychiatric treatment. What is the most effective treatment for her psychiatric disorder? A. cognitive behavioral therapy alone B. dialectical behavioral therapy with adjuvant medication C.interpersonal therapy alone D. selective serotonin reuptake inhibitors with adjuvant antipsychotics E. tricyclic antidepressants and psychodynamic therapy

The correct answer is: B This patient's presentation is most consistent with borderline personality disorder. Borderline personality disorder (BPD) is characterized by affective lability, fear of abandonment, suicidal or self-destructive gestures, unstable, intense impersonal relationships, transient paranoid ideation, and splitting. This patient's history of long-term psychiatric issues, multiple suicide attempts, numerous intense relationships, paranoia, and feelings of abandonment all make borderline personality disorder likely. Moreover, this patient is exhibiting "splitting," a trait often seen people with BPD, in which they either love or hate people they come in contact with. Though treatment is difficult, dialectical behavior therapy, or DBT and medication management has been shown to be helpful. DBT consists of both individual and group therapy. Patients learn how to identify, accept, and tolerate their emotions (known as "mindfulness") and are provided with strategies for dealing with distress in ways that minimize their impulsivity. It has been shown to decrease self-injury behaviors and decrease hospitalizations. Benzodiazepines, selective serotonin reuptake inhibitors (SSRIs), and antipsychotics can be used as well. Answer A: Cognitive behavioral therapy, or CBT, has been shown to be helpful in borderline personality disorder, but most patients also require medication to achieve the best outcome. In CBT, the therapist develops a tight alliance with the patient and works to re-frame one's feelings and behaviors to better interact with the world. DBT expands on CBT through addition of mindfulness. Answer C: Interpersonal therapy alone is not a treatment for borderline personality disorder. This is a short term-therapy that has been used for treatment of depression. Patients usually pick one or two topics to address throughout the therapy sessions. Answer D: SSRIs with adjuvant antipsychotics would be the correct treatment for refractory depression or MDD with psychotic features. These medications can also be used in treatment of borderline disorder but should be combined with therapy. Psychotherapy, especially DBT, is integral to the treatment of borderline personality disorder. Answer E: Psychodynamic therapy has been shown to be effective in borderline personality disorder but tricyclic antidepressants are a particularly poor choice for a patient who has attempted suicide. Bottom Line: Borderline Personality Disorder is characterized by affective lability, fear of abandonment, suicidal or self-destructive gestures, and splitting. It is best treated with a combination of medication and dialectical behavioral therapy. COMBANK Insight : If a question shows a patient exhibiting splitting, the diagnosis is most likely borderline personality disorder.

You are working with a quality assessment agency and notice that some of the nursing home residents with dementia have abrasions on their wrists and ankles, bruising on the posterior thighs, and dirty clothes. The most appropriate course of action is to A. admit the affected residents to the hospital B. contact the affected residents' family members C. report to the Long Term Care Ombudsman Program D. speak to a nursing home representative E.transfer residents to another the facility

The correct answer is: C Elders in institutional settings in all states and territories are protected by state Long Term Care Ombudsman Programs (LTCOP) that receive complaints and advocate on behalf of long-term care residents. The state LTCOP may have authority to investigate and intervene as needed, or it may refer the case to another agency such as Adult Protective Services (APS). Answer A: Unless the resident is in acute medical need and unable to consent for treatment, consent must be obtained for permission for care. If the resident is not in immediate danger, referral to the Long Term Care Ombudsman Program is an appropriate starting point with continued monitoring of as risk residents Answer B: With exception for emergency situations, consent must be obtained prior to contacting of family members in regard to medical issues. Answer D: Communicating concerns to the appropriate representative may help to facilitate change in the institution, but reporting to the appropriate outside agency allows for third-party monitoring and follow-up. Answer E: Removal from facility is appropriate for acute medical need, otherwise reporting process should be started through appropriate reporting agency with follow-up and monitoring. Bottom Line: The Long Term Care Ombudsman Program (LTCOP) receives complaints and advocates on behalf of long-term care residents and community dwelling elders are protected in all states by state adult protective services agencies (APS). These agencies receive and investigate allegations or complaints, and provide social, legal, medical interventions to help the victims. APS and LTCOP agencies are designed to protect the elderly from mistreatment by non strangers, such as family members, non-family caretakers, or employees of the facility where elderly victims reside.

A 36-year-old male presents on magistrate warrant filed by his uncle due to him being off medications for eight months now. He hears incomprehensible voices and answers questions inappropriately. He appears to respond to internal stimuli as he talks to himself. He exhibits reduced affect and looks around the room as if he suspects that something is there. His uncle reports that he has not been eating nor bathing for the past week, and that he has engaged in bizarre behaviors such as wearing his shirt inside out and forgetting to wear his pants. When offered water, he pours the water on himself. His past medical history is significant only for asthma, and family history is positive for unknown psychiatric condition in an aunt. The most likely diagnosis is A. schizoaffective disorder B. schizoid personality disorder C. schizophrenia D.schizophreniform disorder E. schizotypal personality disorder

The correct answer is: C This patient most likely has Schizophrenia. The (abridged) DSM-5 diagnostic criteria include the following: A. Either delusions, hallucinations, disorganized speech, grossly disorganized or catatonic behavior, or negative symptoms. B. Significant functional or interpersonal disturbance. C. Persistent for at least 6 months with at least 1 month of Criteria A symptoms. D. Schizoaffective disorder and depressive or bipolar disorders with psychotic features have been ruled out. Schizophrenia usually presents at a younger age in males (ages 15-24) compared to females (ages 25-34). Females also have a better prognosis with regards to treatment compared to males. Poor prognosis in schizophrenia is associated with the following features: early age of onset, negative symptoms, poor pre-morbid functioning, family history of schizophrenia, and disorganized features. Answer A: It could be schizoaffective disorder, but there is no mention of features of a mood disorder for this patient. Schizoaffective disorder has been reconceptualized in DSM-5 as a longitudinal instead of a cross-sectional diagnosis - more comparable to schizophrenia, bipolar disorder, and major depressive disorder, which are bridged by this condition - and requires that a major mood episode be present for a majority of the total disorders duration after Criterion A has been met. Answer B: Those with schizoid personality disorder have a pervasive pattern of detachment from social relationships and a restricted range of expression of emotions in interpersonal settings with four additional features (e.g., chooses solidarity, little sexual interests, few activities, lacks close friends). Answer D: Schizophreniform disorder fulfills the same above-mentioned Criterion A symptoms of schizophrenia except for the duration of time criteria. Schizophreniform disorder symptoms occur for more than one month but last less than six months, whereas, brief psychotic disorder symptoms last for one month or less. On the other hand, schizophrenia is diagnosed when the symptoms occur for more than six months. Two-thirds of individuals with schizophreniform disorder progress to being diagnosed with schizophrenia. Answer E: Those with schizotypal personality disorder have a pervasive pattern of social and interpersonal deficits marked by acute discomfort with, and reduced capacity for, close relationships as well as by cognitive or perceptual distortions and eccentricities of behavior, beginning by early adulthood and present in a variety of contexts, as indicated by five or more additional criteria (e.g., odd beliefs/thinking, ideas of reference, unusual perceptual experiences, paranoid ideations). Bottom Line: Individuals with disorganized type schizophrenia present with disorganized speech, disorganized behavior, flat or inappropriate affect, and along with some Criterion A symptoms. COMBANK Insight : Two changes were made to the DSM-5 Criterion A for schizophrenia: 1) the elimination of the specific attribution of bizarre delusions and Schneiderian first-rank auditory hallucinations (e.g., two or more voices conversing), leading to the requirement of at least two Criterion A symptoms for any diagnosis of schizophrenia, and 2) the addition of the requirement that at least one of the Criterion A symptoms must be delusions, hallucinations, or disorganized speech. The DSM-IV subtypes of schizophrenia were eliminated due to their limited diagnostic stability, low reliability, and poor validity. Instead, a dimensional approach to rating severity for the core symptoms of schizophrenia is included.

An 11-year-old male presents with his mother to the clinic. She reports her son has been losing his temper at home, disobeying rules at home, and showing resentment towards her when she scolds him. His teacher has complained that he is not following class instructions and often talks back to her in a disrespectful manner. He recently received a two-day suspension notice from the school for his conduct. These behaviors are creating an uncomfortable environment at home for the family. When asked in the interview as to why he is engaging in such behaviors, the patient says, "It is all their fault. They are the ones who do annoying things that make me angry." The most likely diagnosis is A. antisocial personality disorder B. avoidant personality disorder C.conduct disorder D. oppositional defiant disorder E. trichotillomania

The correct answer is: D Oppositional defiant disorder (ODD) comprises of a pattern of negativistic, hostile, and defiant behavior lasting at least 6 months, during which at least 4 or more of the following criteria are present where the child often: Loses temper Argues with adults Actively defies to comply with adults' requests or rules Deliberately annoys people Blames others for his or her mistakes or misbehavior Becomes touchy or easily annoyed by others Angry and resentful Spiteful or vindictive The behaviors are usually directed an authority figure. The behavioral disturbances cause clinically significant impairments in social, occupational,or academic functioning. ODD cannot be diagnosed if conduct disorder is present. There is a gradual onset of symptoms, usually before 8 years of age. The patient in this case scenario demonstrates at least six criteria from the above criterion listed for diagnosis. Answer A: Antisocial personality disorder is diagnosed at or above 18 years of age. Individuals with anti-social personality disorder have long criminal records of repetitive unlawful acts and socially irresponsible behaviors that began prior to the age of 15 years (otherwise known as conduct disorder in adolescents). These individuals lack remorse for the harm they cause and are generally unconcerned for the feelings and rights of others. Most of these individuals are male. Frequent co-morbid disorders include ADHD, somatization disorder, and substance abuse disorders. Answer B: Individuals with avoidant personality disorder desire to have friends but they avoid making friends or avoid going to social gatherings because of an inferiority complex and excessive discomfort or fear of rejection. They often experience extreme shyness and have low self-esteem. Answer C: Conduct disorder presents in children or adolescents with a repetitive and persistent pattern of behavior in which the basic rights of others or major age-appropriate societal norms or rules are violated. Behaviors in conduct disorder include: aggression to animals and people, destruction of property, deceitfulness, theft, and serious violation of rules. Criteria for conduct disorder are met when at least three or more of the following are met: (1) often bullies or threatens others; (2) often initiates fights; (3) has used a weapon or object that can cause serious harm to others; (4) has been physically cruel to others; (5) has been physically cruel to animals; (6) has stolen while confronting a victim (example: mugging); (7) has forced someone into sexual activity; (8) has deliberately engaged in fire setting with the intention of causing serious harm; (9) has deliberately destroyed others' property; (10) has broken into someone else's house or car; (11) often lies to avoid obligations or to obtain favors; (12) has stolen items of non-trivial value without confronting a victim; (13) often stays out at night despite parental prohibitions, beginning before age 13 years; (14) has run away from home at least twice; (15) is often truant from school, beginning before age 13 years. The behavioral disturbances cause clinically significant impairments in social, occupational, or academic functioning. There is an earlier average of onset for boys (age 10-12 years) compared to girls (age 16 years). Prevalence is higher amongst boys compared to girls. The patient in this scenario demonstrates oppositional and defiant behaviors, but has not engaged in physically aggressive or physically threatening behaviors towards other people or animals, thus ruling out conduct disorder. Answer E: Trichotillomania comprises of recurrent pulling of one's hair resulting in noticeable hair loss. An increasing amount of tension is usually experienced immediately before pulling out the hair, and there is immense relief or pleasure after the act. The disturbance causes clinically significant distress in social, occupational, or academic functioning. Medical conditions such as dermatological conditions have to be ruled out along with alopecia (hair loss) due to medication side effects or organic causes. Bottom Line: ODD is diagnosed in children or adolescents with defiant and negativistic behaviors and attitudes towards authority figures.

A 7-year-old second grader presents with his father for recurrent stomach aches, which have caused him to miss school most of the days in the past month. He cries relentlessly in the mornings, saying that he does not want to go to school, and about thirty minutes afterwards, he is found to be happy at home either playing or watching television. He willingly goes shopping with his parents. He denies feeling depressed, denies any history of abuse, and said he does not like to be alone without his parents because he is afraid something bad will happen to them. Physical examination is within normal limits. What is the most likely diagnosis? A. agoraphobia B. disruptive mood dysregulation disorder C. major depressive disorder D. separation anxiety disorder E. social anxiety disorder

The correct answer is: D Separation anxiety disorder involves excessive anxiety concerning separation from the home or from those to whom the individual is attached. Three or more of the following criterion have to be present for diagnosis: recurrent excessive distress when separation from home or major attachment figures occurs or is anticipated, excessive worry about losing or harm befalling major attachment figures, excessive worry that an untoward event (such as kidnapping, getting lost) will lead to separation from the major attachment figure, persistent refusal to go to school or elsewhere because of fear of separation, persistent reluctance to be alone at home or outside without the major attachment figures, persistent refusal to sleep away from home or away from the major attachment figure, recurrent nightmares about separation, or repeated complaints about physical symptoms (such as vomiting, stomach aches, headaches, nausea) when separation from major attachment figures occurs or is anticipated. For diagnosis, the disturbance has to last at least four weeks, and the disturbance causes clinically significant distress or impairment in social, academic, or other important areas of functioning. In this case scenario, the patient has had symptoms (recurrent distress when separation from his parents is anticipated, excessive worry about harm befalling his parents, persistent refusal to go to school, persistent reluctance to be alone without his parents, recurrent nightmares and stomach aches) for more than four weeks, thus pointing towards a diagnosis of separation anxiety disorder. The prevalence of separation anxiety disorder is 4% in school-aged children, and is approximately 1% in adolescents. It occurs equally amongst boys and girls. Answer A: Agoraphobia involves the fear of being in a place or situation from which escape might be difficult or embarrassing or in which help may not be available in the event of having unexpected panic symptoms. Agoraphobic fears typically involve being afraid to go outside of the home alone, being in a crowded place like a shopping center or a concert, standing in a line, traveling on a bus or a train, or being on a bridge. In agoraphobia, the feared situation is avoided or endured with marked distress or requires the presence of a companion. In this case scenario, the patient is willing to go to the shopping center with his parents. His fear is not about being alone in a crowded place, but about making sure the parents are safe and not in danger. He would rather stay at home with his father than go to school without either parent at this point. Answer B: To address concerns about potential over-diagnosis and over-treatment of bipolar disorder in children, a new diagnosis, disruptive mood dysregulation disorder, is included for children up to age 18 years who exhibit persistent irritability and frequent episodes of extreme behavioral dyscontrol. Answer C: Major depressive disorder (MDD) is diagnosed when a patient has at least one depressive episode which encompasses the presence of at least five or more criteria nearly every day within the same two-week period, and represents a change from previous functioning. At least one of the symptoms is depressed mood or anhedonia (loss of interest). The mnemonic that can be useful for the criteria is SIGECAPS, and the following is a description of each criteria: S = decreased sleep or increased sleep; I = decreased interest in most activities; G = increased feelings of worthlessness or guilt; E = decreased energy or fatigue; C = decreased concentration on tasks or activities or in decision-making; A = decreased or increased appetite and weight loss or weight gain may be present; P = psychomotor agitation or retardation; S = recurrent thoughts of death or suicidal ideation. The symptoms cause clinically significant impairments in social, occupational, or academic functioning. In children, MDD presents with sad or irritable mood and/or a loss of interest or pleasure in the child's usual activities. In this case scenario, the patient is engaging in his usual activities at home and does not exhibit a sad or irritable mood throughout the day when he is at home with his father. Rather, his anxiety and crying manifests only when it comes time for separation from his parents. Children with depression may manifest somatic and physical complaints such as headaches, stomach aches, leg pains, etc. They may also exhibit psychotic symptoms and may also have symptoms of school difficulties, refusal to go to school, and weight changes. In this case scenario, the patient's somatic complaints of stomach aches occur only in the mornings when it is time to go to school, and this is classically seen in child with separation anxiety disorder. Answer E: School refusal may be due to social anxiety disorder (previously known as social phobia in DSM-IV). In such cases, the school avoidance is due to fear of being judged negatively by others rather than to worries about being separated from the attachment figures. Bottom Line: Separation anxiety disorder occurs in children and adolescents when they experience excessive anxiety at being separated from attachment figures or when they experience fear about harm befalling the attachment figures. COMBANK Insight : Under DSM-5, Separation anxiety disorder is no longer a disorder that has its onset in childhood; there is now no specified age of onset. Accordingly, separation anxiety disorder is now categorized under Anxiety Disorders rather than Disorders Usually First Diagnosed in Infancy, Childhood, or Adolescence.

A 55-year-old male presents to the office concerned about a mass on his upper arm. The patient believes that it is cancerous and has grown in size since first discovered one year ago. Physical examination reveals a small, soft, painless soft tissue mass that is moderately mobile. A review of his medical records indicates an MRI was performed revealing the mass had an isodensity with adipose tissue. The most likely diagnosis is A. anxiety disorder secondary to a general medical condition B. body dysmorphic disorder C. functional neurological symptom disorder D. illness anxiety disorder E. somatic symptom disorder

The correct answer is: D The disease preoccupation and conviction this patient exhibits despite medical reassurance is characteristic of hypochondriasis, now known as illness anxiety disorder in DSM-5, a somatoform disorder that is diagnosed when the preoccupation persists six months or more, is not of delusional intensity and creates significant impairment. The belief is not restricted to physical appearance as in body dysmorphic disorder but is rather based on fear of having a serious disease. From the description provided the patient most likely has a lipoma but fears the worst and has poor insight into his condition. Answer A: A diagnosis of anxiety disorder due to general medical condition would require evidence that the patient's psychic distress was the direct physiological consequence of a general medical condition and that it is not better accounted for by another mental disorder. In this case the patient most likely has a lipoma that would not cause anxiety symptoms and so his condition is better explained by a diagnosis of hypochondriasis. Answer B: Body dysmorphic disorder is characterized by marked preoccupation with an imagined defect in physical appearance that is excessive and causes distress. In the above case the patient's preoccupation is more with fear of having a serious illness than with his physical anomaly, as evidenced by his participation in many diagnostic tests. Answer C: Patients with conversion disorder, now known as functional neurological symptom disorder in DSM-5, have symptoms involving voluntary motor or sensory function that suggest neurological illness, are not intentionally produced but cannot be explained by a medical condition, substance effect or cultural behavior or experience. There are associated psychological factors associated with the symptoms that are often preceded by stress. If symptoms were intentionally produced, factitious disorder or malingering would be considered. Answer E: Somatization disorder, now known as somatic symptom disorder in DSM-5, is defined as multiple physical complaints (which must include four pain symptoms, two gastrointestinal symptoms, one sexual symptom and one pseudo neurological symptom) beginning before age of 30 years that result in treatment being sought or significant impairment in functioning, with no medical condition or substance fully explaining the symptoms. This patient complains only of the physical lump on his trunk and his fear of it representing serious illness despite reassurance is most consistent with hypochondriasis, a different illness within the category of somatoform disorders. Bottom Line: The somatoform disorders include somatization disorder, conversion disorder, pain disorder, hypochondriasis and body dysmorphic disorder and describe a constellation of illnesses characterized by preoccupation with physical illness or deformity that is not due to a known medical condition. They differ from factitious disorders in that symptoms are not intentionally feigned and from malingering in that there is no primary gain. Rather, patients suffering from somatoform disorders derive secondary gain from assuming the sick role. COMBANK Insight : In DSM-5, somatoform disorders are now referred to as somatic symptom and related disorders. The DSM-5 classification reduces the number of these disorders and subcategories to avoid problematic overlap. Diagnoses of somatization disorder, hypochondriasis, pain disorder, and undifferentiated somatoform disorder have been removed. Individuals previously diagnosed with somatization disorder will usually have symptoms that meet DSM-5 criteria for somatic symptom disorder, but only if they have the maladaptive thoughts, feelings, and behaviors that define the disorder, in addition to their somatic symptoms. Because the distinction between somatization disorder and undifferentiated somatoform disorder was arbitrary, they are merged in DSM-5 under somatic symptom disorder. Individuals previously diagnosed with hypochondriasis who have high health anxiety but no somatic symptoms would receive a DSM-5 diagnosis of illness anxiety disorder (unless their health anxiety was better explained by a primary anxiety disorder, such as generalized anxiety disorder). Some individuals with chronic pain would be appropriately diagnosed as having somatic symptom disorder, with predominant pain. For others psychological factors affecting other medical conditions or an adjustment disorder may be more appropriate.

A 23-year-old man presents to his primary care physician for low energy level. He has been tired and listless for approximately three weeks but has been unable to sleep well. He awakens early in the morning and cannot get back to sleep. He denies difficulty falling asleep at night. He has trouble concentrating on his work as a laboratory technician. He does admit to feeling "mostly worthless," and he also notes a decreased appetite. He no longer enjoys visiting with friends or family. He has no suicidal ideation. He does not smoke but drinks approximately three beers daily. On physical examination, his speech is soft and slow. He moves slowly, and his affect is blunted and congruent with mood. He denies hallucinations and delusions. Which of the following is the most likely diagnosis? A. adjustment disorder with depressed mood B. bipolar II disorder C. dysthymic disorder D. major depressive disorder E. minor depressive disorder

The correct answer is: D This patient is experiencing signs and symptoms consistent with depression. Major depressive disorder is diagnosed by the presence of a major depressed mood or anhedonia PLUS four of the following symptoms for at least two weeks: 1. Excessive Sleep or insomnia 2. Loss of Interest 3. Guilt or feelings of worthlessness 4. Decreased Energy 5. Loss of Concentration 6. Change in Appetite 7. Psychomotor retardation or agitation 8. Suicidality (remember SIG E CAPS) Patients need to have these symptoms all or most of the time for at least two weeks and must not have a history of mania or hypomania. This patient reports three weeks of changes in sleep, concentration difficulties, loss of interest, feelings of worthlessness, and decreased appetite. He also has psychomotor retardation on examination. Taken together, all these features point towards a diagnosis of major depressive disorder. Answer A: Adjustment disorder with depressed mood is characterized by depressed mood not meeting criteria for major depressive disorder in response to a specific event. Criteria for adjustment disorder include development of emotional or behavioral symptoms within three months of a stressful life event, the symptoms are not those bereavement, and the symptoms resolve within six months of the stressor being eliminated. This patient meets criteria for major depressive disorder, as described above, and no inciting event is given in the question stem. Answer B: Bipolar II disorder requires a history of hypomania for diagnosis. There is no history of hypomania included in the stem. Answer C: Dysthymic disorder is diagnosed in patients with low depressed mood for most or all of two years, not meeting criteria for major depressive disorder. This patient meets criteria for major depressive disorder and only has had symptoms for three weeks. Answer E: There is no formal diagnosis of minor depressive disorder in the DSM-IV nor DSM-5. Bottom Line: Major depressive disorder is diagnosed by the presence of a depressed mood or anhedonia PLUS four of the following symptoms for at least two weeks: excessive sleep or insomnia, loss of interest, guilt or feelings of worthlessness, loss of concentration, change in appetite, psychomotor retardation or agitation, and suicidality.

A 54-year-old male complains of decreased appetite, poor concentration, restlessness, and early morning awakenings over the past month. He lacks interest in his usual activities and reports to be unable to experience pleasure. He also reportedly hears voices that say he is worthless. Past medical history is pertinent for hypothyroidism due to thyroid ablation therapy treated with levothyroxine. Laboratory studies are obtained revealing a TSH of 12.9 mcU/mL. The most likely diagnosis is A. adjustment disorder with mixed disturbance of emotions and conduct B. bipolar disorder most recent episode manic with psychotic features C. major depressive disorder with psychotic features D. mood disorder secondary to a general medical condition E.schizoaffective disorder

The correct answer is: D This patient presents with a distinctive period of mood disturbance characterized by anhedonia, change in appetite impaired concentration, psychomotor agitation and sleep disturbance which is characteristic of a major depressive episode. However, psychotic features (mood-congruent auditory hallucinations) are present and the mood disorder was triggered by a medical condition - he has a history of hypothyroidism and there is laboratory evidence of significantly elevated TSH consistent with hypoactive thyroid function. A diagnosis of major depressive disorder cannot be made if the disturbance is better accounted for by a general medical condition or by prescription or illicit substance intoxication/withdrawal. Endocrinopathies are a common differential diagnosis for mood disorders secondary to general medical conditions, with hypothyroidism linked to depression and hyperthyroidism to mania. Other medical conditions that can cause a depressive episode include Parkinson's disease, cerebrovascular disease, cancers (particularly pancreatic) and collagen vascular disease. Answer A: A diagnosis of adjustment disorder requires symptoms of a mood disorder and/or a behavioral disturbance to arise within 3 months of an identifiable stressor. In this case, the symptoms meet criteria for mood disorder secondary to a general medical condition as they were triggered by a medical illness, not a psychosocial stressor. Answer B: The patient's symptoms are consistent with a depressive episode, not a manic episode. A manic episode is characterized by a period of distinctly elevated, expansive or irritable mood lasting at least one week or any duration if hospitalization is needed. Three of the following symptoms (or four if the mood is irritable) are also present: decreased need for sleep, increase in goal-directed activity or psychomotor agitation, distractibility, flight of ideas or racing thoughts, grandiosity and excessive involvement in pleasurable activities that are risky or often result in painful consequences such as sexual indiscretions or foolishly investing money. Uncontrolled hyperthyroidism can cause manic symptoms and anxiety. Answer C: The history of hypothyroidism and laboratory evidence of elevated TSH disqualify a diagnosis of major depression with psychotic features - the patient's symptoms are better accounted for by a diagnosis of mood disorder secondary to hypothyroidism. A major depressive episode is characterized by a period of at least two weeks during which either depressed mood or loss of interest or pleasure (anhedonia) is present. A total of at least five of the following symptoms must be present: depressed mood, anhedonia, significant unintentional weight loss or gain, insomnia or hypersomnia, psychomotor agitation or retardation, fatigue, feelings of worthlessness or excessive/inappropriate guilt, poor concentration or indecisiveness and suicidal ideation. Again, these symptoms must not be better accounted for by a general medical condition or substance intoxication/withdrawal. Answer E: While psychotic features are present, the patient's symptoms do not meet criterion A for schizophrenia: two or more psychotic symptoms must be present over at least one month (delusions, hallucination, disorganized speech, disorganized behavior or catatonia, negative symptoms). If he experienced bizarre delusions or auditory hallucinations consisting of two or more voices conversing or a voice keeping up a running commentary on his behavior or thoughts then only one symptom would be needed. Furthermore, there is no evidence of a two-week period of psychotic symptoms being present in the absence of mood symptoms and as mentioned above the disturbance is linked to his uncontrolled hypothyroidism. Bottom Line: Endocrinopathies are a common differential diagnosis for depression and mania. Psychotic features can be present with mood disorders or in schizoaffective disorder. Remember that in order to make a diagnosis of schizoaffective disorder, there must be evidence of a two-week period during which psychotic symptoms were present in the absence of mood symptoms. COMBANK Insight : There is an entire category of questions on the Psychiatry COMAT devoted to conditions caused by a general medical condition. Examples include diabetes (and a multitude of other conditions) and depression, epilepsy and psychotic disorder, and personality changes due to brain conditions.

A 22-year-old college student presents with voices that say phrases such as "you are the king" and "you must leave." He has lost interest in completing his school assignments, has been watching more television, and now reports that special messages are being relayed to him by news broadcasters. He also has stopped using text messages on his cell phone because he believes the messages are being recorded in a database owned by evil people. He has been having these symptoms for the last 4 months now, and he subsequently dropped out of school this week. Social history reveals that the he grew up with a stable family life. Mental status examination reveals a disheveled appearing male with reduced affect, dysphoric mood, and circumstantiality in thought processes. The most likely diagnosis is A. adjustment disorder B. brief psychotic disorder C.schizoid personality disorder D. schizophrenia E. schizophreniform disorder

The correct answer is: E Schizophreniform disorder fulfills the same criteria as the Axis I diagnosis of schizophrenia (please see explanation for Answer D) except for the duration of time criteria. Schizophreniform disorder symptoms occur for more than one month but last less than six months, whereas, brief psychotic disorder symptoms last for one month or less. On the other hand, schizophrenia is diagnosed when the symptoms occur for more than six months. Two-thirds of individuals with schizophreniform disorder progress to being diagnosed with schizophrenia. In this case scenario, the patient fulfills the symptomatic criteria for schizophrenia but the diagnosis currently would be schizophreniform disorder as his symptoms only started about four months ago. Bottom Line: Schizophreniform disorder occurs when psychotic symptoms occur for less than 6 months but greater than one month.

A 72-year-old overweight male presents with his son for memory problems. History reveals the patient has been forgetting to take his medications on time and to pay his phone bill on time, and has been forgetting to meet his friends for a game of chess on Friday evenings. The patient's son reports this has progressively worsened over the past 8 months. Past medical history includes diabetes, hypertension, migraines, osteoarthritis, and a prior stroke three years ago. Physical examination reveals weakness in left arm and in left leg. The mood is normal with full affect and the patient is alert, however, he believes the year to be 2008 and that the current president is Ronald Reagan. A CT scan of the head is obtained and demonstrates diffuse, microvascular ischemic changes. The most likely diagnosis is A. Alzheimer's dementia B. delirium C.major depressive disorder D. normal pressure hydrocephalus E. vascular dementia

The correct answer is: E Vascular dementia comprises of step-wise progression of cognitive impairments as evidenced by memory impairment and one or more of the following cognitive disturbances: aphasia (disturbance in the formulation and comprehension of language), apraxia (inability to execute learned purposeful movements), agnosia (inability to recognize objects), or disturbance in executive functioning. The etiology of vascular dementia may be stroke, chronic subdural hemorrhage, post-anoxic injury, or diffuse white matter disease. Risk factors for vascular dementia include those associated with vascular disease, vasculitis, or embolic disease. Patients usually have a cardiac history or risk factors including hypertension, heart disease, hypercholesterolemia, diabetes, etc. Focal neurological signs (deficits in sensory or motor function) or laboratory/CT head findings demonstrating cerebrovascular disease may be present as etiologically related to the disturbance. This patient presents with a history of diabetes, hypertension, and stroke, and has deficits of weakness and numbness. His CT scan also demonstrates evidence of stroke. He presents with memory problems concerning previously learned information and presents with disturbances in executive functioning (paying phone bills and taking his medications). Answer A: Alzheimer's dementia comprises of cognitive impairments as evidenced by memory impairment and one or more of the following cognitive disturbances: aphasia (disturbance in the formulation and comprehension of language), apraxia (inability to execute learned purposeful movements), agnosia (inability to recognize objects), or disturbance in executive functioning. Cognitive deficits can cause significant impairments in social or occupational functioning. The course is characterized by gradual onset and continuing cognitive decline. However, there are no focal sensory or motor deficits in Alzheimer's dementia. The cognitive deficits are not due to CNS conditions such as cerebrovascular disease, Parkinson's disease, subdural hematoma, normal pressure hydrocephalus, brain tumor. In this scenario, the patient has sensory and motor deficits along with memory impairments, pointing towards vascular dementia instead of Alzheimer's dementia. Answer B: Delirium involves a disturbance of consciousness with diminished ability to focus, maintain, or shift attention with an alteration in cognition or the development of a perceptual disturbance not attributable to dementia. The level of consciousness may be impaired, which is not the case in dementia. The disturbance evolves over an acute or sub-acute period of few hours or days with a fluctuating course during the day time. There is evidence that the condition is caused by a general medical condition as vulnerable brain structures undergo oxidative stress due to some form of insult to the body (examples: surgery, medical illness, trauma) leading to a reversible condition. This patient's cognitive impairments began eight months ago and his level of consciousness is stable and unimpaired, which does not fit the criteria for delirium. Answer C: Major depressive disorder may cause cognitive impairments and may present with poor concentration in activities. However, for a diagnosis of major depressive disorder, one needs to have at least either depressed mood or decreased interest in activities, and neither symptom is mentioned in this patient's history. His mood is normal, and his forgetting to play chess with his friends on Friday evenings is attributed to the memory impairment, rather than a lack of interest in the activity. Answer D: Normal pressure hydrocephalus (NPH) presents more commonly in males with a "magnetic gait" followed by dementia and urinary incontinence. This patient does not have magnetic gait nor urinary incontinence. Cognitive impairments may include mild memory problems and impaired concentration. CT scan of the head in patients with NPH demonstrates ventriculomegaly and diffuse cerebral atrophy. Diagnosis of NPH relies on clinical improvement of symptoms following a series of lumbar punctures. Bottom Line: Patients with vascular dementia manifest cognitive impairments and memory problems in a step-wise, progressive way along with focal sensory or motor deficits, and they usually have risk factors for stroke or cerebr

A 68-year-old female presents with blepharospasm and axial dystonia after the long-term use of ziprasidone.

The patient in this case scenario is having symptoms of tardive dyskinesia. Tardive dyskinesia (TD) occurs as a potential consequence of the long-term use of anti-psychotic medications (dopamine blockers). TD most often affects the mouth and tongue, presenting as lip smacking, sucking, or facial grimacing. It may also involve limb or truncal choreoathetosis or dystonia. The risk is higher with typical anti-psychotics compared to atypical anti-psychotics. The patient groups at highest risk for the development of TD include: patients with mood disorders, patients with structural brain lesions, elderly females, children, and non-Caucasian patients. The presumed underlying pathophysiology which is thought to cause TD is increased sensitivity to dopamine receptors over time. TD is often found to be generally occurring when the medication is reduced or stopped. It also consists of orofacial dyskinesias, choreoathetosis, motor tics and vocal tics.

What are some ways to differentiate Generalized Anxiety disorder from Panic Disorders?

●A panic disorder is defined as recurrent panic attacks and one month or more of worrying about future attacks or changing a behavior in order to avoid an attack. ●Generalized anxiety disorder is a psychiatric disorder characterized by uncontrollable, excessive worrying that is hard to control, causes significant impairment, and occurs on most days for at least six months. Most patients with generalized anxiety disorder present with symptoms of hyperarousal, hyperactivity, and muscle tension. They typically do not present with cardiorespiratory or gastrointestinal distress. ●Panic attack occurs when there is an intense, spontaneous fear along with cardiorespiratory, gastrointestinal, or neurologic changes of chest pain, shortness of breath, nausea, vomiting, dizziness, or headache.

cognitive impairments of schizophrenia

●Processing speed ●Attention ●Working memory ●Verbal learning and memory ●Visual learning and memory ●Reasoning/executive functioning ●Verbal comprehension ●Social cognition

Disorganization of schizophrenia

●Tangential speech - The person gets increasingly further off the topic without appropriately answering a question. ●Circumstantial speech - The person will eventually answer a question, but in a markedly roundabout manner. ●Derailment - The person suddenly switches topic without any logic or segue. ●Neologisms - The creation of new, idiosyncratic words. ●Word salad - Words are thrown together without any sensible meaning.

What are 5 common Negative symptoms?

●alogia- dysfunction of communication (restriction in fluency/productivity of thought and speech) ●affective blunting-restriction in range and intensity of emotional expression ●asociality -reduced social drive/interaction ●anhedonia -reduced ability to experience pleasure ●avolition-reduced motivation or persistence in the initiation of goal directed behavior

Ramelteon (Rozerem)

●melatonin receptor agonist with high selectivity for human melatonin receptors. ●It is indicated for insomnia characterized by difficulty with sleep onset. MT1 and MT2 are thought to promote sleep and be involved in maintaining circadian rhythm and a normal sleep-wake cycle. Ramelteon does not cause rebound insomnia or withdrawal symptoms at discontinuation and is approved for prolonged use.

A 13-year-old male presents with his mother to the outpatient psychiatric clinic for evaluation of behavioral disturbances. History reveals that he has been home-schooled for the past six years, does well in most of his subjects, and has attended after-school programs at the local small private school for mathematics and soccer. The mother reports that, for the past three months, the patient refuses to go outside for shopping to the mall or grocery store and becomes angry and anxious if another family member asks to take him out for shopping. The family had also booked tickets for a circus show during their upcoming spring break, and the patient adamantly refuses to attend. When asked about why he does not want to go to these places anymore, he replies, "there are just too many people there." Which of the following is the most likely diagnosis? A. agoraphobia without panic attacks B. antisocial personality disorder C.panic attack D. separation anxiety disorder E. social phobia

The correct answer is: A Agoraphobia involves the fear of being in a place or situation from which escape might be difficult or embarrassing or in which help may not be available in the event of having unexpected panic symptoms. Agoraphobic fears typically involve being afraid to go outside of the home alone, being in a crowded place like a shopping center or a concert, standing in a line, traveling on a bus or a train, or being on a bridge. The feared situation is avoided or endured with marked distress or requires the presence of a companion. In this scenario, the patient is home-schooled, and seems to attend the after-school programs at the small private school with no problems. However, the patient is afraid of going to places where there are large crowds of people, and feels very anxious about it. The scenario in the question does not describe a full-blown panic attack, thus making the diagnosis agoraphobia without panic attacks. Answer B: Antisocial personality disorder is diagnosed at or above 18 years of age. Individuals with anti-social personality disorder have long criminal records of repetitive unlawful acts and socially irresponsible behaviors that began prior to the age of 15 years (otherwise known as conduct disorder in children and adolescents). These individuals lack remorse for the harm they cause and are generally unconcerned for the feelings and rights of others. Most of these individuals are male. Frequent co-morbid disorders include ADHD, somatization disorder, and substance abuse disorders. Answer C: A panic attack is a discrete period of intense fear or discomfort, in which four out of twelve criterion symptoms develop abruptly and reached a peak within ten minutes. The ten criterion symptoms include: palpitations or accelerated heart rate, sweating, trembling or shaking, sensations of shortness of breath or smothering, feeling of choking, chest discomfort or chest pain, nausea, feeling dizzy or light-headed, derealization (feelings of unreality) or depersonalization (being detached from oneself), fear of losing control, fear of dying, paresthesias (numbness or tingling sensations), and chills or hot flushes. A panic attack can be specified as co-occurring with another diagnosis such as specific phobia, agoraphobia, or social phobia. Answer D: Separation anxiety disorder occurs in children or adolescents, before the age of eighteen years old, and involves excessive anxiety concerning separation from the home or from those to whom the individual is attached. Three or more of the following criterion have to be present for diagnosis: recurrent excessive distress when separation from home or major attachment figures occurs or is anticipated, excessive worry about losing or harm befalling major attachment figures, excessive worry that an untoward event (such as kidnapping, getting lost) will lead to separation from the major attachment figure, persistent refusal to go to school or elsewhere because of fear of separation, persistent reluctance to be alone at home or outside without the major attachment figures, persistent refusal to sleep away from home or away from the major attachment figure, recurrent nightmares about separation, or repeated complaints about physical symptoms (such as vomiting, stomach aches, headaches, nausea) when separation from major attachment figures occurs or is anticipated. For diagnosis, the disturbance has to last at least four weeks, and the disturbance causes clinically significant distress or impairment in social, academic, or other important areas of functioning. Answer E: Social phobia, otherwise known as social anxiety disorder, comprises of marked and persistent fear of one or more social or performance situations in which the person is exposed to unfamiliar people or to possible scrutiny by others. Social phobia is characterized by fear and avoidance of circumscribed social situations such as eating or speaking in public or more commonly, all social situations. The individual fears that he or she will demonstrate anxiety symptoms or act in a way that will be humiliating or embarrassing. Exposure to the feared social situation provokes anxiety. The feared performance or social situations are avoided or they are endured with intense anxiety or distress. The avoidance and distress interfere significantly with the person's normal routine, academic or occupational functioning, or social activities or relationships. In this scenario, the patient is not anxious about performing an act in public, but anxious about being in crowded places. Bottom Line: Agoraphobia is the fear of being in crowded places or in places from which escape is difficult and may or may not occur with recurrent panic attacks.

negative symptoms of schizophrenia

pathwway-dopamin

Delusions

Delusions are erroneous beliefs that usually involve a misinterpretation of perceptions or experiences. (persecutory, referential, somatic, religious, or grandiose). In persecutory delusions the person believes he or she is being tormented, followed, tricked, spied on, or ridiculed. In referential delusions the person believes that certain gestures, comments, passages from books, newspapers, song lyrics, or other environmental cues are specifically directed at him or her.

A 54-year-old man with chronic alcohol abuse presents with confusion and visual hallucinations. Vital signs are HR 113, BP 164/95, finger stick blood glucose 113 mg/dL, and T 100.5°F. Physical examination reveals a disheveled man with tremors, tongue fasciculations, and agitation. What management should be pursued?

Not as likely serotonin syndrome bc no muscle rigidity/clonus, didn't mention SSRI Likely Alcohol withdrawal symptoms are present and know past of abuse- TX: supportive care and benzodiazepines (diazepam).

A 5-year-old male presents with a sad mood, markedly decreased interest in playing with his toys and watching his favorite cartoons, low energy, decreased appetite, and feeling as if he should not be around anymore but denies thoughts of self-harm. These symptoms have been going on for about three weeks now after recently moving from another town in the middle of the school year. He denies any auditory or visual hallucinations. He does not have any known medical problems and past psychiatric history is significant only for separation anxiety disorder when he started preschool and when he started kindergarten. His family is loving and caring and there appears to be no signs of abuse. The most appropriate treatment is A. cognitive behavioral therapy B. dialectical behavior therapy C. family therapy D.group therapy E. play therapy

The correct answer is: E The child in this case scenario currently has symptoms of a major depressive episode, possibly due to the stressor of moving to a new town, home, and school. He meets 5 of the following 9 criteria during the same 2-week period: depressed mood most of the day and nearly every day, markedly decreased interest or pleasure, decrease or increase in appetite or weight loss/gain, insomnia or hypersomnia nearly every day, psychomotor agitation or retardation, fatigue or loss of energy, decreased ability to think or concentrate, feelings of worthlessness or guilt, recurrent thoughts of death or suicidal ideation. One of the 5 criterion symptoms has to be either depressed mood or decreased interest/pleasure. Play therapy is generally used with young children ages 3-11 years old and would be the best first choice in this case given the patient's age. Play therapy allows them to express themselves and their emotions, feelings, and experiences through a natural and self-guided process as they can relate well to toys and objects through play. Therapists can communicate with children through play and it helps them to identify their strengths and weaknesses, their stressors, and their support systems. Play therapy often begins with first observing the child playing with the toys to elicit the cause of stressor or disturbed behaviors. Interaction through play allows therapist and child to communicate, discuss, and explore issues. Answer A: This child is not old enough for CBT. Cognitive behavioral therapy is designed to help individuals alter negative thoughts, thought distortions, negative self-esteem, and to develop better coping skills for stressors, thus enhancing his/her involvement in normal and rewarding, productive activities. For CBT, individuals need to have some understanding of such negative thoughts and would need to be willing to work 1:1 with a therapist. Given the age of the child in this scenario, play therapy would be a better idea at this time. Answer B: Dialectical behavior therapy (DBT) is a type of therapy that was originally developed by Marsha Linehan to help individuals modify thoughts of self-harm, suicidal thinking, and substance abuse. This form is used often for individuals with borderline personality disorder and chronic suicidality. This approach works towards helping people to increase their emotional and cognitive regulation by learning about the triggers that lead to their emotionally reactive states and helping to develop better coping skills for their stressors. Answer C: Family therapy would be more suitable when there are family dynamics contributing to stressors that are exacerbating symptoms. In this scenario, it appears that the child is experiencing depressive symptoms due to the stressor of moving from another town and leaving his old school and friends behind. Answer D: Group therapy would be more suitable for older kids, adolescents, and adults because these groups of individuals are more likely to be able to sit in group settings and discuss/share about their issues and they can learn from each other. 5-year-old children would have a difficult time managing themselves in a meaningful group discussion. Bottom Line: Play therapy would be the best choice in this case as first choice given patient's stressors and age.

A physician becomes upset because many of her patients in the new clinic are alcoholics. This is causing her to have trouble with concentration and focusing at work, and she often finds herself becoming easily irritable and frustrated with her patients and their families. Social history reveals that she was an only child and was raised primarily by her father who was an alcoholic. She states that she had to spend a lot of time with him alone while her mother went to work. This is consistent with A. countertransference B. pseudocyesis C. rationalization D.reaction formation E. sublimation

The correct answer is: A Countertransference comprises of the reactions that the provider has towards the doctor-patient relationship, and these reactions may be quite negative in which the doctor finds him/herself to be disliking the patient very much and having a challenging time working with that patient. On the other hand, the reactions may be disproportionately positive and may even be idealizing or eroticized. Oftentimes, when countertransference occurs, the doctor feels like the patient reminds them of a close friend or a family member, and this may impact or interfere with the doctor-patient therapeutic relationship. In such cases, it is recommended for the provider to consult with a colleague to discuss how to handle and/or overcome the feelings of countertransference. Answer B: Pseudocyesis, also known as false pregnancy, is a rare condition in which a nonpregnant female patient experiences the signs and symptoms of pregnancy (abdominal distention, labor pains, breast enlargement, hyperpigmentation, cessation of menses, morning sickness, nausea, etc) and the patient then comes to firmly believe that she is pregnant. Pseudocyesis demonstrates the power of the psyche to dominate the soma, possibly via central input at the level of the hypothalamus. There are certain predisposing psychological factors that may occur in some patients such as the following: pathological wish for pregnancy, intense fear of pregnancy, sexual ambivalence, conflicts regarding childbearing, grief reaction to a loss through miscarriage, abortion, tubal ligation, or hysterectomy. Answer C: Rationalization is defined as an unconscious defense mechanism in which irrational or unacceptable behaviors, motives, or feelings are logically justified or made consciously tolerable by plausible and acceptable means. The perception of the event or behavior is usually distorted so that the negative outcome seems reasonable. An example of rationalization is when a teenager fractures his elbow and thinks it is a good thing because it prevented him from getting drugs and getting in trouble with the law. Answer D: Reaction formation is defined as the unconscious defense mechanism in which an individual develops a socialized attitude or an interest that is the direct antithesis of an infantile wish, desire, or impulse that is harbored consciously or unconsciously. These impulses or urges are usually unacceptable to the ego. An example would be a businessman, who engages in fraud accounts to earn more large sums of money, finds himself donating a generous sum of money to a local charitable organization. In other words, he is engaging in charity work to unconsciously avoid the personally unacceptable and guilt-ridden impulses of cheating and fraud that he is doing in his business. Answer E: Sublimation is defined as the unconscious defense mechanism in which the energy associated with unacceptable impulses or drives is diverted into personally and socially acceptable channels; unlike other defense mechanisms, it offers some degree of minimal gratification of the instinctual drive or impulse. An example of this would be a professional wrestler who used to get into many physical fights with others at school and was known as a bully throughout school. Bottom Line: Countertransference comprises of the feelings or emotions the provider has for the patient, and can be positive or negative.

A 6-year-old male was recently diagnosed with attention-deficit/hyperactivity disorder. During a visit at the psychiatrist's office, the patient's mother expresses concern regarding the child's future personal and social development. She says that her brother was diagnosed with the same disorder at a young age and is, unfortunately, now in prison. There is evidence that ADHD is associated with the development of A. antisocial personality disorder B. schizoaffective disorder C. separation anxiety D.substance abuse E. unemployment

The correct answer is: A Attention-deficit/hyperactivity disorder (ADHD) is considered to be a neurobehavioral developmental disorder and affects between 3-5% of school aged children with symptoms mostly starting before 12 years of age. It is characterized by a persistent pattern of impulsiveness, distractibility, and inattention, with or without a component of hyperactivity. Conduct disorder and ADHD frequently co-exist, and these children have high propensity to develop antisocial personality disorder in adulthood. Oppositional defiant disorder, bipolar disorder, and anxiety problems are also commonly found as co-morbid diagnoses in individuals with ADHD, albeit less commonly than conduct disorder. Answer B: Schizoaffective disorder - comparable to schizophrenia, bipolar disorder, and major depressive disorder - requires hallucinations, bizarre delusions, or other psychotic features as well as a major mood episode be present for a majority of the total disorders duration. There is no association between ADHD and schizoaffective disorder. Answer C: Separation anxiety is a normal phenomenon that occurs between 8-14 months of age. There is no association between ADHD and separation anxiety. Answer D: Whether children with ADHD have an increased risk of engaging in substance use during adolescence or adulthood is controversial. Answer E: In studies that followed children with ADHD into early adulthood, the rate of employment of ADHD cohorts did not differ from that of controls. Other studies displayed the latter. Regardless, the association is controversial. Bottom Line: Children with attention-deficit hyperactivity disorder often have the co-morbid diagnosis of conduct disorder, putting them at an increased risk of developing antisocial personality disorder.

A 42-year-old morbidly obese male presents with chronic fatigue. He reports that he feels tired all day despite obtaining 9 hours of sleep daily, and frequently awakens with a headache. He recently was fired from his job and attributes this to difficulty concentrating and repeatedly falling asleep while at work. He later began having trouble in his marital relationship due to a poor libido and believes it is because he "is a worthless person." He reports the feelings of hopelessness are worse in the morning. Vitals reveal a weight of 170 kg (375 lbs) and height of 1.75 m (5' 6") and a blood pressure of 140/91 mmHg. Physical examination is significant for a large pannus and increased neck girth. The most likely diagnosis is A. depressive disorder with melancholic features B. hypothyroidism C.idiopathic hypersomnolence D. narcolepsy E. obstructive sleep apnea

The correct answer is: E Several observational studies have also found that patients with obstructive sleep apnea (OSA) have a 2x higher incidence of depression compared to those without the condition. OSA is a sleep disorder caused by intermittent obstruction of air flow that produces periods of apnea during sleep. Hypoxia results from each apneic episode, which arouses the patient and prevents adequate, restorative sleep. OSA is most commonly seen in middle-aged men. Obesity is the main risk factor, however, familial facial features (deep overbite with small chin) and structural abnormalities (enlarged tonsils, nasal polyps, deviated septum) also may play a role. Clinical features include daytime sleepiness, impaired cognitive function, depression, hypertension, morning headaches, polycythemia, nocturnal choking, nocturia, and decreased libido. Sleeping partners may report loud snoring in all positions and intermittent episodes of silence and/or choking. Diagnosis is made using polysomnography, which demonstrates multiple breathing pauses during sleep (must have at least five obstructed breathing events). Management of mild to moderate OSA includes lifestyle modifications such as weight loss, avoiding alcohol and sedative, and avoiding supine position during sleep. Conservative treatment options for severe OSA (> 20 apneic episodes/night) include continuous positive airway pressure (CPAP) or mandibular repositioning splint. CPAP is currently the treatment of choice due to better adherence and proven efficiency. OSA refractory to conservative management may require surgical treatment with uvulopalatopharyngoplasty or, rarely, tracheostomy. Answer A: This patient would more appropriately be characterized as having depressive disorder due to a general medical condition, being sleep apnea. He does meet criteria for a depressive episode, but there is evidence from the history and physical examination that the disturbance is the direct pathophysiological consequence of another medical condition. According to DSM-5, those with depressive disorder with melancholic features have loss of pleasure in all (or almost all) activities, lack of reactivity to usually pleasurable stimuli, and three (or more) of the following: a distinct quality of depressed mood characterized by profound despondency, despair, and/or moroseness or by so-called empty mood, depression that is regularly worse in the morning, early-morning awakening, marked psychomotor agitation or retardation, significant anorexia or weight loss, or excessive or inappropriate guilt. Answer B: Hypothyroidism presents with similar symptoms as OSA (e.g., increased fatigue, depression, decreased libido) and also can be a predisposing factor for OSA as well as depression. Sleep apnea occurs in some patients with hypothyroidism, mostly as a result of macroglossia due to tissue infiltration. Therefore, it may be appropriate to check a TSH level in all patients with suspected OSA to rule out concomitant thyroid dysfunction. It is important to note that although this answer is correct, it is not the most correct option available in light of the patient's sleep disturbances which negatively impact multiple components of his daily activities. Answer C: Idiopathic hyper somnolence (IHS) is a condition characterized by long sleep duration and daytime sleepiness. Patients commonly require > 10 hours a sleep per night and it is usually quite deep, with significant difficulty arousing. Patients will be tired throughout the entire day and will usually take long (several hour) naps that are unrefreshing. IHS is commonly diagnosed in adolescence or young adulthood. Answer D: Narcolepsy is an inherited sleep disorder that results in the induction of REM sleep involuntarily at random and inappropriate times. Patients commonly experience excessive daytime fatigue and involuntary "sleep attacks" that last several minutes. Other features include cataplexy (loss of muscle tone with intense emotional stimulus), sleep paralysis, and vivid hallucinations while either falling asleep or waking up. This patient's history of obesity is more consistent with the diagnosis of OSA. DSM-5 distinguishes narcolepsy - now known to be associated with hypocretin deficiency - from other forms of hypersomnolence (hypersomnolence disorder). Bottom Line: Obstructive sleep apnea is defined as the coexistence of excessive daytime sleepiness with multiple episodes of obstructed breathing per hour of sleep. The diagnosis is most commonly made in obese middle-aged males. Associated clinical features include depression, difficulty concentrating, hypertension, decreased libido, and loud snoring. Diagnosis is confirmed with polysomnography. Management options include lifestyle modification, CPAP, mandibular repositioning splint, and surgical treatments.

A 45-year-old male presents for follow-up for generalized anxiety disorder. He currently takes buspirone 15 mg three times daily and sertraline 200 mg every morning. These medications do help him cope, but he reports to still be symptomatic at times. A mental status exam reveals a well-groomed appearance, full affect, mild dysphoric mood with some nervousness, well-organized thought processes, fair concentration, and normal intellectual functioning without memory problems. Further history reveals the patient is interested in alternative non-pharmacologic therapy for his anxiety control. The psychiatrist mentions a research study involving osteopathic manipulative therapy. In order to support the hypothesis, subjects require a pre-treatment PET scan. The most likely location of an abnormality in this patient population will be within the A. amygdala B. caudate nucleus C.globus pallidus D. hippocampus E. putamen

The correct answer is: A Brain imaging studies have most consistently implicated the amygdala, anterior cingulate cortex, and insula in the pathophysiology of anxiety. In subjects with anxiety, studies are demonstrating that increased brain activity in the amygdala is demonstrated on PET scans. The limbic system anatomically comprises the subcallosal, the cingulate, and the parahippocampal gyri, the parahippocampal formation, the amygdaloid nucleus, the mammillary bodies, and the anterior thalamic nucleus. The limbic system, especially the amygdala, is involved in the bottom-up processing of anxiety, while the prefrontal cortex is involved the top-down regulation of anxiety. The amygdala is associated with anxiety, fear, and emotion. Specific brainstem nuclei, such as the locus coeruleus and raphe nucleus, release neurotransmitters and are interconnected with these regions. Chronic states of anxiety, worry, and hypervigilance are mediated in part by the GABA neurons from the limbic system. Through its connections, the amygdala can influence the body's response to environmental changes. In the sense of fear, the amygdala can change the heart rate, blood pressure, skin coloration, and the respiration rate. Answer B: The basal nuclei, also known as the basal ganglia, are applied to a collection of masses of gray matter situated within each cerebral hemisphere. The basal nuclei play a crucial role in the control of posture and voluntary movements. They are the corpus striatum, the amygdaloid nucleus, and the claustrum. One of these areas, the corpus striatum, is completely divided by the internal capsule which is a band of nerve fibers, into the caudate nucleus and the lentiform nucleus. The caudate nucleus is a large C-shaped mass of gray matter that is closely related to the lateral ventricle and lies lateral to the thalamus. The caudate nucleus can be divided into a head, a body, and a tail. The caudate nucleus and the putamen form the main sites that receive input to the basal nuclei. The caudate nucleus, along with other brain areas, is responsible for voluntary movements. Dopaminergic neuronal degeneration in the caudate nucleus, substantia nigra, globus pallidus, and putamen leads to Parkinson's disease. On the other hand, in Huntington's disease, there is a degeneration of the GABA-secreting, substance P-secreting, and acetylcholine-secreting neurons of the striatonigral-inhibiting pathway. Thus, the dopamine secreting neurons of the substantia nigra become overactive, and the nigrostriatal pathway inhibits the caudate nucleus and the putamen. This inhibition produces the abnormal movements seen in Huntington's disease. CT scans will show enlarged lateral ventricles in patients with Huntington's disease due to degeneration of the caudate nuclei. Answer C: The basal nuclei, also known as the basal ganglia, are applied to a collection of masses of gray matter situated within each cerebral hemisphere. The basal nuclei play a crucial role in the control of posture and voluntary movements. They are the corpus striatum, the amygdaloid nucleus, and the claustrum. One of these areas, the corpus striatum, is completely divided by the internal capsule which is a band of nerve fibers, into the caudate nucleus and the lentiform nucleus. The lentiform nucleus is buried deep in the white matter of the cerebral hemisphere, and is related medially to the internal capsule, which separates it from the caudate nucleus and the thalamus. A vertical plate of white matter divides the lentiform nucleus into a larger, darker portion, the putamen, and an inner lighter portion, the globus pallidus. The lighter color of the globus pallidus arises from the presence of a high concentration of myelinated nerve fibers. The globus pallidus forms the major site from which the output leaves the basal nuclei. Dopaminergic neuronal degeneration in the caudate nucleus, substantia nigra, globus pallidus, and putamen leads to Parkinson's disease. Answer D: The hippocampus, together with the parahippocampal gyrus, lies posterior to the amygdala, and is wrapped into the medial temporal lobe. The hippocampus plays a key role in the formation of anterograde declarative memory formation, which involves the formation of anterograde, new memories related to declarative memory, which involves facts and knowledge. Answer E: The basal nuclei, also known as the basal ganglia, are applied to a collection of masses of gray matter situated within each cerebral hemisphere. The basal nuclei play a crucial role in the control of posture and voluntary movements. They are the corpus striatum, the amygdaloid nucleus, and the claustrum. One of these areas, the corpus striatum, is completely divided by the internal capsule which is a band of nerve fibers, into the caudate nucleus and the lentiform nucleus. The lentiform nucleus is buried deep in the white matter of the cerebral hemisphere, and is related medially to the internal capsule, which separates it from the caudate nucleus and the thalamus. A vertical plate of white matter divides the lentiform nucleus into a larger, darker portion, the putamen, and an inner lighter portion, the globus pallidus. The putamen, along with the caudate nucleus, forms the main sites that receive input to the basal nuclei. Dopaminergic neuronal degeneration in the caudate nucleus, substantia nigra, globus pallidus, and putamen leads to Parkinson's disease. On the other hand, in Huntington's disease, there is a degeneration of the GABA-secreting, substance P-secreting, and acetylcholine-secreting neurons of the striatonigral-inhibiting pathway. Thus, the dopamine secreting neurons of the substantia nigra become overactive, and the nigrostriatal pathway inhibits the caudate nucleus and the putamen. This inhibition produces the abnormal movements seen in Huntington's disease. Bottom Line: In subjects with anxiety, the amygdala demonstrates increased brain activity on PET scans, as the amygdala is associated with the body's response to anxiety, fear, and emotion.

A 28-year-old female with paranoid schizophrenia presents with intermittent restlessness, agitation, and a fixed upward stare. She states that her eyes hurt and rates the pain as 5 out of 10. She feels like she cannot voluntarily make her eyes look forward. History reveals that she was diagnosed with schizophrenia at 18 years of age and has been treated with various medications since that time. She has no known significant medical problems except for mild eczema of her elbows. The most appropriate treatment is A. benztropine B. dantrolene C. haloperidol D.metoclopramide E. scopolamine ophthalmic solution

The correct answer is: A Dystonia is defined as an extrapyramidal motor disturbance consisting of slow, painful, sustained contractions of the axial or appendicular musculature. Dystonic reactions are characterized by intermittent spasmodic or sustained involuntary contractions and are caused by medications that antagonize the dopamine pathway, such as the typical anti-psychotic medications (such as haloperidol, fluphenazine, etc). Acute dystonic reactions include torticollis (neck), buccolingual crisis, trismus (jaw), oculogyric crisis (eyes), and facial grimacing. Dystonic reactions are not life-threatening unless they involve the airway (such as the larynx). Oculogyric crisis is a dystonic reaction of the ocular musculature and the treatment of choice for dystonia is benztropine or diphenhydramine. Benztropine and diphenhydramine are anti-parkinsonian agents which combat parkinsonian symptoms such as the extrapyramidal side effects seen with typical anti-psychotics. Answer B: Dantrolene sodium is a direct-acting skeletal muscle relaxant that acts by abolishing excitation-contraction coupling in muscle cells, probably by action on the ryanodine receptor. It reduces the muscle destruction and possible hyperthermia that may be occurring. It is the only specific and effective treatment for malignant hyperthermia, a rare, life-threatening disorder triggered by general anesthesia. It is also used in the treatment of neuroleptic malignant syndrome, muscle spasticity, 3,4-methylenedioxymethamphetamine (ecstasy) intoxication, serotonin syndrome, and 2,4-dinitrophenol poisoning. This patient's vital signs are within normal limits and she is denying any physical complaints other than symptoms of oculogyric crisis, and is not experiencing any fever, sweating, increased pulse or BP, hypotension, ischemia, etc. Thus, she is not undergoing a hyperthermic crisis. Answer C: Haloperidol is a typical anti-psychotic, dopamine 2 antagonist, used to treat psychotic disorders such as schizophrenia, schizoaffective disorder, or mood disorders with psychotic features. Haloperidol works by blocking dopamine 2 receptors, reducing the positive symptoms of psychosis and reducing combative, explosive, and hyperactive behaviors. By blocking dopamine 2 receptors in the striatum, it can cause motor side effects and extra-pyramidal side effects such as the dystonic reaction described in the case scenario. Thus, it cannot be used to treat dystonia because it would exacerbate the problem. Answer D: Metoclopramide is an antiemetic and gastroprokinetic agent that antagonizes the dopamine pathway, which could potentially exacerbate the dystonic reaction. Metoclopramide relieves heartburn and GERD symptoms, treats nausea and vomiting, helps with loss of appetite, facilitates gastric emptying in diabetic gastroparesis. Answer E: Scopolamine ophthalmic solution is a cycloplegic that paralyzes the ciliary muscle (loss of accommodation) but would have no affect on a dystonic reaction. Bottom Line: Dystonic reactions are treated with benztropine or diphenhydramine.

A 62-year-old male with a history of depression presents to the emergency department after fainting. Vitals reveal his blood pressure while sitting and after standing for 5 minutes to be 150/90 mmHg and 142/75 mmHg, respectively. Which of the following medications should be altered in order to reduce continued adverse effects? A. imipramine B. nortriptyline C.sertraline D. trazodone E. venlafaxine

The correct answer is: A Imipramine is a TCA shown to cause orthostatic hypotension. Anytime an elderly patient presents with new-onset syncope, you should first consider medication changes to be responsible. In this case, the patient was recently started on antidepressant therapy, most likely with imipramine. Tricyclic antidepressants are not associated with tyramine containing foods. They have largely been replaced for the treatment of depression by SSRI's and SNRI's. Today they are commonly used to treat chronic pain syndromes. Tricyclics work by inhibiting the uptake of serotonin and norepinephrine at the nerve terminal. They can cause antimuscarinic like side effects such as dry mouth, dry eyes, blurry vision and urinary retention. Answer B: Nortriptyline is also a tricyclic antidepressant but is least likely in this class of medications to cause orthostatic hypotension. It can cause cardiac symptoms and convulsions. Answer C: Sertraline is a selective serotonin re-uptake inhibitor. It blocks the re-uptake of serotonin in the presynaptic cleft. It is used as the first-line treatment for depression and requires approximately 4-8 weeks of administration before seeing its' effect. Adverse reactions include sexual dysfunction (secondary to serotonin levels) and increased risk of suicidal ideation in adolescents. They can cause serotonin syndrome which can be treated with cyproheptadine. Answer D: Trazodone is used in the treatment of depression with significant insomnia. Adverse effects include hypotension, sedation, and priapism. Answer E: SNRI's such as venlafaxine can be used to treat depression and chronic pain syndromes such a painful diabetic neuropathy. They work by inhibiting the reuptake of serotonin and norepinephrine. They can cause serotonin syndrome if combined with SSRI's or tricyclics. Bottom Line: Imipramine is a TCA known to be associated with cases of orthostatic hypotension.

A 19-year-old college student is referred to clinic after a failed suicide attempt 2 weeks ago. Following medical stabilization, she spent 4 days in inpatient treatment, where she was started on paroxetine and released. Upon entering the office, her hair is disheveled, and she is wearing a sleeveless black dress even though it is 45ºF outside. She talks with tangential, rapid, and pressured speech, though she does confirm she does not drink or use recreational drugs. She also reports that the antidepressant is working, and she feels fantastic now. Throughout the conversation, her eyes keep shifting to the left repeatedly, and upon inquiry, she confesses that a monkey has been following her for the past week and is now climbing the bookshelves in the office. She does not feel threatened by it, it does not speak to her, and she had never seen it prior to her release from the inpatient facility. Furthermore, she says that she has a special power that allows her to turn stuffed animals into living beings, and that is how the monkey came into existence. Which of the following is the most likely diagnosis? A. bipolar disorder, type I B. bipolar disorder, type II C.depression with psychotic features D. schizoaffective disorder E. schizophrenia

The correct answer is: A Patients with presumed unipolar depression that have a rapid, overwhelming upswing in mood following the start of treatment with an antidepressant should be screened for bipolar disorder since certain antidepressants, especially paroxetine, can send a bipolar type I patient into mania. Bipolar I disorder requires at least one manic episode for diagnosis, and the first manic episode usually occurs in the early 20s. Manic episodes are usually briefer than depressive episodes and, in two-thirds of patients, the transition between mania and depression occurs without an intervening period of euthymia. Lifetime suicide rates range between 10 to 15%. Diagnostic criteria for a manic episode consists of a period of abnormally elevated, expansive, or irritable mood that lasts for at least one week and includes at least three of the following: 1) distractibility, 2) decreased need for sleep, 3) flight of ideas, 4) inflated self-esteem, 5) increase in goal-directed activity, 6) pressured or rapid speech, 7) partaking excessively in high-risk, pleasurable activities. Moreover, true manic episodes can manifest with psychotic features, including hallucinations. This patient is exhibiting several classic signs of mania: euphoria, pressured speech, racing thoughts, and has psychotic features, as evidenced by her hallucination and belief that she has special abilities. Answer B: Bipolar type II patients experience hypomanic episodes, which last for at least 4 days, have no marked impairment in social or occupational functioning, and have no psychotic features. This patient's symptoms appear to have been present for at least the past week and her hallucinations rule out Bipolar II disorder. Answer C: The patient is not exhibiting any current symptoms of depression; instead, she reports that her mood is great. Answers D & E: She is unlikely to have schizophrenia or schizoaffective disorder based on this presentation because her psychosis and other symptoms appear to be associated with her mood state. In order to diagnosis her as schizophrenic or schizoaffective, she would have to exhibit psychotic features during at least 2 weeks of a normal mood state. Bottom Line: Antidepressants given without a mood stabilizer such as lithium can rapidly induce a manic state in bipolar patients being treated for depression. COMBANK Insight : Diagnostic criteria for bipolar disorders now include both changes in mood and changes in activity or energy. The DSM-IV diagnosis of bipolar I disorder, mixed episodes - requiring that the individual simultaneously meet full criteria for both mania and major depressive episode - is replaced with a new specifier "with mixed features." Particular conditions can now be diagnosed under other specified bipolar and related disorder, including categorization for individuals with a past history of a major depressive disorder whose symptoms meet all criteria for hypomania except the duration criterion is not met (i.e., the episode lasts only 2 or 3 days instead of the required 4 consecutive days or more). A second condition constituting another specified bipolar and related disorder variant is that too few symptoms of hypomania are present to meet criteria for the full bipolar II syndrome, although the duration, at least 4 consecutive days, is sufficient.

A 35-year-old male is brought to the emergency room by ambulance after having two episodes of full body tremors. He has no previous seizure history. His wife states that he was taking an antidepressant already, and just started an additional antidepressant yesterday. She also states that he seemed agitated prior to going to work and slept very poorly last night. Vital signs reveal a temperature of 40.2°C (104.5°F), blood pressure of 185/98 mmHg, heart rate of 125/min, respiratory rate of 22/min, and oxygen saturation of 96% on room air. Physical examination reveals that he is warm to the touch, diaphoretic, and exhibits myoclonus and muscle rigidity. The most likely cause of his symptoms is A. elevated serotonergic neurotransmission B. enhanced GABA receptor reactivity C. increased dopamine release D.low circulating norepinephrine levels E. stimulation of OP3 receptors

The correct answer is: A Serotonin syndrome is classically known as a rare triad of mental status changes, autonomic hyperactivity, and neuromuscular abnormalities that occurs in the setting of increased serotonergic transmission due to serotonergic agents. Serotonin syndrome occurs more commonly in the setting of a drug interaction between a serotonergic agent and a monoamine oxidase inhibitor (MAOI). It can also result from the use of multiple medications with serotonergic actions or from overdoses involving serotonergic agents. Other symptoms include: shivering, restlessness, diaphoresis, tremor, hyperreflexia, myoclonus, and ataxia. It is also associated with the use of selective serotonin reuptake inhibitors. Tremor is a more frequently found peripheral motor finding. Fever is less frequent and less profound. The onset tends to be acute and occurs soon after taking the offending medication. Other symptoms include "wet dog shakes" (intermittent whole body tremors), and ultimately may lead to death from hyperthermia. Treatment consists of cooling, benzodiazepines, and supportive care. Answer B: Several anti-anxiety medications, such as benzodiazepines, work by attaching to the GABA receptor and increase the affinity for GABA binding. GABA acts to decrease the excitability of neurons and leads to drowsiness, confusion, slurred speech, and muscle relaxation. Answer C: Increased dopamine release is an effect of cocaine and amphetamines. Many of the symptoms can be similar to serotonin syndrome, and using stimulants with a drug that increases serotonin may potentiate the effects of serotonin syndrome. However, the question stated that the patient does not use any illicit drugs, and it is increased serotonin that causes serotonin syndrome. Common symptoms include agitation, decreased sleep, tachycardia, hyperthermia, and hypertension. However, myoclonus, muscular rigidity. and intermittent whole body shakes are generally seen in serotonin syndrome. Answer D: Norepinephrine is a neurotransmitter linked to depression. Many classes of antidepressants act to increase circulating levels of norepinephrine in order to enhance mood and increase energy (SSRIs, SNRIs, TCAs, etc). Decreased levels of norepinephrine would have the opposite effect, causing depressed mood, apathy, and possibly hypotension. Answer E: The OP3 receptor (otherwise known as the mu receptor) is one of the opioid receptors. Stimulation of the OP3 receptor causes analgesia, respiratory depression, euphoria, decreased GI mobility, and miosis. Bottom Line: Serotonin syndrome tends to be seen in patients who are taking antidepressants and presents with hyperthermia, muscle rigidity, tremors and altered mental status.

An 82-year-old male with a documented workup was diagnosed with Alzheimer's dementia was started on donepezil 8 months ago. He presents to the office with his daughter who expresses frustration over continued slow decline in her father's cognitive function. She notes that he still loses his keys and credit card and at times exhibits word-finding difficulties. However, he maintains his activities of daily living without assistance and with use of behavioral techniques has been successfully maintained at home with his wife. Which of the following interventions is most appropriate at this time? A. continue donepezil B. continue donepezil and add rivastigmine C.discontinue donepezil D. discontinue donepezil and start rivastigmine E. obtain an MRI of the brain

The correct answer is: A The management goals in dementia include maximizing patient functioning, maintaining quality of life and creating a safe living environment. Treatment options include behavioral techniques and pharmacotherapy. It is important to remember that current drug therapy has not been shown to alter disease progression. Instead, these medications can help to maintain functioning and delay nursing home placement. Common pharmacotherapy options include the acetylcholinesterase inhibitors (donepezil, rivastigmine and galantamine) and NMDA receptor antagonists (memantine). The nucleus basalis of Meynert and diagonal band of Broca provide the main cholinergic input to the hippocampus, amygdala, and neocortex, which are lost in patients with Alzheimer's dementia. Donepezil inhibits centrally-acting the centrally-active enzyme that breaks down acetylcholine and thus makes more acetylcholine available. Answer B: There are no indications for treatment with two agents from the same class (both are acetylcholinesterase inhibitors). Addition of memantine, an NMDA receptor antagonist, could be considered in the treatment of moderate to severe Alzheimer's dementia as it works to normalize glutamate levels, a neurotransmitter though to cause neurodegeneration when present in excessive quantities. Answer C: There is no reason to stop the patient's current medication as he is tolerating it without side effects. Common side effects of donepezil include nausea, vomiting, diarrhea and loss of appetite. Again, the role of pharmacotherapy is not to prevent disease progression but rather to maintain functioning and delay nursing home placement. Answer D: Rivastigmine, like donepezil, is an acetylcholinesterase inhibitor that prevents breakdown of acetylcholine thereby increasing its concentration in the hippocampus and neocortex. There is no indication for switching from one acetylcholinesterase inhibitor to another as this patient is tolerating donepezil. Answer E: The question stem notes that the patient has a documented diagnosis of Alzheimer's dementia, so it can be assumed that the appropriate medical workup has been performed (including lab studies to rule out reversible medical causes of dementia and brain MRI to rule out structural lesions). Since there are no acute complaints or changes in mentation or consciousness noted, a repeat brain MRI would not be indicated at this time. Bottom Line: Dementia is a progressive and irreversible illness associated with decreased acetylcholine due to loss of cholinergic neurons in the locus ceruleus. Common pharmacotherapy options include the acetylcholinesterase inhibitors and NMDA receptor antagonists. It is important to educate patients and families about the limitations of drug therapy - they do not alter disease progression but rather delay nursing home placement.

A 14-year-old boy is brought to the office by his mother. She is concerned that he has "gotten in with the wrong crowd." He has been skipping school, and he was arrested for assaulting an elderly woman last week. He has no significant medical history. He was treated briefly for depression at the age of 8 after his mother left his father because he was violent toward her, the patient, and his sister. When interviewed alone, he admits to smoking a pack of cigarettes a day and using marijuana regularly. He says his behavior "is not a big deal. It's just the way things work that the strong prey on the weak." This patient is at increased risk for developing which of the following as an adult? A. antisocial personality disorder B. borderline personality disorder C. conduct disorder D.generalized anxiety disorder E. histrionic personality disorder

The correct answer is: A The patient currently exhibits signs of conduct disorder, which is only recognized in children. In adults, the same behavior is classified as antisocial personality disorder, which requires that features of conduct disorder be present in childhood. It is more common in males than females, and more common in people who grew up in abusive families. Answer B: Borderline personality disorder, which is characterized by a pattern of unstable relationships, is most common in women from abusive families. It has no association with conduct disorder. Answer C: This patient currently has conduct disorder, which puts him at increased risk for developing antisocial personality disorder. Answer D: Anxiety disorders are also more common in abused children, but this patient is not exhibiting signs of anxiety. Answer E: Histrionic personality disorder is characterized by overly dramatic behavior. Bottom Line: Children with conduct disorder often go on to develop antisocial personality disorder.

A 2-year-old boy is brought to the emergency department by his mother because she could not calm him down after she found him banging a toy against the wall. He screamed and hit her when she tried to stop him. He has no history of behavioral disturbance and no recent medical illness. His 8-year-old brother is being treated for attention deficit hyperactivity disorder (ADHD) and his 6-year-old sister has been experiencing severe separation anxiety. His father is recovering from influenza pneumonia. On physical examination, the temperature is 39.2°C (102.6°F), pulse is 160/min, respirations are 15/min, and the blood pressure is 140/95 mm Hg. He appears fearful. His pupils are 5 mm and reactive bilaterally. Cardiac auscultation reveals rapid heart rate with a normal rhythm. Which of the following is the most likely cause of these findings? A. amphetamine ingestion B.autism spectrum disorder C. cocaine ingestion D. fluoxetine ingestion E. influenza

The correct answer is: A The patient has likely taken his brother's ADHD medication and is presenting with classic symptoms of amphetamine toxicity. Therapeutic doses in older children are enough to cause toxicity in a toddler. Physical findings in amphetamine toxicity include the following: tachycardia or bradycardia, hyperthermia, pupillary dilatation, elevated or lowered blood pressure, perspiration or chills, nausea or vomiting, evidence of weight loss, psychomotor agitation or retardation, muscular weakness, respiratory depression, chest pain, or cardiac arrhythmias, disorientation, memory loss, seizures, dyskinesias, dystonias, or coma. Answer B: Children with autism spectrum disorders present with developmental abnormalities, such as impaired communication skills and abnormal social interactions. This child's symptoms are not consistent with this diagnosis. Answer C: While cocaine intoxication produces a similar syndrome, the history of a sibling taking medication to treat ADHD, in this case, points to amphetamines as the cause of the patient's problems. Answer D: Patient's with SSRI toxicity may present with serotonin syndrome, which includes ataxia, diaphoresis, hyperreflexia, and diarrhea. Furthermore, the history provided in the stem does not point towards anyone in the family taking a selective serotonin inhibitor fluoxetine, but does suggest that his brother is likely taking an amphetamine. Answer E: While this patient's fever and dad with influenza may appear to point towards an acute infection, behavioral disturbances and sympathomimetic symptoms would not be seen. Bottom Line: Amphetamine toxicity can occur with low doses in young children taking ADHD medication and can present with sympathomimetic crisis.

A 34-year-old male undergoes a sleep study after experiencing multiple episodes of daytime somnolence that have been interfering with his work as a university campus security officer. The study reveals decreased REM sleep during the night. His past medical history includes eczema and seasonal allergies. He has never had any surgeries. He denies any past psychiatric or neurologic history as well. He eats a well-balanced diet and was exercising regularly at the gym until several months ago, when he started experiencing the multiple episodes of daytime somnolence. The most effective means of increasing REM sleep involves medical therapy that increases the circulating levels of A. acetylcholine B. dopamine C. histamine D.norepinephrine E. serotonin

The correct answer is: A There are various types of eye movements during sleep, mostly slow rolling eye movements occurring early in sleep and disappearing as sleep progressed, and rapid eye movements associated with irregular breathing and increased heart rate. The sleep phase associated with the slow rolling rhythmic eye movements is labeled as non-rapid eye movement sleep (NREM) and the sleep phase associated with the fast, erratic eye movements is labeled rapid eye movement sleep (REM). REM sleep is also associated with dreaming. Increased levels of circulating acetylcholine (ACh) have been associated with the prolongation of both REM sleep and total sleep time, whereas, the other neurotransmitters listed above either decrease or have no effect on REM sleep. Medications that act as ACh agonists increase REM sleep, and ACh antagonists decrease REM sleep. Ach agonists include the acetylcholinesterase inhibitors like rivastigmine, neostigmine, donepezil, physostigmine, etc. Ach antagonists include anti-muscarinic agents like atropine, ipratropium, scopolamine, and diphenhydramine. Answer B: Dopamine has no effect on REM sleep. However, it does cause a decrease in total sleep time. Answer C: Histamine is not associated with sleep changes. Answer D: Norepinephrine decreases both total sleep time and REM sleep and would have negative effects on the patient in the above case. Norepinephrine and serotonin levels do not increase with the cortical arousal and desynchronization that accompanies REM sleep. In fact, the exact opposite occurs. Both neurotransmitters are at their lowest levels during REM. Furthermore, norepinephrine and serotonin agonists (like the SSRIs and the SNRIs) suppress REM activity, and antagonists (like ondansetron and granisetron) increase REM activity. Thus, it appears that both of these neurotransmitters play a complimentary role with acetylcholine, in that they act to control and suppress REM activity, while acetylcholine acts to initiate and maintain REM sleep. Answer E: Increased levels of serotonin result in prolonged total sleep time, along with delta-wave sleep. Norepinephrine and serotonin levels do not increase with the cortical arousal and desynchronization that accompanies REM sleep. In fact, the exact opposite occurs. Both neurotransmitters are at their lowest levels during REM. Furthermore, norepinephrine and serotonin agonists (like the SSRIs and the SNRIs) suppress REM activity, and antagonists (ondansetron and granisetron) increase REM activity. Thus, it appears that both of these neurotransmitters play a complimentary role with acetylcholine, in that they act to control and suppress REM activity, while acetylcholine acts to initiate and maintain REM sleep. Bottom Line: Elevated levels of acetylcholine result in prolonged REM and total sleep times.

A 46-year-old female is brought to the emergency department by her husband due to altered mental status after ingesting some pills. History reveals that her husband found an empty pill bottle in their bathroom next to the patient who was confused. He reports that she has a history of major depression since her early twenties and has been on various medications throughout the course of her life. Her past medical history is significant for hyperlipidemia, irritable bowel syndrome, and mild hypertension. Vitals reveal the following: Temperature: 39.1ºC (102.4ºF) Blood pressure: 110/70 mmHg Heart rate: 105/min Respiratory rate: 22/min Physical examination reveals dilated pupils and facial flushing. An electrocardiogram is obtained and displays a widened QRS and a prolonged QT interval. Question 1 of 2 in this set Which of the following medications should be promptly discontinued? A. amitriptyline B. cyclobenzaprine C. quetiapine D. risperidone E. sertraline

The correct answer is: A This patient has a history of major depressive disorder which is commonly treated with either a serotonin reuptake inhibitor (SSRI), a serotonin-norepinephrine reuptake inhibitor (SNRI), or a tricyclic antidepressant (TCA). Amitriptyline belongs to the TCA class and is indicated for depression and also helps for insomnia, fibromyalgia, headaches, and neuropathic pain. It also increases norepinephrine and serotonin neurotransmission. While the most common side effects of TCAs such as amitriptyline are blurred vision, nausea, diarrhea, increased appetite, heartburn, fatigue, weakness, urinary retention, dry mouth, and constipation; they are also known to cause delirium, hyperthermia, and tachycardia, along with cardiac arrhythmias. Overdose can occur with ingestion of as little as 10 times the normal daily dose, and the toxicity is usually due to QTc prolongation and widening of the QRS complexes, leading to ventricular arrhythmias that can possibly be fatal. Answer B: Cyclobenzaprine is a muscle relaxant whose main side effect is drowsiness. Answers C & D: Risperidone and quetiapine are both atypical anti-psychotic medications used in the treatment of schizophrenia, psychotic disorders, bipolar disorder, etc. This class of medications has been known to cause lethargy along with confusion, however cardiac arrhythmias are uncommon. They may cause the rare development of neuroleptic malignant syndrome, seizures, and hyperglycemia. And in elderly patients with dementia-related psychosis, there is an increased risk of cerebrovascular events and death with these medications. Answer E: Sertraline is an SSRI that is also used in the treatment of depression, however SSRIs are significantly less toxic than TCAs and overdose is rare. Serotonin syndrome occurs more commonly in the setting of a drug interaction between a serotonergic agent and a monoamine oxidase inhibitor (MAOI). It can also result from the use of multiple medications with serotonergic actions or from overdoses involving serotonergic agents. Serotonin syndrome is classically known as a triad of mental status changes, autonomic hyperactivity, and neuromuscular abnormalities that occurs in the setting of serotonergic agents. Other symptoms include: shivering, restlessness, diaphoresis, tremor, hyperreflexia, myoclonus, and ataxia. It is also associated with the use of selective serotonin reuptake inhibitors. Tremor is a more frequently found peripheral motor finding. Fever is less frequent and less profound. Bottom Line: An overdose of TCAs such as amitriptyline may cause prolongation of the QTc interval and widened QRS complexes, leading to possibly fatal arrhythmias. Cardiac conduction abnormalities and toxicity occurs in TCA overdoses because these medications inhibit the fast sodium channels in the conduction system which leads to an increase in the duration of depolarization and prolongs refractory periods. Sodium bicarbonate is the mainstay anti-arrhythmic therapy in TCA intoxication and is warranted in all patients with TCA poisoning who develop widening of the QRS complex or a ventricular arrhythmia. Patients may require alkalinization of their serum, along with pressors and/or ventilatory support to maintain survival. TCAs have a low threshold for toxicity. Due to the toxicity and lethality of TCA overdoses, it is recommended to either prescribe limited quantities of the medication at a time, or to use non-TCA medications, especially in acutely suicidal patients.

A 50-year-old male presents with multiple recurrent episodes of chest pain, shortness of breath, and sweating. He has several episodes per week with no apparent trigger, all lasting about half an hour in duration. He does admit that there has been an increased amount of stress at work recently and that he has been working fourteen-hour days six days per week. He had similar symptoms in the past and has had an intensive medical work-up including a negative cardiac stress test. During the encounter he becomes diaphoretic and complains of palpitations. Question 1 of 2 in this set Which of the following would be best for rapid relief of symptomology? A. benzodiazepine B. beta blocker C.psychodynamic therapy D. selective serotonin reuptake inhibitor E. tricyclic anti-depressant

The correct answer is: A This patient is suffering from panic disorder, which is an anxiety disorder causing recurrent panic attacks in addition to fear of when the next attack may occur and behavioral changes lasting at least one month. Panic attack is the sudden onset of four or more of the following symptoms: palpitations or increased heart rate, sweating, trembling, shortness of breath, chest pain, nausea, feeling dizzy or faint, depersonalization, fear of losing control, fear of dying, paresthesias, and chills or hot flashes. Benzodiazepines have been shown to reduce panic severity. There are a few different choices in this drug class and all have differing durations of action, but all have a relatively quick onset of action. Panic disorder is separate from generalized anxiety disorder (GAD), which is at least six months of excessive worry which inhibits a person's daily academic, social, personal, or occupational functioning. Management of panic disorder involves the use of cognitive behavioral therapy in addition to a selective serotonin reuptake inhibitor as well as a benzodiazepine for use during panic attacks. Answer B: Beta blockers are commonly used for the treatment of congestive heart failure and coronary artery disease as well as hypertension. Some beta blockers such as propanolol have been used for performance anxiety or social phobia, but not for use in a panic attack or in generalized anxiety disorder. Answer C: Psychodynamic therapy is similar to psychoanalysis and focuses on the unconscious mind. One randomized trial showed that it may be effective, but it is not appropriate for acute therapy or rapid relief. Cognitive behavioral therapy is the most appropriate non-pharmacologic management for long-term therapy. Answer D: Selective serotonin reuptake inhibitors have been proven to treat generalized anxiety disorder, depression, and obsessive compulsive disorder, but are not for acute treatment of panic attacks. These medications are first line therapy for panic disorder, however, a benzodiazepine would be more appropriate in the use of an acute panic attack. Answer E: Tricyclic antidepressants do work for panic attacks, but they are not the first line therapy and take about four weeks for maximal response. Bottom Line: Benzodiazepines are treatment of choice for acute panic attacks. SSRI's are most appropriate for long-term therapy. Depression is commonly associated with panic disorder. Up to one-third of panic disorder cases are associated with depression. Patients begin to have a fear of the panic attacks, which can lead to phobias (particularly agoraphobia or fear of being unable to escape a situation or other specific phobias) and/or depression.

A 25-year-old female presents with chronic abdominal pain of 5 years' duration. History reveals non-specific constipation, bloating, nausea, menstrual cramps with heavy flow, back pain and headaches with blurred vision. Patient is requesting a "full body" MRI to help "figure out" her pain. Social history reveals that she is divorced and is having difficulty maintaining steady employment due to stress. Past medical history is pertinent for an appendectomy, a cholecystectomy, and an EGD. Physical examination reveals a soft, non-tender abdomen with normal bowel sounds, normal cranial nerve exam, normal lumbar range of motion with full strength in arms and legs and normal deep tendon reflexes with intact sensation. Routine laboratory studies reveal a normal complete blood count, erythrocyte sedimentation rate, serum amylase level, and comprehensive metabolic panel. All other previous studies have been negative. The most appropriate management is A. function-based office visits B. referral for exploratory laparotomy C.referral to multiple specialists D. symptom-based office visits E. trial of nortriptyline

The correct answer is: A This patient most likely has Somatic Symptom Disorder. The DSM-5 diagnostic criteria include: A. One or more somatic symptoms that are distressing or result in significant disruption in daily activity. B. Excessive thoughts, feelings, or behaviors related to the somatic symptoms or associated health concerns as manifested by at least one of the following: disproportionate and persistent thoughts about the seriousness of one's symptoms; persistently high level of anxiety about health or symptoms. 3. Excessive time and energy devote to these symptoms or health concerns. C. Although any one somatic symptom may not be continuously present, the state of being symptomatic is persistent (typically more than 6 months). Treatment includes scheduling regular visits, acknowledging symptoms, communicating with other clinicians, assessing and treating diagnosable medical and psychiatric disorders, limiting tests and referrals, reassuring the patient that grave medical diseases have been ruled out, and making functional improvement the goal of treatment. Answer B: Surgery without clinical indication of organic pathology should be avoided in somatoform disorder. Patients have multiple procedures with negative results and are at risk for unintended sequela and complications. Management should focus on function and not symptoms with reassurance. Answer C: Without clinical indication for referral, the patient with somatoform disorder is a risk for multiple, repetitive, unnecessary testing which reinforces that focus on symptoms and does not address the psychological component. Answer D: Patients with somatoform disorder need collaborative care with reassurance and focus on function, not symptoms. Frequent visits, without discussion of visit limitations, can result in increased urgent visits, phone calls, reinforcing the sick role and unnecessary referrals and testing. Answer E: Patients with somatization often have comorbid depressive and anxiety disorders. Somatization frequently resolves when these psychiatric syndromes are appropriately treated. Pharmacotherapy and psychotherapy are each beneficial, and there is no evidence to indicate one is better than the other. The choice of treatment, including the combination of pharmacotherapy and psychotherapy, depends upon availability and preferences. Fluoxetine or another SSRI is the preferred first line treatment, and tricyclics such as nortriptyline are considered second line. Bottom Line: Somatoform disorders are characterized by multiple clinically significant, but unexplained, physical symptoms that cause significant emotional distress and lack primary or secondary gain. After appropriate rule out of organic pathology, treatment includes management of psychiatric comorbidities reassurance, avoidance of unnecessary test and referrals. COMBANK Insight : In DSM-5, somatoform disorders are now referred to as somatic symptom and related disorders. The DSM-5 classification reduces the number of these disorders and subcategories to avoid problematic overlap. Diagnoses of somatization disorder, hypochondriasis, pain disorder, and undifferentiated somatoform disorder have been removed. Individuals previously diagnosed with somatization disorder will usually have symptoms that meet DSM-5 criteria for somatic symptom disorder, but only if they have the maladaptive thoughts, feelings, and behaviors that define the disorder, in addition to their somatic symptoms. Because the distinction between somatization disorder and undifferentiated somatoform disorder was arbitrary, they are merged in DSM-5 under somatic symptom disorder. Individuals previously diagnosed with hypochondriasis who have high health anxiety but no somatic symptoms would receive a DSM-5 diagnosis of illness anxiety disorder (unless their health anxiety was better explained by a primary anxiety disorder, such as generalized anxiety disorder). Some individuals with chronic pain would be appropriately diagnosed as having somatic symptom disorder, with predominant pain. For others psychological factors affecting other medical conditions or an adjustment disorder may be more appropriate.

A 19-year-old female presents after imposing self-inflicted wounds on herself. History reveals that she left home for college two months ago. This is the first time she has lived away from her parents and she comes from a well-functioning, good family home. She has no significant past medical or psychiatric history. She reports constant worrying about her grades, and she has intermittent nightmares about failing her exams. She subsequently has trouble concentrating on homework, has some trouble falling asleep, and has a decreased appetite. When she received a grade of 82% on her first Physics exam, she locked herself in her room for two days and made superficial cuts on her wrists. She denied that the cuts were an attempt to kill herself, but said she wanted to relieve her worry. She denies any physical complaints and vital are within normal limits. The most likely diagnosis is A. acute stress disorder B. adjustment disorder C.borderline personality disorder D. generalized anxiety disorder E. major depressive disorder

The correct answer is: B Adjustment disorder is the development of emotional or behavioral symptoms in response to an identifiable stressor(s) occurring within 3 months of the onset of the stressor(s). These symptoms or behaviors are clinically significant as evidenced by either of the following: (1) marked distress that is in excess of what would be expected from exposure to the stressor; (2) significant impairment in social or occupational (academic) functioning. The stress-related disturbance does not meet criteria for another Axis I diagnosis, and is not merely an exacerbation of a personality disorder. Once the stressor has terminated, the symptoms do not persist for more than an additional six months. In this case scenario, the patient has experienced an identifiable stressor of being away from home for the first time and she has become stressed out from her academics, affecting her daily functioning. The behaviors of locking herself up and cutting herself demonstrate that she has poor coping skills when dealing with her stressors. Answer A: Acute stress disorder involves exposure to a traumatic event and a subsequent response of intense fear, helplessness, or horror. The traumatic event involves a threat of death to self or others, or a threat to the physical integrity of self or others. Symptoms occur within 4 weeks of the traumatic event and persist for at least 2 days and up to less than 4 weeks. Either while experiencing or after experiencing the traumatic event, the individual has three or more of the following dissociative symptoms: a sense of numbing or detachment, a reduction in awareness of surroundings, derealization, depersonalization, dissociative amnesia. The traumatic event is re-experienced in the form of dreams or images, marked anxiety or hypervigilance exists, and marked avoidance of stimuli occurs. The impairments cause significant distress in social, occupational, or other important areas of functioning. In this case scenario, the patient was not exposed to a traumatic event that threatened the life or physical integrity of self or others. Answer C: Borderline personality disorder involves a pervasive pattern of unstable interpersonal relationships, self-image, affects, and marked impulsivity beginning by early adulthood, and present in a variety of contexts, as indicated by at least five or more of the following: (1) frantic efforts to avoid real or imagined abandonment; (2) a pattern of unstable interpersonal relationships characterized by attenuating between the extremes of idealization and devaluation; (3) self-identity disturbance; (4) impulsivity in at least two areas that are potentially self-damaging (such as spending, sexual behaviors, reckless driving, binge eating, etc); (5) recurrent suicidal behaviors or gestures; (6) affective instability due to a marked reactivity of mood; (7) chronic feelings of emptiness; (8) inappropriate, intense anger and difficulty with controlling the anger; (9) transient, stress-related paranoid ideation. Borderline personality disorder is seen as occurring over a chronic course throughout the lifetime. Although the patient is only 19 years old, if she had borderline personality disorder, she would have exhibited some symptoms in her earlier teenage years. This case scenario describes a patient with no past psychiatric history though. Answer D: Generalized anxiety disorder manifests with excessive anxiety and worry, occurring more days than not for at least six months, about multiple issues or activities. Individuals find it difficult to control their anxiety. The anxiety and worry are associated with three or more of the following six symptoms: restlessness or feeling on edge, being easily fatigued, difficulty concentrating, irritability, muscle tension, disturbance with sleep. The anxiety or worry causes significant distress in social, occupational, or other areas of functioning. In this case scenario, the duration of symptoms described is less than six months, and she does not exhibit excessive worry about multiple aspects of her life, thus making this answer incorrect. Answer E: Major depressive disorder (MDD) is diagnosed when a patient has at least one depressive episode which encompasses the presence of at least five or more criteria nearly every day within the same two-week period, and represents a change from previous functioning. At least one of the symptoms is depressed mood or anhedonia (loss of interest). The mnemonic that can be useful for the criteria is SIGECAPS, and the following is a description of each criteria: S = decreased sleep or increased sleep; I = decreased interest in most activities; G = increased feelings of worthlessness or guilt; E = decreased energy or fatigue; C = decreased concentration on tasks or activities or in decision-making; A = decreased or increased appetite and weight loss or weight gain may be present; P = psychomotor agitation or retardation; S = recurrent thoughts of death or suicidal ideation. The symptoms cause clinically significant impairments in social, occupational, or academic functioning. In this case scenario, the patient is engaging in her school work and studying for exams, and she does not exhibit symptoms of depressed mood and does not appear to have anhedonia. Bottom Line: Adjustment disorder is characterized by symptoms that are in excess of what would be expected from an identifiable stressor. COMBANK Insight : It is crucial to know the duration of symptoms in order to diagnose psychiatric disorders. Many psychiatric disorders overlap in symptomatology, but it is the duration of symptoms that will lead one to the correct answer.

An 18-year-old male presents to the emergency department with confusion and agitation. He is aggressive toward staff throughout the evaluation. Vital signs reveal blood pressure of 186/93 mmHg and pulse of 119/min. Mental status examination reveals psychomotor agitation, labile affect and flight of ideas. He appears internally preoccupied and expresses fear that the nursing staff is trying to kill him. Physical examination reveals mydriasis. The most likely diagnosis is A. alcohol intoxication B. amphetamine intoxication C. cocaine withdrawal D.opioid withdrawal E. phencyclidine intoxication

The correct answer is: B Amphetamine intoxication is characterized by autonomic hyperactivity with hypertension, tachycardia and diaphoresis, pupillary dilation (mydriasis) and psychomotor agitation that can progress to paranoid psychosis with delirium. A urine drug screen can detect amphetamines for up to approximately 48 hours after last use. Treatment consists of benzodiazepines for agitation and phentolamine for significant hypertension - beta-blockers should be avoided as they lead to unopposed alpha-adrenergic stimulation, which can precipitate hypertensive crisis. Answer A: While acute alcohol intoxication can cause behavioral disinhibition with agitated or aggressive behavior, pupillary changes are not associated. Inebriation with very high blood alcohol levels is marked by unpredictable behavior, euphoria, labile mood, poor judgment and ataxia but not by frank psychosis as described in the question stem nor the degree of psychomotor agitation described - remember that alcohol is a central nervous system depressant. Answer C: Since amphetamines and cocaine are both psychostimulants they share similar withdrawal syndromes with characteristic post use "crash" with dysphoria and hypersomnolence. Intoxication with amphetamines and cocaine produce the same symptoms of agitation, autonomic hyperactivity, pupillary dilation (mydriasis) and psychosis and so can be differentiated via urine drug screen. Answer D: Opioid withdrawal produces pupillary dilation (mydriasis), nausea, stomach cramps, diarrhea, myalgia, yawing and rhinorrhea. This type of withdrawal can be very uncomfortable but is rarely life threatening, unlike alcohol and sedative-hypnotic withdrawal which can cause seizure. Answer E: Phencyclidine intoxication produces pupillary constriction (miosis), nystagmus and tachycardia. It is characterized by belligerence and unpredictable violent behavior and is often accompanied by psychosis. Treatment includes benzodiazepines for sedation and haloperidol for severe psychosis. Bottom Line: Recall that only alcohol and sedative-hypnotic (benzodiazepine and barbiturate) withdrawal are life-threatening due to risk of seizure. Pupillary constriction (miosis) occurs with opiate and phencyclidine intoxication while pupillary dilation (mydriasis) occurs with cocaine or amphetamine intoxication, LSD intoxication and opiate withdrawal. Nystagmus can occur with phencyclidine intoxication, opiate withdrawal, and in Wernicke's encephalopathy due to chronic alcohol consumption.

A 55-year-old male presents with complaints of forgetfulness and depressed mood related to his recent divorce. He reveals that he is under a lot of stress from work due to repeated tardiness and complaints about his physical appearance. History reveals that he has 2-3 beers every night and "a couple" of shots of liquor before bed to help him sleep. The most appropriate recommendation at this time is A. acamprosate trial B. brief intervention with repeat office visits C. disulfiram trial D.fluoxetine trial E. naltrexone trial

The correct answer is: B Brief intervention is a short-term counseling strategy based on motivational enhancement therapy that concentrates on changing patient behavior and increasing patient compliance with therapy. It has been shown to be effective for helping socially stable problem drinkers to reduce or stop drinking and for motivating alcohol-dependent patients to enter long-term alcohol treatment. Generally conducted in four or fewer sessions, lasting from a few minutes to an hour, this intervention is designed for health professionals who are not specialists in addiction. Overall, thirty to sixty percent of alcoholics maintain at least one year of abstinence with psychosocial therapies alone. Answer A: Acamprosate is considered a first line medication after failure of psychological interventions alone. The principal anti-drinking neurochemical effect has been attributed to the modulation of glutamate neurotransmission at metabotropic-5 glutamate receptors. Acamprosate appears to be safe and well tolerated and may almost double the abstinence rate among recovering alcoholics. Answer C: Disulfiram is an aversive agent that does not directly influence motivation to drink, but discourages drinking by causing an unpleasant physiologic reaction when alcohol is consumed. Disulfiram inhibits aldehyde dehydrogenase and prevents the metabolism of alcohol's primary metabolite, acetaldehyde. Drinking alcohol while taking disulfiram results in the accumulation of acetaldehyde in the blood, which causes unpleasant effects such as sweating, headache, dyspnea, lowered blood pressure, flushing, palpitations, nausea, and vomiting. This anti-craving medication is considered second line and should be monitored closely due to side effects. Answer D: Fluoxetine, selective serotonin reuptake inhibitor, has been found to be effective in decreasing depressive symptoms and the level of alcohol consumption in depressed alcoholics and can be used in conjunction with psychological interventions Answer E: Naltrexone exerts its principal pharmacological effects through blockade of the mu-opioid receptor. Endogenous opioids are involved in modulating the expression of alcohol's reinforcing effects and is considered a first line medication after failure of psychological interventions alone. Several studies have shown that naltrexone reduces alcohol consumption in male and female alcoholics and is effective, when combined with psychosocial treatment, in reducing relapse rates. Bottom Line: Brief intervention is a short-term counseling strategy based on motivational enhancement therapy that concentrates on changing patient behavior and increasing patient compliance with therapy. Thirty to sixty percent of alcoholics maintain at least one year of abstinence with psychosocial therapies alone.

An 82-year-old male presents with visual hallucinations of animals and small children which he describes as vivid and colorful. History reveals blindness due to bilateral retinal detachments and sensorineural hearing loss. Mental status examination reveals an appropriately groomed elderly male who is calm and cooperative with full affect and linear thought processes. He is alert and attentive throughout the interview. The most likely diagnosis is A. brief psychotic disorder B. Charles Bonnet syndrome C. delirium D.Lewy body dementia E. schizophrenia

The correct answer is: B Charles Bonnet Syndrome is a condition seen in visually impaired individuals characterized by the presence of visual deprivation hallucinations that are complex in nature. These hallucinations are secondary to sensory deprivation leading to false perceptions rather than psychosis. No other features of a psychotic or mood disorder are present and patients have insight, realizing their visions are not real. Answer A: In order for a diagnosis of brief psychotic disorder to be made the patient must exhibit delusions, hallucinations, disorganized speech or grossly disorganized behavior for between one day and one month's duration. While the described patient is experiencing hallucinations, the time course of the symptoms is not mentioned. Furthermore, the patient has a general medical condition (blindness) that can cause his perceptual disturbance and so a diagnosis of brief psychotic disorder cannot be made. Answer C: Delirium is a cognitive disorder characterized by disturbance in consciousness, inattention, and changes in cognition such as memory impairment and disorientation. It develops acutely over a short period of time and the disturbances in consciousness and cognition fluctuate. Most importantly, the disturbance is caused by the physiological consequences of a general medical condition as evidenced by history, physical exam and laboratory data. Delirium is a common cause of acute confusion and psychosis in elderly hospitalized patients. However, the patient described has no acute change in cognition or consciousness that would fulfill criteria for a delirium diagnosis. Answer D: When an elderly patient presents with vivid visual hallucinations in the absence of other signs and symptoms of primary psychotic disorder (such as delusions, grossly disorganized speech and behavior or negative symptoms), dementia with Lewy bodies may be considered. It is a cognitive disorder characterized by dementia, Parkinsonism, visual hallucinations which are often vivid visions of animals or people, sudden changes in cognition, attentiveness and alertness and hypersensitivity to treatment with antipsychotic agents (which produce paradoxical effects or severe extrapyramidal symptoms). However, the patient described in the question stem displays none of these supportive features and no cognitive disorder is described. Bottom Line: The onset of complex visual hallucinations in an otherwise healthy blind patient with no psychiatric history and no other evidence of psychotic or mood disorder is suggestive of Charles Bonnet Syndrome. Patients with this syndrome realize that the hallucinations are not real (and so have insight). The differential diagnosis for visual hallucinations includes primary psychotic disorder, substance-induced psychotic disorder, delirium and dementia.

A 5-year-old previously healthy boy is brought to the pediatrician by his parents because of incontinence of stool over the past several months. At first they thought he was "going through a phase," but after the problem persisted they began to think something was wrong. Their son recently began kindergarten and had been doing well until around the time his bowel control problem started. He began to withdraw from other children. He also recently started flapping his hands uncontrollably. The parents can think of no particularly stressful events in the child's life and they note that his development had always been normal. His vaccinations are up to date. On examination, the boy appears well and appropriately developed for his age. He refuses to make eye contact, however, and it is difficult to get him to cooperate with the examination. The only physical abnormality is that his gait is clumsy when running. He speaks to his parents mostly in two-word sentences and uses articles incorrectly, saying "me" when he means "you." Which of the following is the most likely diagnosis? A. autism spectrum disorder B. childhood disintegrative disorder C.encopresis D. Rett's disorder E. schizophrenia

The correct answer is: B Childhood disintegrative disorder is characterized by normal development in early childhood and then loss of previously acquired skills. Onset is between the ages of 2 and 10, and boys are more commonly affected than girls. Loss of skills occurs in at least two of the following areas: language, social skills, bowel or bladder control, play, and motor skills. Impairment also exists in at least two of: social interaction, use of language, and presence of stereotyped or repetitive behavior. Answer A: Autism Spectrum Disorder involves persistent deficits in social communication and interaction across multiple contexts as well as restricted, repetitive patterns of behavior, interests, or activities. Symptoms must be present in the early developmental period but may not become fully manifest until social demands exceed limited capacities (or may be masked by learned strategies in later life). This child in this stem had social skills but then continued to lose them, rather than not progress as expected in normal development. Answer C: Encopresis is the loss of bowel control after the age of 4 and is not caused by a general medical condition (except constipation). This patient has a loss of language and social skills, making childhood disintegrative disorder the correct diagnosis. Answer D: Rett's disorder is characterized by normal development until age 5 months and disintegration in previously acquired milestones starting between age 5 months and 48 months. Rett's disorder is seen only in girls. Answer E: Schizophrenia with childhood onset usually develops after a period of normal, or near normal, development. A prodromal state has been described in which social impairment and atypical interests and beliefs occur, which could be confused with the social deficits seen in autism spectrum disorder. Hallucinations and delusions are features of this disorder. Bottom Line: Childhood disintegrative disorder is characterized by normal development until age 2 and then loss of previously acquired skills. Onset is between the ages of 2 and 10, and boys are more commonly affected than girls. COMBANK Insight : Autistic disorder, Asperger's disorder, and pervasive developmental disorder have been consolidated into the autism spectrum disorder. Symptoms of these disorders represent a single continuum of mild to severe impairments in the two domains of social communication and restrictive repetitive behaviors/interests rather than being distinct disorders. This change is designed to improve the sensitivity and specificity of the criteria the diagnosis of autism spectrum disorder and to identify more focused treatment targets for the specific impairments identified.

A 20-year-old female with a history of anxiety is brought to the emergency department by her boyfriend because she suddenly lost her vision. Her vision loss came on suddenly after an argument with her mother about whether she would return to school in the fall. When interviewed, the patient acknowledges that she is concerned about her problem but calmly says, "I'm sure everything will be okay." She has no headache, dizziness, weakness, or other neurological symptoms apart from the visual loss. Her medications include lorazepam as needed for anxiety, depot medroxyprogesterone for birth control, and a daily multivitamin. On examination, her pupils are equally round and reactive to light. Extraocular motions are intact bilaterally. Funduscopic examination is normal. Pulmonary, cardiac, gastrointestinal, and musculoskeletal examinations are normal. Which of the following is the most likely diagnosis? A. factitious disorder B. functional neurological symptom disorder C.idiopathic intracranial hypertension D. malingering E. Munchausen syndrome

The correct answer is: B Conversion disorder, now known as functional neurological symptom disorder in DSM-5, is characterized by symptoms suggesting a neurological disorder that are preceded by conflict or stress and cannot be explained by a general medical condition or the effects of a substance. Patients with conversion disorder have at least one neurological symptom (sensory or motor) and are often unusually calm or unconcerned when describing their symptoms, which include severe manifestations such as paralysis, or as in this patient's case, blindness. Symptoms usually resolve within a month, and 25% of patients will go on to have recurrent episodes, especially in times of stress. Conversion disorder is treated by gently educating the patient about the psychological nature of their problem. If necessary, adjunctive medications for mood or anxiety may also be helpful. This patient's symptoms began after an argument with her mother, which is likely the stressor in this scenario. Other common stressors are divorce or sudden unemployment. The symptoms are not feigned and are not attributable to malingering or a factitious disorder, and there is no mention of any secondary gain, which would be necessary for malingering or Munchausen syndrome. If she was malingering, the stem imply that she feigned these symptoms to get out of going back to school. If she had Munchausen syndrome, the stem might imply that she was feigning these symptoms to be admitted to the hospital or have others take care of her. Additionally, this patient's history and examination do not reveal any organic cause for her symptoms; however, a medical cause must still be fully ruled out prior to receiving the diagnosis. Answers A & D & E: The symptoms are not feigned and are not attributable to malingering or a factitious disorder. There is no mention of any secondary gain, which would be necessary for malingering or Munchausen syndrome. If she was malingering, the stem imply that she feigned these symptoms to get out of going back to school. If she had Munchausen syndrome, the stem might imply that she was feigning these symptoms to be admitted to the hospital or have others take care of her. Answer C: Idiopathic intracranial hypertension may lead to blindness in extreme cases due to increased intracranial pressure. This is unlikely due to the normal fundoscopic examination and lack of headaches or other neurologic complaints.

A 49-year-old female is brought to the emergency room by ambulance for altered mental status. Emergency personnel note that she is very agitated and combative. She complains of insects crawling over her skin and is witnessed attempting to shrug them off of her body. A review of the medical record reveals a history of arthritis, carpal tunnel syndrome, and alcohol abuse. Vitals reveal a temperature is 38.8ºC (102ºF), heart rate of 129/min, and blood pressure of 178/101 mmHg. Physical examination reveals her pupils to be 8 mm bilaterally, mild generalized diaphoresis, diffuse low frequency tremor, and severe psychomotor agitation. A full physical examination is limited due to lack of cooperation with the examiner. Mental examination reveals significant psychomotor agitation, tangential speech, disorganized thought processes such that she believes the medical personnel are FBI agents. Question 1 of 2 in this set The most likely diagnosis is A.acute alcohol intoxication B. delirium tremens C. marijuana intoxication D. neuroleptic malignant syndrome E. phencyclidine intoxication

The correct answer is: B Delirium tremens (DTs) describes late, acute withdrawal symptoms with an onset of 48-96 hours after the last drink and cessation from heavy drinking. DTs are otherwise known as alcohol withdrawal delirium. Symptoms of DTs include the following: tremors, an increase in psychomotor activity, vivid hallucinations, illusions, delusions, profound disorientation and confusion, increased autonomic activity (tachycardia, hyperhidrosis, fever, dilated pupils). Patients may also experience formication (tactile hallucinations that feel like insects crawling all over the skin). The clinical presentation is highly suggestive for acute alcohol withdrawal delirium. Delirium tremens typically presents within 48 to 96 hours after cessation of alcohol consumption. Symptoms include tachycardia, psychomotor agitation, combativeness, hypertension, and fever. Left untreated, it can be fatal. Treatment is with intravenous benzodiazepines to control psychomotor agitation as well as supportive measures. In this case scenario, the patient presents with fever, tachycardia, hypertension, formication, agitation, confusion, disorientation with reality, paranoia, and dilated pupils. In addition, given her history of long-standing alcohol abuse, the most likely diagnosis given the clinical scenario is DTs. Answer A: Alcohol intoxication presents with slurred speech, gait disturbances, altered mental status, nausea, and vomiting. No delirium, tremor, or autonomic hyperactivity develops. Answer C: Marijuana intoxication presents with feelings of euphoria, paranoia, increased appetite, and a decrease in alertness. Answer D: Neuroleptic malignant syndrome (NMS) is an idiosyncratic and life threatening complication of neuroleptic medication(s) that is an emergency and can be fatal if not treated skillfully in a timely manner. As indicated by the name, it is most commonly associated with neuroleptic medications such as haloperidol, chlorpromazine, risperidone, etc. NMS is defined as the development of severe muscular rigidity, an elevated temperature (hyperthermia) in the setting of recent neuroleptic use, and two or more of the following symptoms: profuse diaphoresis, tremors, dysphagia, incontinence, changes in level of consciousness, mental status changes, mutism, elevated or labile blood pressure, tachycardia, tachypnea, leukocytosis, and laboratory evidence of muscular injury (elevated CPK). The symptoms should not be better accounted for by substance use or a general medical condition or a neurological condition. In this case scenario, the patient has elevated blood pressure, tachycardia, hyperthermia, tachypnea, severe muscular rigidity, and diaphoresis. There is no mention of neuroleptic medication use or psychiatric or neurological disorders. Thus, alcohol withdrawal is a more likely scenario. Answer E: Phencyclidine (PCP) intoxication presents with horizontal and vertical nystagmus, paranoia, combativeness, and hallucinations. Bottom Line: Delirium tremens typically presents within 48 to 96 hours after cessation of alcohol consumption. Symptoms include tachycardia, psychomotor agitation, combativeness, hypertension, and fever. Left untreated, it can be fatal. Treatment is with intravenous benzodiazepines to control psychomotor agitation, and supportive measures. Management of delirium tremens includes benzodiazepines to control psychomotor agitation and to prevent progression to severe withdrawal. Generally these patients are put on the Clinical Institute Withdrawal Assessment for Alcohol Scale (CIWA) as a measure of withdrawal severity. Those scoring 8 or above (depending upon the institution) receive supplemental benzodiazepines to dampen symptoms. In patients with refractory delirium tremens, barbiturates (specifically phenobarbital) can be very effective when given with benzodiazepines.

A 64-year-old male complains of mild memory problems and auditory hallucinations of a man speaking to him about his future. He states to know that this man is not real, and that he started hearing the voice about six months ago. Past medical history is pertinent for two strokes, hypertension, and gout. The patient is alert and oriented to person and place but not with time and situation. Mental status examination reveals a neat appearing male with reduced affect, dysphoric mood, concrete thought processes, fair abstract ability, and impaired short-term recall. He can only recall two out of three objects after five minutes, and when asked how many legs does a dog have, he replies, "three." Physical examination reveals visual disturbances for which he wears glasses and slow gait due to left-sided weakness. He denies any past psychiatric history as well as alcohol or drug abuse. The most likely diagnosis is A. Briquet's syndrome B. Ganser's syndrome C.Munchausen syndrome D. Munchausen syndrome by proxy E. Wernicke's encephalopathy

The correct answer is: B Ganser's syndrome is a poorly understood condition and has been classified by some as a sub-type of factitious disorder while others have a classified it is a dissociative disorder. The ratio of men to women with this disorder is 2:1. Patients with Ganser's syndrome often present with clouded consciousness, somatic complaints, dissociative symptoms, and perceptual disturbances (pseudohallucinations that the patient knows are not real). The hallmark sign in patients with Ganser's syndrome is paralogia, the responding of approximate answers to questions. For example, if asked how many legs a zebra has, one may answer "three." Such an answer provides the correct category of the response (number of legs), but provides an incorrect but almost correct answer to the question. Some reports have noted the finding of Ganser's syndrome in individuals with strokes and traumatic brain injuries. Some reports have labeled a form of Ganser's syndrome as prison psychosis, in which prisoners will approximate answers to questions to obtain leniency for courts, etc. Some case reports of Ganser's syndrome have identified psychological stressors that have precipitated the syndrome. Answer A: Somatization disorder is otherwise known as Briquet's syndrome, named after a British physician who studied patients with long courses of multi-systemic complaints. However, modern classification has labeled the condition as somatization disorder (now known as somatic symptom disorder in DSM-5). Answer C: The term, Munchausen syndrome, is often used interchangeably with the DSM diagnosis of factitious disorder, but it actually refers to a severe sub-type of factitious disorder, which involves traveling from hospital to hospital or from clinic to clinic, and the presence of pseudologia fantastica (pathological, habitual lying in which extensive histories are elaborated). Factitious disorder involves the intentional, voluntary production of physical or psychological signs or symptoms that are not explained by any other physical or mental disorder. The primary motivation for the behavior is to assume the sick role and incentives (financial, legal, etc) are absent. Answer D: In Munchausen syndrome by proxy, otherwise known as factitious disorder by proxy, an individual intentionally produces physical signs or symptoms in someone under their own care. A common scenario of Munchausen syndrome by proxy occurs when a mother intentionally deceives medical providers about her child's illness by either relaying false information or by inducing illness or injury in the child. The motivation for the caretaker is to indirectly assume the sick patient role. Cases of Munchausen syndrome by proxy have included situations of caretakers of the elderly or for cognitively impaired adults. Answer E: Wernicke's encephalopathy is an acute neurological disorder seen in alcoholics characterized by nystagmus, ataxia, ophthalmoplegia (ocular abnormalities such as horizontal nystagmus, lateral orbital palsy, gaze palsy), and/or confusion. Wernicke's encephalopathy occurs due to thiamine deficiency or malabsorption, which occurs frequently in those who abuse alcohol. Thiamine is a co-factor for several important enzymes. It can be fatal but is most often reversible with thiamine. It is important to give thiamine before administering glucose because if given vice versa, the encephalopathy can rapidly worsen. Wernicke's encephalopathy may resolve after treatment or may progress into the more severe, often irreversible condition of Korsakoff syndrome. In this case scenario, the patient denied any alcohol abuse. Bottom Line: The hallmark sign of Ganser's syndrome is the response of approximate answers when asked questions, and may present with pseudohallucinations, clouded consciousness, and somatic complaints.

A 34-year-old male patient presents with depressed mood, decreased appetite, decreased interest in his usual activities, and low energy. These symptoms have been going on for about three weeks now, and he has not been able to go to his part-time job recently because he has been staying at home, paranoid that he will be "eaten alive by the zombies" if he goes out too much. His mother reports that he has been paranoid, hallucinating, with disorganized thoughts in the past when he was not depressed and sometimes, these symptoms would last about seven months or more. Patient denies any history of alcohol abuse or dependence, and denies any recent illicit drug use but did use marijuana in his early 20s. He has no known medical problems. Vitals are within normal limits. The most likely diagnosis is A. delusional disorder B. schizoaffective disorder C. schizophrenia D.schizophreniform disorder E. severe depression with psychotic features

The correct answer is: B Schizoaffective disorder comprises an uninterrupted period of illness during, which at some time, there is either a major depressive episode, a manic episode, or a mixed episode concurrent with symptoms that meet Criterion A for schizophrenia. During the same period of illness, there have been delusions or hallucinations for at least two weeks in the ABSENCE of prominent mood symptoms. Symptoms that meet criteria for a mood episode are present for a substantial portion of the total duration of the active and residual periods of the illness. The disturbance is not due to the direct physiological effects of a substance or a general medical condition. Schizophrenia is characterized by at least two or more of the following Criterion A, each present for a significant portion of time, continuously for more than six months: (1) delusions; (2) hallucinations; (3) disorganized speech; (4) grossly disorganized or catatonic behaviors; (5) negative symptoms. Delusions, disorganized speech, hallucinations, and agitation are considered to be positive symptoms, associated with dopamine receptors. Negative symptoms of schizophrenia are associated with muscarinic receptors and include characteristics such as flattened affect, social withdrawal, anhedonia (lack of interest), apathy, and poverty of thought. Only one of the above five criterion symptoms is required if delusions are bizarre or hallucinations consist of a voice keeping up a running commentary on the individual's behavior or thoughts, or two or more voices conversing with each other. For the diagnosis of a major depressive episode, five or more of the following symptoms have been present during the same two-week period and represents a change from previous functioning: (1) depressed mood most of the day, nearly every day; (2) markedly decreased interest or pleasure in most activities; (3) significant weight loss when not dieting and due to decreased appetite or weight gain from increased appetite; (4) insomnia or hypersomnia nearly every day; (5) psychomotor agitation or retardation nearly every day; (6) fatigue or loss of energy nearly every day; (7) feelings of worthlessness or excessive or inappropriate guilt nearly every day; (8) decreased ability to think or concentrate; (9) recurrent thoughts of death or suicidal ideation. At least one of the symptoms has to be either (1) depressed mood or (2) loss of interest or pleasure. These symptoms cause clinically significant distress or impairment in social, occupational, or other important areas of functioning. A manic episode consists of a distinct period of abnormally and persistently elevated, expansive, or irritable mood, lasting at least one week (or any duration if hospitalization is necessary). During the period of mood disturbance, three or more of the following symptoms have persisted and have been present to a significant degree: (1) inflated self-esteem or grandiosity; (2) decreased need for sleep and feeling rested after only three hours of sleep; (3) rapid and/or pressured speech; (4) flight of ideas or racing thoughts; (5) distractibility; (6) increase in goal-directed activity (either socially, at work or school, or sexually) or psychomotor agitation; (7) excessive involvement in pleasurable activities that have a high potential for painful consequences (examples: engaging in unrestrained buying sprees, hypersexual behaviors, or foolish business investments). This manic mood disturbance is sufficiently severe to cause marked impairment in occupational functioning or in usual social activities or relationship with others, or to necessitate hospitalization to prevent harm to self or others, or there are psychotic features. A mixed episode occurs when criteria are met for both a manic episode and for a major depressive episode nearly every day during at least a one-week period. The mood disturbance is sufficiently severe to cause marked impairment in occupational functioning or in usual social activities or relationship with others, or to necessitate hospitalization to prevent harm to self or others, or there are psychotic features. Thus, in this case scenario, the patient presents with a depressive episode concurrently with psychotic symptoms of schizophrenia, and he has schizoaffective disorder, as manifested by a history of psychotic symptoms for at least two weeks in the absence of mood symptoms. Answer A: Delusional disorder involves delusions of at least one month's duration that do NOT meet the criteria for schizophrenia. Delusions can involve olfactory or tactile hallucinations if they are related to the delusion. Answer C: In addition to fulfilling the above-mentioned Criterion A symptoms of schizophrenia, the catatonic type of schizophrenia is dominated by at least two of the following criteria: (1) motoric immobility as evidenced by catalepsy (including waxy flexibility) or stupor; (2) excessive motor activity (purposeless and not influenced by external stimuli); (3) extreme negativism (extreme motiveless resistance to all instructions or maintenance of a rigid posture against attempts to be moved) or mutism; (4) peculiar voluntary movements such as bizarre posturing, stereotyped movements, prominent mannerisms, or prominent grimacing; (5) echolalia (uncontrollable and immediate repetition of vocalizations made by someone else) or echopraxia (involuntary repetition or imitation of another person's actions). Answer D: Schizophreniform disorder fulfills the same Criterion A from diagnosis of schizophrenia (please see above) except for the duration of time criteria. Schizophreniform disorder symptoms occur for more than one month but last less than six months, whereas, brief psychotic disorder symptoms last for one month or less. On the other hand, schizophrenia is diagnosed when the symptoms occur for more than six months. Two-thirds of individuals with schizophreniform disorder progress to being diagnosed with schizophrenia. Answer E: Major depression may be classified as with psychotic features if there is the presence of delusions or hallucinations during the major depressive episode. Bottom Line: Schizoaffective disorder comprises an uninterrupted period of illness during, which at some time, there is either a mood episode concurrent with symptoms that meet Criterion A for schizophrenia. During the same period of illness, there have been delusions or hallucinations for at least two weeks in the ABSENCE of prominent mood symptoms. COMBANK Insight : Schizoaffective disorder is reconceptualized as a longitudinal instead of a cross-sectional diagnosis in DSM-5 - more comparable to schizophrenia, bipolar disorder, and major depressive disorder, which are bridged by this condition - and requires that a major mood episode be present for a majority of the total disorders duration after Criterion A has been met.

The parents of a healthy 10-year-old boy hear him screaming in his room two hours after he went to bed. When they enter the room he is sitting up rigidly in bed and crying. He appears fearful and is diaphoretic, flushed, and tachypneic. They are unable to wake him. Eventually he goes back to silent sleep and sleeps through the rest of the night without incident. In the morning he has no recollection of the event and does not recall any particularly frightening dreams. Which of the following is the most likely diagnosis? A. nightmare disorder B. non-REM sleep arousal disorder, sleep terror type C. panic disorder D.REM sleep behavior disorder E. sleep-related seizures

The correct answer is: B Sleep terror disorder occurs during stage III or stage IV sleep, and events are either not recalled at all, or only distantly remembered by patients. Sleep terror disorder typically occurs in children, is more common in boys, occurs in one to six percent of children, and tends to have a genetic component. Sleep terror also a high comorbid association with sleepwalking disorder. It includes autonomic and behavioral manifestations of fear, and patients remain unresponsive to external stimuli, making it difficult to awaken. If they do awaken, they are typically confused and have no recollection of the event. This disorder occurs during the non-REM stages of sleep and is not associated with dreaming. This child does not awaken during the parents attempts, he has no recollection of the event in the morning, and he does not report a dream, all of which are consistent with a sleep terror disorder. Typically, no treatment is necessary, however, small doses of diazepam at bedtime can be effective. Answer A: Nightmare disorder is characterized by frightening dreams during REM sleep. Patients are easily aroused (or may wake up on their own) during this time, which sets it apart from sleep terror disorder. Frequently, patients remember the exact dream in the morning, while episodes during sleep terror disorder are not typically recalled. Answer C: Panic attacks may also cause abrupt awakenings from deep REM sleep accompanied by fearfulness, but these episodes produce rapid and complete awakening without the confusion, amnesia, or motor activity typical of NREM sleep arousal disorders. Answer D: REM sleep behavior disorder also occurs during REM sleep and is characterized by "acting out" dreams. This includes walking, kicking, punching, etc., which can lead to injuries of the patient or any partner in bed. Answer E: Some types of seizures can produce episodes of very unusual behaviors that occur predominantly or exclusively during sleep. Nocturnal seizures may closely mimic NREM sleep arousal disorders but tend to be more stereotypic in nature, occur multiple times nightly, and be more likely to occur from daytime naps. The presence of sleep-related seizures does not preclude the presence of NREM sleep arousal disorders. Bottom Line: Sleep terror disorder is characterized by unresponsiveness during the event and amnesia after the event. It occurs during NREM sleep.

A 20-year-old female with a history of generalized anxiety disorder presents with an exacerbation of life stressors. History reveals that she is often angry with her spouse for unknown reasons and is found counting or repeating words silently in a ritualistic manner. She also is fixated on having everything in her apartment symmetrical and may spend one to three hours per day cleaning in this fashion. Which of the following compulsions is most likely to be discovered upon further history taking? A. avoiding social situations B. excessive washing C. ignoring the urge to recheck tasks D. persistent thoughts of contamination E. repetitive unwanted images

The correct answer is: B The most common compulsion associated with this disorder is excessive grooming, hand washing, tooth brushing, etc. This occurs in approximately 60% of patients. Obsessions are defined by the following 4 criteria: Recurrent and persistent thoughts, impulses, or images are experienced at some time during the disturbance as intrusive and inappropriate and cause marked anxiety and distress. Persons with this disorder recognize the pathologic quality of these unwanted thoughts (such as fears of hurting their children) and would not act on them, but the thoughts are very disturbing and difficult to discuss with others. The thoughts, impulses, or images are not simply excessive worries about real-life problems. The person attempts to suppress or ignore such thoughts, impulses, or images or to neutralize them with some other thought or action. The person recognizes that the obsessional thoughts, impulses, or images are a product of his or her own mind (not imposed from without, as in thought insertion). Compulsions are defined by the following 2 criteria: An individual performs repetitive behaviors (eg, hand washing, ordering, checking) or mental acts (eg, praying, counting, repeating words silently) in response to an obsession or according to rules that must be applied rigidly. The behaviors are not a result of the direct physiologic effects of a substance or a general medical condition. The behaviors or mental acts are aimed at preventing or reducing distress or preventing some dreaded event or situation. However, these behaviors or mental acts either are not connected in a way that could realistically neutralize or prevent whatever they are meant to address or they are clearly excessive. Answer A: Avoidance of social situations is not considered an compulsion. Answer C: Rechecking tasks is a compulsion, but the act of ignoring them is not. Regardless, the overwhelming desire to recheck tasks is very common and occurs in approximately 30% of all OCD patients. A person suffering from OCD attempts to ignore, avoid, or suppress obsessions or to neutralize them with another thought or action (e.g., performing a compulsion). Answers D & E: This is an obsession. Obsessions are repetitive and persistent thoughts (eg, of contamination), images (eg, of violent or horrific scenes), or urges (eg, to stab someone). Bottom Line: The most common compulsion associated with obsessive-compulsive disorder is excessive grooming, hand washing, tooth brushing, etc. This occurs in approximately 60% of patients. COMBANK Insight : This item asks a basic question: what is a compulsion vs. obsession? Take time to identify the meaning of the final question and understanding the differences between the answer choices provided. Many COMAT items will surround the basic foundations of the specialty at hand - in this case the history and physical.

A previously healthy 15-year-old boy is brought to the doctor by his parents because he has been unusually sleepy recently. He says that he finds that he will often fall asleep unexpectedly in class, even when he has slept well the night before. He frequently takes afternoon naps and will awaken feeling unrefreshed. He reports vivid dreams both while sleeping at night and napping during the day. He also reports at times feeling paralyzed when he first awakens, though he is able to breathe, and the feeling passes within a couple of minutes. After his parents are asked to leave the room, he admits to smoking marijuana occasionally with friends and to drinking up to six beers at a time on the weekends. He says he is doing well in school but is worried because he is unable to concentrate when he is tired. On physical examination, he is a well-nourished, well-appearing adolescent with no focal neurological findings or other relevant findings. Which of the following is the best recommendation for his condition? A. avoid daytime naps B. maintain a regular sleep schedule C.refrain from participating in sports D. start alcohol/drug rehabilitation E. start light therapy

The correct answer is: B The patient is most likely suffering from narcolepsy, which classically presents with the tetrad of excessive daytime sleepiness, cataplexy, hypnagogic hallucinations, and sleep paralysis, though all four symptoms are only seen rarely in children. Narcolepsy is thought to be a genetic disorder and is the result of neurotransmitter dysfunction as well as abnormal immune modulation. This disorder typically manifests in teenage years or early twenties, but can present in young children as well. Diagnosis requires thorough history taking and is confirmed with a polysomnogram that is necessary to rule out other sleep disorders, and will typically show short REM latency. The presence of true cataplexy is also highly suggestive of the diagnosis. Treatment consists of maintaining a regular sleep schedule, timed daily naps, moderate exercise, including participation in sports, and emotional support. Stimulant drugs can also be added, such as amphetamines and methylphenidate, for increased energy and awareness. Refraining from eating sugary foods has also been shown to have some possible benefit. Answers A & C: Daytime naps and moderate exercise may help regulate his sleep cycle. He should not be counseled to avoid these activities. Answer D: While this patient should cut back on his alcohol intake and stop using marijuana, there are unrelated to his narcolepsy and its outcome. Answer E: Light therapy is not indicated in treatment of narcolepsy. Bottom Line: Narcolepsy classically presents with the tetrad of excessive daytime sleepiness, cataplexy, hypnagogic hallucinations, and sleep paralysis, though all four symptoms are only seen rarely in children.

A 22-year-old female presents for evaluation at the suggestion of her mother. History reveals the patient recently started professional school which has required her to move across the country. She has kept in close contact with family back home via telephone and after a recent conversation her mother became worried about her complaints of depressed mood, nervousness, lack of appetite, poor concentration and difficulty sleeping for the past 3 months. Her grades have dropped, she has failed several exams and she now worries that a significant scholarship will be revoked as a result. Mental status examination reveals a quiet and withdrawn female with an irritable and anxious affect who is noted to frequently pace the halls and fidget with her hands. Follow-up with her primary care physician back home reveals the results of routine laboratory studies including a TSH of 2.8 mIU/L. The most likely diagnosis is A. acute stress disorder B. adjustment disorder with mixed anxiety and depressed mood C.bipolar II disorder D. major depressive disorder with anxious features E. mood disorder secondary to general medical condition

The correct answer is: B The question stem describes a classic adjustment disorder - a stressful life event has resulted in maladaptive symptoms of anxiety (worry, insomnia, poor appetite) that are creating significant impairment in functioning (losing a scholarship). While the patient is going through life changes and would be understandably stressed, her distress is in excess of what would be expected in this situation since her grades have dropped to the point of losing a scholarship. With adjustment disorders the symptoms (which can range from depressed mood to anxiety to disturbance of conduct such as agitation and aggression) begin within 3 months after the stressful life event and resolve within 6 months. Treatment includes supportive psychotherapy and possible pharmacotherapy for the associated symptoms of depression, anxiety and/or insomnia. Because this patient's symptoms are clearly linked to psychosocial factors, the correct answer is adjustment disorder rather than another Axis I anxiety disorder such as generalized anxiety disorder. There is no mention of mood symptoms that would characterize a mood disorder diagnosis. Answer A: Acute stress disorder is considered when a patient is exposed to a traumatic event and either while experiencing or after experiencing this event they develop three or more dissociative symptoms such as depersonalization, derealization, dissociative amnesia, reduced awareness of surroundings or a subjective sense of numbing/detachment. However, the traumatic event is further defined as one in which the patient experiences, witnesses or is confronted with actual or threatened death or serious injury or threat to the physical integrity of self or others which elicited a response of intense fear, helplessness or horror. While the question stem describes a stressful exam failure, moving away to school certainly does not fit the criteria for a traumatic event and the patient is exhibiting symptoms of depression and anxiety rather than dissociative symptoms. Answer C: Bipolar II Disorder is a mood disorder defined by the presence of hypomanic and depressive episodes. Hypomania is characterized by at least four days of persistently irritable or expansive mood with associated distractibility, decreased need for sleep, pressured speech, grandiosity, racing thoughts, increase in goal-directed activity and excessive risky behavior like spending sprees or indiscriminate sexual activity. The degree of impairment is not so severe as to require hospitalization and it is this caveat, along with the minimum duration of symptoms, that differentiates hypomania from mania and so excludes a diagnosis of bipolar I disorder. The question stem describes anxiety symptoms in the context of a new life stressor and so adjustment disorder is the most appropriate diagnosis. There is no mention of a hypomanic episode and so bipolar II disorder would not be included in the differential diagnosis. Answer D: Major depressive disorder is defined as the presence of at least five of the following symptoms on most days for at least one month: depressed mood, anhedonia, sleep disturbance, excessive feelings of guilt, fatigue, poor concentration, change in appetite, psychomotor agitation or retardation and suicidal thoughts (at least one symptom must be depressed mood or anhedonia). While the question stem mentions some irritability as well as changes in sleep and appetite, the main symptom the patient is experiencing is anxiety with low appetite, insomnia and psychomotor agitation Her symptoms started after a stressful life event (starting professional school far away from her family). Thus, adjustment disorder with anxiety is the most appropriate diagnosis. Note that there are several other subtypes of adjustment disorder including adjustment disorder with depressed mood - this would be an appropriate diagnosis if the question stem described prominent symptoms of depressed mood or decreased interest or pleasure in most activities. Answer E: The question stem references the patient's TSH, which is within normal limits (with the normal range being 0.4 to 4 mIU/L. This rules out a depressive disorder secondary to general medical condition like hypothyroidism. Hypothyroidism can cause depression while hyperthyroidism can cause anxiety symptoms. The constellation of symptoms that are clearly linked to psychosocial stressors makes adjustment disorder the best answer choice. Bottom Line: There are several subtypes of adjustment disorders: adjustment disorder with depressed mood, with anxiety, with mixed anxiety or depressed mood, with disturbance of conduct, with mixed disturbance of emotions and conduct, and unspecified. When significant mood, anxiety or behavioral symptoms which cause excessive distress or clinical impairment develop up to three months after a clearly identifiable life stressor and end within six months, one of these diagnoses is made. The association of symptoms with psychosocial factors differentiates it from other Axis I mood and anxiety disorders.

A 28-year-old male complains of sadness, oversleeping, fatigue, and decreased interest in activities of three weeks' duration. He reports eating more than usual, particularly craving carbohydrates, and gaining 4.52 kg (10 lbs) as a result within one week. He states that all of these feelings and behaviors occur nearly every day. He was having difficulty completing his tasks at work and was ultimately fired due to lack of productivity. History reveals past depressive episodes occurring since early adolescence. He reflects back on a period last year when he had periods of insomnia associated with a flurry of ideas and subsequent accomplishments, and he states that he wishes to have that feeling again. The most likely diagnosis is A. adjustment disorder with depression B. bipolar disorder, depressed phase C. hypomanic episode D.major depressive disorder E. manic episode

The correct answer is: B The question stem presents a patient currently in a depressive episode. It describes atypical features of depression with hypersomnia, hyperphagia and carbohydrate craving. These depressive symptoms have caused significant impairment in functioning as he was ultimately fired from his job. It also mentions that his depression first began in adolescence and eludes to a manic episode he experienced last year. Because of this past manic episode the patient meets criteria for bipolar disorder and is currently in a depressed phase. Bipolar patients generally spend the majority of their illness in depression and so careful questioning about past periods of elevated mood is important in making an accurate diagnosis. Furthermore, studies have shown a greater overall depression severity in bipolar patients versus those suffering from unipolar depression (major depressive disorder). Atypical depressive symptoms (also known as "reverse" neurovegetative signs) have been associated with bipolar depression although they can be seen in patients with major depression. Earlier age of illness onset is also characteristic of bipolar disorder versus other mood disorders, and this patient began experiencing mood symptoms as an adolescent. The mainstay of treatment for bipolar depression is a mood stabilizer as use of standard antidepressants also is controversial due to concerns about inducing mania, hypomania or rapid cycling. The DSM-5 criteria for a manic episode requires the following: A. A distinct period of abnormally and persistently elevated, expansive, or irritable mood and abnormally and persistently increased activity or energy, lasting at least 1 week and present for most of the day, nearly every day. B. During this period three of the following symptoms have persisted representing a noticeable change from usual behavior: inflated self-esteem or grandiosity, decreased need for sleep, more talkative, flight of ideas, distractibility, increased goal-directed activity, or excessive involvement in activities that have a high potential for painful consequences. C. The episode is sufficiently severe to cause marked impairment in social or occupational functioning or to necessitate hospitalization to prevent harm to self or others, or there are psychotic features. The DSM-5 criteria for a hypomanic episode requires the following: A. A distinct period of abnormally and persistently elevated, expansive, or irritable mood and abnormally and persistently increased activity or energy, lasting at least 4 consecutive days and present for most of the day, nearly every day. B. During this period three of the following symptoms have persisted representing a noticeable change from usual behavior: inflated self-esteem or grandiosity, decreased need for sleep, more talkative, flight of ideas, distractibility, increased goal-directed activity, or excessive involvement in activities that have a high potential for painful consequences. E. The episode is NOT severe enough to cause marked impairment in social or occupational functioning. Answer A: Adjustment disorders are characterized by the onset of mood, anxiety and/or behavioral symptoms up to three months after a significant life stressor. These symptoms resolve within six months and are clearly associated with psychosocial factors. There is no significant life stressor mentioned in the question stem. It mentions that the patient was fired from his job but this was due to his already present depressive symptoms interfering with his work performance. The following subtypes exist for adjustment disorder: with depressed mood, with anxiety, with mixed anxiety or depressed mood, with disturbance of conduct, with mixed disturbance of emotions and conduct, and unspecified. Note that manic or hypomanic symptoms are not part of an adjustment disorder. This patient's combined history of mania and depression make bipolar depression the appropriate diagnosis. Answer C: The question stem does describe a past period of elevated and expansive mood with increase in goal-directed activity, symptoms characteristic of both mania and hypomania. However, the key is that these symptoms were seen in a past episode and the current episode is characterized by depressive symptoms - making bipolar depression the most appropriate answer. Remember that bipolar I disorder is diagnosed when there is at least one current or past episode of mania (which lasts seven days or longer or is severe enough to require hospitalization) and bipolar II is diagnosed when there is current or past hypomania (which last at least four days and is not severe enough to necessitate hospitalization). Thus, it is duration and symptom severity that differentiates bipolar I from bipolar II disorder. Depression is a core feature of both illnesses. Answer D: The patient does meet criteria for a current major depressive episode as he has a greater than two week history of depressed mood, sleep disturbance, anhedonia, change in appetite and poor concentration. However, the question stem mentions a past episode of expansive mood characteristic of mania and so the correct diagnosis is bipolar depression rather than unipolar depression. While depression is a core feature of bipolar illness and most patients spend the majority of time in the depressive rather than manic phase, bipolar disease must be carefully differentiated from major depression as the treatments differ. This patient would benefit from a mood stabilizer rather than a standard antidepressant alone (such as a selective serotonin reuptake inhibitor) as the use of standard antidepressants alone in bipolar disorder raises concerns about induction of mania. Answer E: The question stem does describe a one week period of expansive mood characteristic of a manic episode, however this was a past episode that occurred one year ago. The current episode is depression and so the most appropriate answer choice is bipolar depression. Based on the duration of the past episode of elevated mood, that past episode would be characterized as mania (which lasts at least seven days or is severe enough to require hospitalization) and thus this patient suffers from bipolar I depression. Hypomania, in which symptoms last at least four days and are not severe enough to cause hospitalization, is seen with bipolar II disorder. Depression is a core feature of both illnesses. Bottom Line: Careful questioning is required to differentiate major depressive disorder from bipolar disorder and clinicians evaluating patients complaining of depression must search for past periods of elevated mood and/or mixed symptoms. Differentiating these two illnesses is important because their course and treatment differ.

A 19-year-old male presents with a behavioral disturbance after his brother suddenly died in a work-related accident 1 month ago which he witnessed. History reveals that he complains of difficulty sleeping, forgetfulness, numbness, and feeling detached from life. A review of systems indicates that he experiences flashbacks related to the event. The most appropriate management for this patient is A. amitriptyline B. cognitive behavioral therapy C. psychological debriefing D.selegiline E. trazodone

The correct answer is: B This patient likely has Posttraumatic Stress Disorder (PTSD). Although limited evidence exists for providing direct psychological intervention within the first month after trauma, the use of cognitive behavior therapy (CBT) has been supported. In the cognitive therapies, patients are assisted in thinking about the traumatic event and themselves more realistically. It can also include additional approaches such as education and coping skills training. CBT alone or in combination with hypnosis is more effective than supportive counseling in preventing PTSD. Overall, CBT is the most effective intervention for reducing symptoms and decreasing the future incidence of PTSD. Other forms of psychotherapy for PTSD include exposure therapy, coping skills training, and eye movement desensitization and reprocessing (imagining a scene from the trauma and attempting to reduce the associated anxiety). Answers A & D & E: Selegiline is a monoamine oxidase inhibitor, trazodone is a serotonin modulator, and amitriptyline is a tricyclic antidepressant. First line pharmcotherapy for PTSD includes either an SSRI or an SNRI. First line pharmcotherapy for PTSD includes either an SSRI or an SNRI. There is insufficient evidence of the effectiveness of tricyclic antidepressants (TCAs), monoamine oxidase inhibitor (MAOIs), serotonin modulators (e.g., trazodone), or atypical antidepressants (e.g., mirtazapine) for PTSD. Answer C: Psychological debriefing is a short intervention provided immediately after a person has experienced trauma. Critical Incident Stress Debriefing is a widely used version that is often part of a multi-component program. It is typically delivered in a group setting two to 10 days after a traumatic event and is meant to mitigate emotional distress by allowing patients to share emotions about the event, providing education and tips on coping, and attempting to normalize patients' reactions to trauma. However, most studies that have shown benefits have limitations. It appears that use of Critical Incident Stress Debriefing at best results in no effect and at worst harms participants by impeding natural recovery. Therefore, this practice is not warranted. Bottom Line: Individuals with PTSD experience marked cognitive, affective, and behavioral responses to stimuli reminding them of trauma they experienced, eg, flashbacks, severe anxiety, and fleeing or combative behavior. A trauma-focused cognitive-behavioral therapy (CBT), medication (SSRI or SNRI), or a combination of both modalities is recommended. Cognitive-behavioral therapy has been found to be more effective than supportive counseling for short-term treatment and prevention of PTSD. COMBANK Insight : DSM-5 criteria for PTSD differ significantly from the DSM-IV criteria. The stressor criterion A is more explicit with regard to events that qualify as "traumatic" experiences. Also, DSM-IV Criterion A2 (subjective reaction) has been eliminated. Whereas there were three major symptom clusters in DSM-IV - re-experiencing, avoidance/ numbing, and arousal - there are now four symptom clusters in DSM-5, because the avoidance / numbing cluster is divided into two distinct clusters; avoidance and persistent negative alterations in cognitions and mood. This latter category, which retains most of the DSM-IV numbing symptoms, also includes new or re-conceptualized symptoms, such as persistent negative emotional states. The final cluster - alterations in arousal and reactivity - retains most of the DSM-IV arousal symptoms. It also includes irritable behavior or angry outbursts and reckless or self-destructive behavior. PTSD is now developmentally sensitive in that diagnostic thresholds have been lowered for children and adolescents. Furthermore, separate criteria have been added for children age 6 or younger

A 30-year-old female presents to the clinic complaining of palpitations. She has been unemployed for the past few months. While she is home, she suddenly feels like her heart is jumping out of her chest when she notices herself worrying about her financial situation. These episodes are often associated with sweating and paresthesias. She has had several episodes over the past few months. Her mother had a history of myocardial infarction and hyperlipidemia which is why she is worried she is having a heart attack as well. Physical examination and vitals are unremarkable. The patient is given Lorazepam and her condition improves. Which of the following is the most appropriate long-term management at this time? A. diazepam B. paroxetine C.propranolol D. psychodynamic therapy E. regadenoson stress test

The correct answer is: B This patient's presentation of sudden onset of palpitations, fear of the unknown and financial situation along with paresthesias is consistent with panic attacks. These attacks are characterized by persistent concern or anxiety about specific factors like finances and health. Selective serotonin reuptake inhibitors (SSRI) are the first line therapy for panic disorder. Most common SSRIs used include paroxetine, fluoxetine, and sertraline as well as SNRI venlafaxine which are all FDA approved for panic disorder. Typically it is started as low dose therapy and if a patient has even a partial response to it, the dose can be slowly increased. However, if the patient has not shown any signs of improvement after six to eight weeks on a therapeutic dose, the medication can be tapered off and another class of medication can be tried instead. Psychological therapy with cognitive behavioral therapy depending on availability of resources and patient preference can be used as well to enhance the pharmacologic drug effect. Answer A: Diazepam is a benzodiazepine which helps curb symptoms of anxiety by potentiating the effects of endogenous GABA. Benzodiazepines are more useful for acute symptom relief during the period before an SSRI takes effect or in addition to an SSRI for partial responders. They are particularly useful in those who are severely symptomatic and functionally impaired, which currently this patient is not per the history. Side effects include dependence especially in someone with a history of drug or alcohol abuse. Other side effects include amnesia, drowsiness and impaired psychomotor performance. Answer C: There is no clear role of propanolol in management of panic disorder since palpitations is only a clinical manifestation of the underlying psychological disorder. Trials have found monotherapy with beta-blockers or clonidine, an alpha-2 noradrenergic agonist, to be no more effective for panic disorder than placebo. Answer D: Psychodynamic therapy may be effective for panic disorder and agoraphobia, but cognitive behavioral therapy is more appropriate for these conditions. Cognitive behavioral therapy is proven to be beneficial for many psychological disorders including generalized anxiety disorder, PTSD, anorexia nervosa, and panic disorders. Answer E: Even though this patient has a positive family history of heart disease, her symptoms of palpitations with history of constant worrying in absence of chest pain is more consistent with panic attacks than cardiac syndrome. Additionally, the patient is only 30 years old with a family history of coronary artery disease. She is very likely capable of undergoing an exercise or treadmill stress test as opposed to a ragadenoson or pharmacologic test. However, this patient should at least be screened with an electrocardiogram. Bottom Line: SSRI's are first line treatment for management of panic attacks.

A 34-year-old female presents to the emergency department with confusion, mydriasis and dry mucous membranes. She appears flushed with vital signs revealing a blood pressure of 98/64 mmHg and heart rate of 122/min. Mental status examination reveals flat affect, hopelessness and active suicidal ideation with the patient admitting to "eating some pills." The most likely medication ingested is A. acetaminophen B. amitriptyline C. hydrocodone D.lithium E. methylphenidate

The correct answer is: B Tricyclic antidepressants are potentially fatal in overdose and serious suicidal ideation with high risk of attempt is a relative contraindication to their use in treatment of mood disorders. Overdose can result in seizure and fatal cardiac conduction abnormalities due to blockade of sodium channels in the myocardium. ECG must be monitored for prolonged PR interval or widening of the QRS interval, which can signal progression to ventricular tachycardia or torsades de pointes. The treatment for arrhythmia consists of sodium bicarbonate, which increases extracellular sodium and neutralizes the pH to decrease drug binding to sodium channels. The anticholinergic effects of tricyclics leads to confusion, tachycardia, dilated pupils (mydriasis), dry mouth, urinary retention and constipation. Answer A: A patient who acutely overdosed on acetaminophen would present with nausea, vomiting, fever, pancytopenia as well as possible renal failure and hypoglycemic coma. Acetaminophen overdose can be potentially fatal due to hepatotoxicity and patients should be immediately treated with the antidote (N-acetyl cysteine or NAC) to protect against this without waiting for the serum acetaminophen levels to result. Gastric lavage and activated charcoal are also utilized in acute ingestions to prevent further absorption. Answer C: Opioid overdose presents with pupillary constriction (miosis) and CNS depression manifested by decreased level of consciousness (on a continuum from drowsiness to coma based on quantity ingested) and decreased respiratory drive. Patients often appear unconscious, pale and cyanotic. Pulmonary edema is a common complication due to central respiratory inhibition. The most emergent step in treatment is ensuring ventilation and airway support. Naloxone, an opioid antagonist, is often used to reverse the CNS depressant effects of the ingested drug and precipitates opioid withdrawal. Answer D: Acute lithium toxicity causes vomiting, ataxia, course tremor, dysarthria, confusion, seizure and bradycardia. Arrhythmia can result and ECG should be monitored. ECG changes due to lithium toxicity include flattened T waves, ST segment depression and QTC prolongation. Serial serum lithium levels must be trended as up to 12 hours or longer may be required before peak serum levels are reached in an overdose. The serum level does not necessarily correlate with clinical signs of toxicity and so treatment should be based on clinical manifestations. Treatment is supportive with whole bowel irrigation for large acute ingestion (as activated charcoal does not prevent absorption) and IV hydration. Hemodialysis is the treatment of choice for severe toxicity with lithium level greater than 4 mEq/L or a level greater than 2.5 mEq/L with the patient exhibiting significant symptoms of toxicity or are unable to tolerate intravenous hydration. Severe toxicity can be treated with hemodialysis since lithium has no protein or tissue binding. Answer E: Psychostimulant overdose is characterized by pupillary dilation (mydriasis), fever, diaphoresis, psychomotor agitation and hypervigilance that in some patients can progress to paranoid psychosis with delirium. Autonomic hyperactivity with tachycardia and hypertension is characteristic with stroke and myocardial infarction as potential sequela. Treatment includes use of benzodiazepines for agitation and phentolamine for significant hypertension. Bottom Line: Tricyclic antidepressants can cause significant cardiac arrhythmias due to sodium channel inhibition and are potentially fatal in overdose. It is important to become familiar with the signs, symptoms and treatments of the most common overdoses encountered in emergency psychiatry. These include illicit substances (opioids, cocaine, amphetamines, phencyclidine) as well as psychotropic medications (lithium, tricyclic antidepressants) and over the counter medications (acetaminophen, salicylates).

A 6-year-old believes that as long as he physically stays outside of his bedroom it will not be bed time. This is an example of A. a formal operation B. a loose association C. concrete thinking D. dereism E. idiosyncratic thinking

The correct answer is: C Concrete thinking is characterized by actual things, events, and immediate experience, rather than by abstractions. This thinking is seen in young children, in certain cognitive mental disorders, and in schizophrenic persons. As the third stage of Piaget's theory of cognitive development, concrete thinking, starts at age 7 and last till age 11. The child begins to use inductive logic, using a specific experience to apply to a general principal, but has difficulty with hypothetical concepts. Answer A: Formal operation is the fourth stage of Piaget's theory of cognitive development, starting at age twelve and lasting into adulthood. This stage emphasizes deductive logic and the emergence of the ability to use abstract thought for hypothetical situations. Answer B: Loosening of association (also known as derailment or asyndetic thinking) is a thought disorder characterized by disorganized thinking that jumps between ideas that seem entirely unrelated. This can be seen in patients with schizophrenia and during episodes of mania. Answer D: Autistic thinking is a term used to refer to thinking not in accordance with consensus reality that emphasizes preoccupation with inner experience. The thoughts are largely narcissistic and egocentric, with emphasis on subjectivity rather than objectivity, and without regard for reality. The term is used interchangeably with dereism and is characteristically seen in schizophrenia and autistic disorder. Answer E: Also known as Dereistic thinking, this term used to refers to thinking not in accordance with the facts of reality and experience and following illogical reasoning. This term is also used interchangeably with autistic thinking, but idiosyncratic emphasizes disconnection from reality and autistic thinking emphasizes preoccupation with inner experience. Bottom Line: Concrete thinking is characterized by actual things, events, and immediate experience, rather than by abstractions. This thinking is seen in young children and in those who have lost or never developed the ability to generalize, such as in schizophrenia and certain cognitive disorders.

A 75-year-old man with hypertension, type 2 diabetes mellitus, and congestive heart failure is admitted to the hospital because of an exacerbation of his heart failure. He states that last night he could not lay flat and woke up multiple times gasping for breath. When interviewed on admission he says that he wants comfort care only because, "this life is just not worth living." His wife confirms that he has told her many times that the next time he has an exacerbation he does not want aggressive treatment. He is given supplemental oxygen by nasal cannula and morphine to relieve the feeling of breathlessness. Several hours after his arrival he becomes unresponsive. His wife is present when he dies the next day. Later, when describing her experience to her sister she tells her that she is sad about his death but does not show any emotion. Which of the following defense mechanisms is she most likely using? A. denial B. intellectualization C. isolation of affect D. repression E.suppression

The correct answer is: C Defense mechanisms are often used during times of stress to relieve anxiety and protect oneself from conflict and traumatic events. Defense mechanisms are classified into the following: 1) mature defense mechanisms which are healthy and adaptive and seen in normal adults, 2) neurotic defenses, which are commonly seen in obsessive-compulsive patients and adults under stress, 3) immature defenses which are seen in children, teens, psychotic patients, and, at times, in non-psychotic patients. Isolation of affect is one of the neurotic defense mechanisms, and refers to the unconscious limitation of the feeling of emotion associated with a stressful life event. The patient's wife in this vignette is unable to show sorrow for her husband's death when talking about it afterwards, but she does say that she feels sad. Answer A: Denial is a primitive defense mechanism in which the patient denies the existence of a problem or fact because it is too painful for them to acknowledge it. This patient is not denying that her husband died. Answer B: Intellectualization is the overuse of rational thought in place of emotion to distance the person from painful events or feelings. If she was using intellectualization, she might describe how his medical conditions were treated and why, medically, he made the best decision for comfort care. Answer D: Repression is the unconscious prevention of an undesirable thought or feeling from reaching consciousness. She does consciously express her sadness. Answer E: Suppression is a mature defense mechanism in which the subject consciously ignores unacceptable impulses or emotions in order to achieve a desired outcome. Bottom Line: Isolation of affect is the unconscious limitation of the feeling of emotion associated with a stressful life event. COMBANK Insight : Isolation of affect can be confused with intellectualization. If the subject in the question is thinking instead of feeling they are using intellectualization. If they are just not feeling, it is isolation of affect.

A 32-year-old male spies on his attractive female neighbor and believes that she is giving him secret signals that she likes him. History reveals that she was reading a book in her front yard and at the same time, he was checking his mail and received a heart-shaped advertisement. He believes that she placed it there earlier that morning for him. Later he saw her at the grocery store and noticed that she put one of her thumbs upward indicating that she would like him to purchase some flowers for her. He lives alone and works a successful job at the post office. He does not have any medical or past psychiatric problems. The most likely diagnosis is A. Capgras syndrome B. Cotard syndrome C. delusional disorder D.fetishistic disorder E. schizotypal personality disorder

The correct answer is: C Delusional disorder requires delusions lasting for at least 1 month's duration and NOT meeting criteria for schizophrenia. Tactile and olfactory hallucinations may be present if they are related to the delusional theme. Apart from the impact of the delusion(s) or its ramifications, functioning is not markedly impaired and behavior is not obviously odd or bizarre. If mood episodes have occurred concurrently with delusions, their total duration has been brief relative to the duration of the delusional periods. Criterion A of Schizophrenia requires two (or more) of the following, each present for a significant portion of time during a 1-month period (or less if successfully treated):delusions, hallucinations, disorganized speech (e.g., frequent derailment or incoherence)grossly disorganized or catatonic behavior, negative symptoms (i.e., affective flattening, alogia, or avolition). Criteria A of Schizophrenia requires only one symptom if delusions are bizarre or hallucinations consist of a voice keeping up a running commentary on the person's behavior or thoughts, or two or more voices conversing with each other. Answer A: Capgras syndrome has been named for the French psychiatrist who described the illusion des sosies (illusion of doubles). Individuals with Capgras syndrome have a delusion of misidentification that a familiar person has been replaced by an impostor. Capgras syndrome has been seen to occur after sudden brain damage and may be associated with schizophrenia, epilepsy, dementia, and other organic brain disorders. Answer B: Cotard syndrome or nihilistic delusional disorder. Patients with this syndrome complain of having lost not only possessions, status, strength, but also their heart, blood, and intestines. They believe that the world around them has been reduced to nothingness. This rare syndrome is usually considered to be a precursor to a schizophrenic or depressive episode. Answer D: Fetishistic disorder, otherwise known as fetishism, occurs when individuals present with recurrent, intense sexually arousing fantasies, sexual urges, or behaviors involving the use of nonliving objects such as female undergarments, high heels, lace stockings, shoes, jackets, boxing shorts, etc. The nonliving objects are not limited to articles of female clothing used in cross-dressing or devices designed for the purpose of tactile genital stimulation. These fantasies, sexual urges, or behaviors occur over a period of at least six months and they cause clinically significant distress or impairment in social, occupational, or other important areas of functioning. Answer E: Schizotypal personality disorder is marked by a pervasive pattern of social and interpersonal deficits marked by acute discomfort with, and reduced capacity for, close relationships as well as by cognitive or perceptual distortions and eccentricities of behavior. This personality disorder usually manifests by early adulthood and presents in a variety of contexts, as indicated by at least five or more of the following: (1) ideas of reference; (2) odd beliefs or magical thinking that influences behaviors and is inconsistent with cultural norms; (3) unusual perceptual experiences; (4) odd thinking and odd speech marked by circumstantiality, metaphors, etc; (5) suspicious or paranoid ideation; (6) inappropriate or constricted affect; (7) odd, eccentric, or peculiar behaviors; (8) lack of close friends other than first-degree relatives; (9) excessive social anxiety that does not diminish with familiarity and tends to be associated with paranoid fears rather negative self-esteem. Individuals with schizotypal personality disorder often embrace magical beliefs or thinking (cults, superstitions, illusions, perceptual eccentricities). They may be socially inept with odd behaviors, bizarre beliefs, and eccentric clothing. Bottom Line: Delusional disorder involves delusions of at least one month's duration and is not marked by psychosis. COMBANK Insight : Criterion A for delusional disorder no longer has the requirement that the delusions must be non-bizarre in DSM-5. A specifier is now included for bizarre type delusions to provide continuity with DSM-IV.

A 30-year-old man is brought to the emergency department after a minor motor cycle accident. Police state that when they arrived on the scene the patient was alert and oriented, but when asked to present his license he said he did not have one. When asked what his employment was, he states that he has only been in the city for the past week, but cannot seem to recall where he lived prior to this or what his previous occupation was; however, this does not seem to concern him. He informed police that he had just bought the motorcycle that night when he saw an advertisement in the newspaper for it. He states his name is Tom Baker, but cannot provide a social security number or numbers for friends or family. He is fully dressed and speaks coherently. He is attentive to the police interview and is fully cooperative. When the police attempt to identify him through their computer system they find no record of prior police contact. On physical examination, he is alert and oriented. His neurological examination is non-focal and physical examination is unremarkable. While he is in the emergency department, a woman from a neighboring state calls looking for her husband who disappeared earlier that week. She comes in and identifies the patient and says that earlier in the week he witnessed a homicide while working at a convenience store near his home. Question 1 of 2 in this set Which of the following is the most likely diagnosis? A. depersonalization disorder B.dissociative amnesia C. dissociative fugue D. dissociative identity disorder E. post-traumatic stress disorder

The correct answer is: C Dissociative fugue usually occurs after an individual experiences a series of difficulties or traumatic events in his life. Individuals experiencing dissociative fugue suddenly and unexpectedly disappear, usually ending up in locations far from home and cannot seem to recall their past. They typically adopt a new identity and have no recollection of their previous one. Dissociative fugue is a rare disorder and is a way in which people defend themselves against overwhelming circumstances in their lives. Diagnostic criteria are as follows: - Sudden, unexpected departure from home to a new location. - Patient is unable to recall his past. - Patient is unaware of identity and usually creates a new one. - Cannot occur in an individual with dissociative identity disorder and cannot be the result of a medical condition or substance abuse. - The condition must be distressing or must impair social or occupation functioning. Answer A: Depersonalization disorder is a response to extreme stress where the patient feels that elements of the external environment are impermanent and may change size or shape and that other people are unreal. Answer B: Dissociative amnesia also is a response to a traumatic event but involves less memory loss. Patients typically do not leave home or invent a new identity. Answer D: Dissociative identity disorder was formerly known as multiple personality disorder and is also a response to trauma with more than one identity inhabiting the same individual. Many individuals with dissociative identity disorder also have dissociative amnesia. Answer E: Post traumatic stress disorder is a response to life-threatening stress that involves re-living the stressful event. DSM-5 criteria for PTSD differ significantly from the DSM-IV criteria. The stressor criterion A is more explicit with regard to events that qualify as "traumatic" experiences. Also, DSM-IV Criterion A2 (subjective reaction) has been eliminated. Whereas there were three major symptom clusters in DSM-IV - re-experiencing, avoidance/ numbing, and arousal - there are now four symptom clusters in DSM-5, because the avoidance / numbing cluster is divided into two distinct clusters; avoidance and persistent negative alterations in cognitions and mood. This latter category, which retains most of the DSM-IV numbing symptoms, also includes new or re-conceptualized symptoms, such as persistent negative emotional states. The final cluster - alterations in arousal and reactivity - retains most of the DSM-IV arousal symptoms. It also includes irritable behavior or angry outbursts and reckless or self-destructive behavior. PTSD is now developmentally sensitive in that diagnostic thresholds have been lowered for children and adolescents. Furthermore, separate criteria have been added for children age 6 or younger. Bottom Line: Dissociative fugue is characterized by an inability to remember most or all of one's past coupled with travel far from home. It is usually triggered by a traumatic event. There is no indicated pharmacologic treatment for dissociative fugue. The approach to treatment usually consists of obtaining a full psychiatric history, which is sometimes aided by use of hypnosis if it remains unclear what the initial psychological stressor was. Once the precipitating event has been identified, psychodynamic psychotherapy is helpful in aiding the patient in dealing with the stressor in a healthier way and is useful long-term to prevent the recurrence of a dissociative fugue. Psychodynamic therapy primarily relies on developing patient insight. Psychodynamic psychotherapy is based upon the idea that childhood experiences, past unresolved conflicts, and previous relationships significantly influence an individual's current situation in life. Adult relationships are understood to be a byproduct of unconscious patterns that begin in childhood. Psychodynamic psychotherapy uncovers the unconscious patterns of interpersonal relationships, conflicts, and desires with the goal of improved functioning. Answers A & B: Cognitive and behavioral therapies can be used individually or in combination as a program of interventions known as cognitive behavioral therapy or CBT. In cognitive therapy, the therapist helps the patient identify and correct distorted, maladaptive beliefs. Behavioral therapy uses thought exercises or real experiences to facilitate symptom reduction and improved functioning. CBT is best for depression, generalized anxiety disorder, post-traumatic stress disorder, panic disorder, eating disorders, and obsessive compulsive disorder, as well as several medical conditions (e.g., insomnia, smoking, low back pain). Answer C: Interpersonal therapy (IPT) addresses interpersonal difficulties that lead to psychological problems. Interpersonal psychotherapy focuses on the individual's interpersonal life in four problem areas: grief over loss, interpersonal disputes, role transitions, and interpersonal skill deficits. Answer D: Motivational interviewing is a type of psychotherapy that is used in primary care and mental health care to encourage patients to change maladaptive behaviors. Bottom Line: Treatment for dissociative fugue is largely supportive, but psychodynamic psychotherapy is helpful help the patient better deal with the stressor and minimizes the risk of recurrence.

An adolescent male presents for sleep and behavioral disturbances that are causing him problems in school and at home. History reveals that his arms often feel weak and limp after awakening from sleep and that it takes a few minutes for him to be able to fully move his arms. His parents note that, at home, if something funny is being discussed and everyone around him start to laugh, he suddenly has transient paralysis of his arms and legs and has to be seated comfortably for few minutes before he regains strength. He also reports seeing images of little wizards and magicians surrounding him when he is about to fall asleep, but he is unsure if this is part of a dream. He notes that he is able to easily fall asleep for short 10-15 minute naps throughout the day. Past medical history is positive only for asthma, and social history reveals a healthy, normal childhood in a supportive home. Physical examination is negative for focal or cranial nerve deficits. An electroencephalogram is performed revealing earlier than usual onset of REM when he falls asleep. The most likely diagnosis is A. absence seizures B. insomnia disorder C. narcolepsy D. nightmare disorder E. somniloquism

The correct answer is: C Narcolepsy falls under the category of dyssomnias. Dyssomnias are primary sleep disorders that result in complaints of either sleeping too little or too much. Narcolepsy is defined by the following tetrad: (1) sleep paralysis that occurs upon falling asleep or waking up; (2) sleep attacks with sleep-onset REM periods (brief, 10-15 minute naps that occur in inappropriate circumstances); (3) cataplexy (a condition of sudden and transient, bilateral weakness or paralysis, usually triggered by strong emotions such as anger, laughter, crying); (4) hypnagogic hallucinations (hallucinations upon falling asleep). Narcolepsy is rare, and onset is usually in the late teens and early twenties, and manifests a chronic course. Research has shown that genetic factors play a role, as 90-100% of Asian and Caucasian individuals with narcolepsy possess the HLA-DR2 and DQw1 phenotypes compared with 20-40% of Asian and Caucasian individuals without narcolepsy. Although narcoleptics have increased daytime sleepiness with decreased nighttime sleep, total sleep time does not increase over a 24-hour period. Answer A: Absence seizures, otherwise known as petit mal seizures, occur primarily during childhood and rarely occur after puberty. They are characterized by the temporary arrest or suspension of consciousness for approximately 5 to 10 seconds. Oftentimes, family members will not notice the absence seizure episodes, but on the other hands, teachers at school may notice that the child has blank, staring episodes for short intervals during the day. When absence seizures are not treated, approximately 70 to 100 episodes can occur in a given day, and such episodes can impair the child's ability to do well in school. If absence seizures are suspected, the physician can usually confirm the diagnosis by asking the child to hyperventilate, as this maneuver will precipitate an episode. Other signs that may be seen include rhythmic blinking and automatisms (spontaneous motor or verbal behaviors). Diagnosis may also be confirmed with the pathognomonic EEG finding of a spike and wave pattern that occurs at a frequency of 3 cycles per second, especially when the child hyperventilates. Answer B: Insomnia comprises of either objective daytime sleepiness and/or subjective feelings of deficient, unrefreshing sleep, and these complaints are not due to a psychiatric or medical condition. Individuals complain of decreased daytime functioning, and are often anxious at bedtime. Cognitive-behavioral therapy and various pharmacologic agents are available for treating insomnia. Answer D: Nightmare disorder comprises of frequent, repeated awakenings from the major sleep period or naps with detailed recall of extremely frightening dreams, usually involving threats to survival, security, or self. These awakenings usually occur during the second half of the sleep period. Upon awakening from the frightening nightmares, the individual immediately becomes alert and oriented. These episodes cause distress in daily, important areas of functioning. Answer E: Somniloquism, known as sleep-talking, arises during NREM (non-REM) sleep periods 1 and 2. It involves vocalizations ranging from simple words and phrases to complete conversations. It is frequently spontaneous, but may be elicited by speaking to the sleeper. Bottom Line: Narcolepsy often starts in adolescence or early twenties, and comprises of a tetrad of: cataplexy, sleep paralysis, sleep-onset REM sleep attacks, and hypnagogic hallucinations. COMBANK Insight : DSM-5 distinguishes narcolepsy - now known to be associated with hypocretin deficiency - from other forms of hypersomnolence (hypersomnolence disorder).

A 27-year-old law intern presents complaining of distressing, repetitive thoughts. History reveals the patient often thinks water may be accidentally running from a faucet left open, and these thoughts prompt her to check any faucet up to fifty times after using it at home, work, or elsewhere, either in the kitchen or in the bathroom. This takes up a significant portion of her time, causing her to be late for work in the morning and to sleep later at night than usual after her shower. She has a supportive family, and reports being shy as a child, but overcame the shyness in high school when she started participating more in after-school activities. She has no known medical problems and past psychiatric history reveals a depressive episode in her early twenties when she failed a final exam. Physical exam is within normal limits. The most likely diagnosis is A. avoidant personality disorder B. generalized anxiety disorder C. obsessive compulsive disorder D.obsessive compulsive personality disorder E. paranoid personality disorder

The correct answer is: C Obsessive compulsive disorder (OCD) comprises of obsessions or compulsions, or both. Obsessions are defined as recurrent and persistent thoughts, impulses, or images that are intrusive and cause marked anxiety. The thoughts are not simply worries about real-life problems. In this case scenario, the patient has obsessive, intrusive thoughts about the water running from the faucet. In OCD, the individual attempts to suppress these thoughts with some other thought or action. The individual realizes that the thoughts are a product of his or her own mind. Compulsions are repetitive behaviors that the individual feels driven to perform in response to an obsession. The compulsions are aimed at reducing distress and are clearly excessive. This patient's intrusive thoughts led her to check the faucet an excessive number of times to make sure it was closed. In OCD, the symptoms interfere with the individual's normal routine and daily social or occupational functioning. Thus, OCD is considered to be ego-dystonic because the individual does not feel in synchrony with his or her symptoms, and recognizes that the obsessions and compulsions are irrational. Individuals with OCD usually seek treatment, unlike individuals with obsessive compulsive personality disorder. Answer A: Individuals with avoidant personality disorder desire to have friends but they avoid making friends or avoid going to social gatherings because of an inferiority complex and excessive discomfort or fear of rejection. They often experience extreme shyness and have low self-esteem. They fear humiliation by others, and often do not open up easily to others. Answer B: Generalized anxiety disorder manifests with excessive anxiety and worry, occurring more days than not for at least six months, about multiple issues or activities. Individuals find it difficult to control their anxiety. The anxiety and worry are associated with three or more of the following six symptoms: restlessness or feeling on edge, being easily fatigued, difficulty concentrating, irritability, muscle tension, disturbance with sleep. The anxiety or worry causes significant distress in social, occupational, or other areas of functioning. In this case scenario, the patient does not have anxiety about multiple issues or aspects of her life, but rather, her anxiety is about her obsessions and compulsions leading to distress. Answer D: Individuals with obsessive-compulsive personality disorder (OCPD) display a pervasive pattern of preoccupation with orderliness, perfection, and mental and interpersonal control, at the expense of flexibility, openness, and efficiency. This personality disorder manifests by early adulthood and presents in a variety of contexts, as indicated by four or more of the following: preoccupation with details, lists, order, organization, or schedules to the extent that the major point of the activity is lost; shows perfectionism that interferes with completion of the task; excessive devotion to work and productivity to the exclusion of leisure activities and friendships; is inflexible about matters of morality or ethics; is unable to discard worn-out objects even when they have no sentimental value; is reluctant to delegate tasks or to work with others unless they submit to exactly his or her way of doing things; adopts a miserly spending style towards both self and others; shows rigidity, ritualism, and stubbornness. OCPD is considered to be ego-syntonic because the individual's feelings and behaviors are in synchrony with his or her self-image, and thus, they do not seek treatment. He or she believes the feelings and behaviors to be a rational part of his or her identity. OCPD is not a sub-syndromal type of OCD, and is not characterized by obsessions and the behaviors in OCPD are not performed in response to obsessions. Answer E: Individuals with paranoid personality disorder have pervasive and persistent mistrust of others including family, friends, and neighbors. They believe that these people are out to get them, and they often start law suits. They are reluctant to confide in others and misjudge the loyalty of others. They pride themselves in being rational, but to others, they appear unemotional and hypervigilant. They bear grudges, and often socially isolate themselves. Bottom Line: OCD comprises of obsessive, intrusive thoughts or compulsive behaviors, or both, that cause marked anxiety and distress. COMBANK Insight : New disorders within the new OCD chapter (now separate from the anxiety chapter) of DSM-5 include hording disorder, excoriation (skin-picking) disorder, substance/medication-induced obsessive-compulsive and related disorder, and obsessive-compulsive and related disorder due to another medical condition.

A 7-year-old male presents with poor school performance. His teachers note him to be inattentive, distractible, and disorganized. The report also includes that he is disobedient, seems not to be listening when spoken to, and does not remain seated at his desk. He does not wait to be called on and blurts out answers. He has difficulty interacting with peers because he butts in to games at recess. His mother describes him as "explosive" at home and complains that he demands attention. Structural examination reveals an anterior radial head dysfunction. The most appropriate initial therapy is A. alprazolam B. haloperidol C. methylphenidate D.osteopathic manipulation of the radial head E. placement in special education classes

The correct answer is: C The child exhibits symptoms of inattention (difficulty sustaining attention in tasks, not seeming to listen when spoken to directly, difficulty organizing tasks, easily distracted by extraneous stimuli), hyperactivity (fidgeting, leaving his seat) and impulsivity (blurting out answers, difficulty awaiting turn, intruding on others) consistent with a diagnosis of attention-deficit/hyperactivity disorder. Diagnosis requires six or more symptoms of inattention and of hyperactivity-impulsivity that are persistent for six months or more and have caused significant impairment in two or more settings (school, work, home). At least some symptoms must be present before age 12 years and do not occur exclusively during the course of another psychiatric illness. First line treatment is with CNS stimulant medication including methylphenidate, dextroamphetamine and mixed amphetamine salts, which act as dopamine agonists. Their precise mechanisms of action in ADHD is unknown but are considered first line treatment due to their efficacy and tolerability. Common side effects include headache, insomnia and decreased appetite. Answer A: Alprazolam is a benzodiazepine sedative-hypnotic that binds to the GABA receptor. It is indicated in the treatment of anxiety disorder and its use is limited by risk for dependence. There is no indication for use of a benzodiazepine for treatment of the above symptoms, which represent ADHD due to the prominent symptoms of inattention, hyperactivity and impulsivity. The appropriate pharmacotherapy is a CNS stimulant. Answer B: Haloperidol is a conventional antipsychotic that blocks dopamine receptors to reduce positive symptoms of psychosis. While medications like haloperidol are sometimes used emergently to reduce combative, explosive and hyperactive behaviors it is not indicated in this setting as the symptoms are consistent with attention-deficit/hyperactivity disorder and not a primary psychotic disorder. The appropriate treatment is a CNS stimulant medication. Psychosis would be considered as a cause of the explosive and impulsive behavior only if it was accompanied by delusions, hallucinations, disorganized speech and/or negative symptoms such as flat affect. Answer D: The question stem identifies a radial head dysfunction, however this physical exam finding bears no relation to the presenting symptoms, which are causing impairment at school and home. Therefore, the best initial therapy is treatment of the patient's attention-deficit/hyperactivity disorder with a CNS stimulant medication. Once this is initiated it is appropriate to consider osteopathic manipulation to address the somatic dysfunction. Answer E: The patient presents with a chief complaint of poor school performance and symptoms consistent with a diagnosis of attention-deficit/hyperactivity disorder. There is no evidence of a learning disorder based on the information provided, however treatment of attention-deficit/hyperactivity disorder should always include evaluation and treatment of any co-existing learning disorders if present. There is no indication for special education placement at this time. Treatment consists of initiation of a CNS stimulant medication. Psychosocial interventions such as social skills groups and behavioral interventions at school and home are often added. Bottom Line: Attention-deficit/hyperactivity disorder is the most common behavioral disorder among American children. Symptoms of inattention, impulsivity, hyperactivity and emotional lability appear before age 7 and cause persistent impairment in at least two settings. The first line treatment is pharmacotherapy with CNS stimulants. Psychosocial interventions are often added and evaluation for coexisting learning disorders is important. COMBANK Insight : Several changes have been made to the diagnostic criteria for ADHD in DSM-5. Examples have been added to the criterion items to facilitate application across the life span; the age at onset description has been changed (From "some hyperactive-impulsive or inattentive symptoms that caused impairment were present before age 7 years" to "Several inattentive or hyperactive-impulse symptoms were present prior to age 12"); subtypes have been replaced with presentation specifiers that map directly to the prior subtypes; a comorbid diagnosis with autism spectrum disorder is now allowed; and a symptom threshold change has been made for adults, to reflect the substantial evidence of clinically significant ADHD impairment, with the cutoff for ADHD of five symptoms, instead of six required for younger persons, both for inattention and for hyperactivity and impulsivity.

A male recently learned that a lot of people will be laid off at work. His family is having financial difficulties and his wife had to take a second night job, so he's at home taking care of the children now, one of which is sick. He complains of daytime fatigue yet wakes up in the early morning hours and is unable to fall back to sleep. He believes that he is incompetent and that is why his family is experiencing financial difficulties, ignoring the fact that his treatment of his daughter's illness was an unexpected expense. He often feels as though his situation is hopeless but denies desire to die or end his life, rather expressing desire to support his family. He is reluctant to take medication as he feels it would be a sign of weakness. The most appropriate management is A. alprazolam B. bupropion C. psychological counseling D.psychological testing E. zolpidem

The correct answer is: C The patient is experiencing depressive symptoms (sleep disturbance with early morning awakenings, fatigue, hopelessness, worthlessness and feelings of excessive guilt in response to a psychosocial stressor (financial difficulties) consistent with an adjustment disorder. At this time he does not meet the criteria for major depressive disorder since there is no clear complaint of depressed mood or anhedonia - one of which, in addition to five other depressive symptoms (depressed mood or irritability, decreased interest or pleasure in most activities, change in appetite, sleep disturbance, psychomotor agitation/retardation, loss of energy, guilt/worthlessness, diminished concentration, suicidality) must be present for a diagnosis to be made. For this patient the first step in treatment is psychotherapy - in particular cognitive behavioral therapy - to increase insight, address the patients helpless core beliefs identified in the question stem which contribute to his depressive symptoms and modify behavior. Answer A: Alprazolam is a short acting benzodiazepine which enhances the inhibitory effects of GABA and is approved for the treatment of panic disorder and generalized anxiety disorder. It is also used off-label for the treatment of mania/agitation as an adjunct to a mood stabilizer as well as for catatonia and insomnia. The patient described in the question stem is suffering from an adjustment disorder with depressed mood and there is no mention of anxiety symptoms. He is experiencing terminal insomnia which is characteristic of a melancholic type of depression. It is important to remember that insomnia is often a symptom of some other primary psychiatric or physical illness and if so the first line treatment should address that primary illness - in this case the patient should receive psychological counseling to deal with the stressors that precipitated his depression symptoms which include terminal insomnia. Answer B: Psychotropic medication is indicated for patients experiencing symptoms of major depression that significantly impair daily functioning or serious symptoms of depression including psychosis, catatonia or suicidal thoughts. Antidepressants can decrease the length and severity of major depressive episodes - selective serotonin reuptake inhibitors (SSRIs) are first line treatment due to tolerability. Bupropion's mechanism of action is norepinephrine dopamine reuptake inhibitor and is generally a second-line antidepressant for patients who fail SSRIs or when there are concerns about weight gain and sexual dysfunction as these side effects are not associated with bupropion versus SSRIs. This patient is experiencing symptoms consistent with adjustment disorder rather than major depression since his symptoms were triggered by a psychosocial stressor and do not meet full criteria for a major depressive episode (which requires the presence of either depressed mood or anhedonia to be present in addition to associated symptoms like insomnia, fatigue and guilt). The most effective treatment for adjustment disorders is supportive psychotherapy, although pharmacotherapy may be indicated in some instances for treatment of associated symptoms like insomnia. Answer D: Common uses of psychological testing include aiding in the differential diagnosis of psychiatric symptoms (particularly when the diagnosis is not clear), assessment of cognitive impairment and intelligence and to help establish a treatment plan for patients who have failed standard treatment. The diagnosis of an adjustment disorder is clear in the above case and psychotherapy is first line treatment for that condition. Answer E: Zolpidem is a non-benzodiazepine hypnotic approved for the short-term treatment of insomnia. However, insomnia is often a manifestation of other psychiatric or medical illness which warrants investigation and separate treatment. There is a risk of dependency and tolerance with hypnotic agents. In treating insomnia it is important to evaluate and treat underlying illness which can contribute to the sleep disturbance, teach sleep hygiene techniques and use medication as a last resort and only for a short time. This patient is experiencing symptoms of depression, which includes terminal insomnia, and so initial treatment should focus on the stressors that led to his adjustment disorder with depressed mood. Bottom Line: Psychotherapy to increase insight and modify behavior is the first line treatment for adjustment disorders.

A 16-year-old female is brought to the outpatient clinic by her parents for dizziness and frequent cramping. History reveals the symptoms present primarily during physical exertion and may be associated with vomiting or dry heaving. Her symptoms are currently interfering with her school work causing her to stay home from school on multiple occasions. Her mother expresses concern that her symptoms are due to the family's recent move to a different high school district and feels that her symptoms will subside once she becomes acclimated to this new environment. Vitals are within normal limits and reveal her to be in the 86th percentile for height and weight for her age and gender. Physical examination reveals facial acne, erosion of the lingual surface of the teeth and bilateral parotid enlargement. Mental examination reveals a well dressed, neat appearance, poor eye contact, linear thought processes, and mild psychomotor agitation. Question 1 of 3 in this set The most likely diagnosis is A. adjustment disorder B. binge-eating disorder C. bulimia nervosa D.Kleine-Levin syndrome E. major depressive disorder with atypical features

The correct answer is: C This patient is demonstrating the classic signs and symptoms of bulimia nervosa. Bulimia nervosa is typically seen in young females, who fall within the average to slightly above average weight range, who present with dizziness, muscle weakness, and cramping during exercises. On physical examination, signs of volume depletion may be noted. Perimolysis (erosion of the tooth enamel) and enlargement of the parotid glands and tonsils (secondary to vomiting) carry a high index of suspicion for bulimia over other conditions. Bulimia nervosa is managed with selective serotonin reuptake inhibitors as well as cognitive behavioral therapy and/or psychotherapy. Answer A: Adjustment disorder is the development of emotional or behavioral symptoms in response to an identifiable stressor(s) occurring within 3 months of the onset of the stressor(s). These symptoms or behaviors are clinically significant as evidenced by either of the following: (1) marked distress that is in excess of what would be expected from exposure to the stressor; (2) significant impairment in social or occupational (academic) functioning. The stress-related disturbance does not meet criteria for another Axis I diagnosis, and is not merely an exacerbation of a personality disorder. Once the stressor has terminated, the symptoms do not persist for more than an additional six months. In this case scenario, the patient may be adjusting to her new environment, but the diagnosis of adjustment disorder is unlikely because of her physical examination findings. Answer B: Some individuals binge eat but do not engage in regular inappropriate compensatory behaviors such as purging or using laxatives. In these cases, the diagnosis of binge-eating disorder should be considered. Answer D: Kleine-Levin syndrome is a rare sleep disorder characterized by recurrent hypersomnia, hyperphagia, and hypersexuality. It predominately affects adolescent males of Jewish origin. Answer E: Overeating is common in major depressive disorder, but individuals with this disorder do not engage in inappropriate compensatory behaviors (e.g., purging, using laxatives) and do not exhibit the excessive concern with body shape and weight characteristics. Bottom Line: The classic presentation of bulimia nervosa is a female of average to above average weight, who presents with dizziness, dehydration, muscle weakness, possible parotid gland enlargement, and erosion of the tooth enamel on physical examination. Laxative abuse is very common in patients with bulimia nervosa and questions should be directed toward their use. Common laxatives include bisacodyl, cascara, senna, and high fiber supplements. Abuse of stimulant laxatives can lead to severe constipation, due to their damaging effects on the myenteric plexus. Caffeine, pseudoephedrine, phenylpropanolamine, and synthroid may also be used to try to increase metabolic rate and calorie loss. Always question patients with bulimia regarding the use of natural herbal substances as these agents can have profound effects on heart rate and blood pressure. During physical examination of patients suspected of having eating disorders, it is always important to inspect the hands. Patients with bulimia nervosa may have callus formation on the dorsal surface of the index finger and long finger of the hand from repetitive contact with the teeth during self-induced vomiting. This is referred to as the " Russell sign". Other signs of bulimia nervosa include bilateral parotid enlargement, cutaneous manifestations such as sudden, diffuse hair loss, acne, dry skin, and nail dystrophy. Other signs may include bradycardia, tachycardia, hypothermia, or hypotension, as well as edema.

A 25-year-old male presents to the emergency department for odd behavior. When you approach him, he anxiously asks you why there are so many monkeys running around the emergency department. Vital signs reveal a blood pressure of 169/101 mmHg, a heart rate of 112/min, and respiratory rate of 21/min. His girlfriend tells you that he is a chronic drug abuser who recently abruptly stopped his drug habits 2 days ago in attempt to become sober. She says she does not know what drug he uses as he has abused various drugs in the past. Which of the following is the mechanism of action of the most likely etiologic agent? A. decreases glutamate receptor conductance B. increases canabinoid receptor conductance C. increases gama-aminobutyric acid conductance D.increases nicotinic receptor conductance E. increases opioid receptor conductance

The correct answer is: C This patient is presenting with a classic case of acute benzodiazepine withdrawal. Benzodiazepines work by modifying gamma-aminobutyric acid (GABA) receptors to increase conductance of this inhibitory channel. Benzodiazepines are commonly used for the treatment of seizure control, anxiety, alcohol withdrawal, insomnia, control of drug associated agitation, muscle relaxants, and pre-anesthetic agents. Because of their common use and availability, they have a propensity for abuse. There is a synergistic effect when combined with alcohol use. Symptoms of intoxication include amnesia, ataxia, somnolence, respiratory depression. Flumazenil can be used as an antidote for acute intoxication. The use of benzodiazepine in pregnancy can increase risk for cleft lip or palate, lower muscle tone, and withdrawal symptoms in the developing fetus. As with most withdrawal syndromes, symptoms are similar to what benzodiazepines (and similar to alcohol withdrawal) are used to treat including rebound anxiety, seizures, auditory or visual hallucinations, insomnia, including death. Hypertension and tachycardia are a hallmark finding. Withdrawal symptoms can last 6 months up to 12 months. Answer A: A glutamate receptor antagonist, such as phencyclidine (PCP), is an abused drug that causes symptoms of violence, belligerence, psychosis, impulsiveness, psychomotor agitation, fever, tachycardia, vertical/horizontal nystagmus, hypertension, impaired judgment, ataxia, seizures, delirium. Withdrawal symptoms include similar symptoms of intoxication with sudden onset of severe, random violence. Answer B: A cannabinoid receptor agonist such as marijuana causes intoxication symptoms including euphoria, impaired judgment, increased appetite, dry mouth, conjunctival injection, hallucinations, paranoia. Withdrawal symptoms include depressed mood, lack of appetite, and anger. Answer D: Nicotine is a nicotinic receptor agonist that causes relaxation, sharpness, calmness, and alertness. Nicotine withdrawal symptoms include nausea, insomnia, anxiety, depression, and irritability. Answer E: An opioid agonist, such as heroin, is an abused drug that causes apathy, central nervous system depression, constipation, pupillary constriction, and respiratory depression. Acute intoxication can be treated by use of naloxone to block opioid receptors. Withdrawal symptoms include insomnia, anorexia, fever, dilated pupils, rhinorrhea, vomiting, diarrhea. Bottom Line: Benzodiazepines work by modifying gamma-aminobutyric acid (GABA) receptors to increase conductance of this inhibitory channel.

A 29-year-old female presents with her sister for a follow-up evaluation. She has been coming to the clinic for three years, but she recently has been forgetting to take her scheduled risperidone. Subsequently, she has been hearing voices telling her about the green man who will help her fight the evil witch. She also reports seeing little animals walking around. She believes that she is a fairy and that the green man and these animals are from a magical land where she will get to go and they will fight the witch confidently. Her sister states that she has been eating well and has no known medical problems. Mental examination reveals that she exhibits some thought blocking and is not completely oriented with reality. She appears neat in appearance, but has a reduced affect with fair eye contact. The most likely diagnosis is A. schizoaffective disorder B. schizoid personality disorder C. schizophrenia D.schizophreniform disorder E. schizotypal personality disorder

The correct answer is: C This patient most likely has Schizophrenia. The (abridged) DSM-5 diagnostic criteria include the following: A. Either delusions, hallucinations, disorganized speech, grossly disorganized or catatonic behavior, or negative symptoms. B. Significant functional or interpersonal disturbance. C. Persistent for at least 6 months with at least 1 month of Criteria A symptoms. D. Schizoaffective disorder and depressive or bipolar disorders with psychotic features have been ruled out. Schizophrenia usually presents at a younger age in males (ages 15-24) compared to females (ages 25-34). Females also have a better prognosis with regards to treatment compared to males. Poor prognosis in schizophrenia is associated with the following features: early age of onset, negative symptoms, poor pre-morbid functioning, family history of schizophrenia, and disorganized features. Answer A: It could be schizoaffective disorder, but there is no mention of features of a mood disorder for this patient. Schizoaffective disorder has been reconceptualized in DSM-5 as a longitudinal instead of a cross-sectional diagnosis - more comparable to schizophrenia, bipolar disorder, and major depressive disorder, which are bridged by this condition - and requires that a major mood episode be present for a majority of the total disorders duration after Criterion A has been met. Answer B: Those with schizoid personality disorder have a pervasive pattern of detachment from social relationships and a restricted range of expression of emotions in interpersonal settings with four additional features (e.g., chooses solidarity, little sexual interests, few activities, lacks close friends). Answer D: Schizophreniform disorder fulfills the same above-mentioned Criterion A symptoms of schizophrenia except for the duration of time criteria. Schizophreniform disorder symptoms occur for more than one month but last less than six months, whereas, brief psychotic disorder symptoms last for one month or less. On the other hand, schizophrenia is diagnosed when the symptoms occur for more than six months. Two-thirds of individuals with schizophreniform disorder progress to being diagnosed with schizophrenia. In this case scenario, it appears that the patient has been having the psychiatric condition for at least three or more years. Answer E: Those with schizotypal personality disorder have a pervasive pattern of social and interpersonal deficits marked by acute discomfort with, and reduced capacity for, close relationships as well as by cognitive or perceptual distortions and eccentricities of behavior, beginning by early adulthood and present in a variety of contexts, as indicated by five or more additional criteria (e.g., odd beliefs/thinking, ideas of reference, unusual perceptual experiences, paranoid ideations). Bottom Line: Undifferentiated type schizophrenia meets the Criterion A symptoms of schizophrenia but does not meet the criteria for paranoid, disorganized, or catatonic types of schizophrenia. COMBANK Insight : Two changes were made to the DSM-5 Criterion A for schizophrenia: 1) the elimination of the specific attribution of bizarre delusions and Schneiderian first-rank auditory hallucinations (e.g., two or more voices conversing), leading to the requirement of at least two Criterion A symptoms for any diagnosis of schizophrenia, and 2) the addition of the requirement that at least one of the Criterion A symptoms must be delusions, hallucinations, or disorganized speech. The DSM-IV subtypes of schizophrenia were eliminated due to their limited diagnostic stability, low reliability, and poor validity. Instead, a dimensional approach to rating severity for the core symptoms of schizophrenia is included.

A 20-year-old male presents with restlessness. History reveals episodes of insomnia, hypersexuality and racing thoughts followed by depression with suicidal ideation. The appropriate pharmacological maintenance therapy is A. aripiprazole monotherapy B. lithium and amitriptyline C. lithium and clonazepam D. lithium and quetiapine E.selective serotonin reuptake inhibitor monotherapy

The correct answer is: D Bipolar disorder type I is characterized by at least 1 episode of mania, followed by depression. Mania is defined as persistently elevated or irritable mood with increased energy or activity lasting at least 1 week and present most days. Episodes of mood disturbance are accompanied by at least 3 of the following: inflated self-esteem or grandiosity, decreased need for sleep, increased or pressure speech, racing thoughts or flights of ideas, distractibility, psychomotor agitation and excessive involvement in high risk activities. The mood disturbance is sufficiently severe to cause marked impairment in social or occupational functioning and is not attributable to the effects of a substance or medical condition. Quetiapine plus lithium or valproate is superior to monotherapy for maintenance treatment and can be used for acute mania, bipolar depression and maintenance therapy. Quetiapine is an atypical antipyschotic that is an antagonist at the dopamine and serotonin receptors. Answer A: Aripiprazole, a second generation atypical antipsychotic and partial dopamine agonist, can be used in acute mania but is not indicated in bipolar depression or maintenance therapy. Answer B: Tricyclic antidepressants should be avoided because of possibility of inducing symptoms of rapid cycling. When a patient with bipolar disorder becomes depressed, a selective serotonin reuptake inhibitor or bupropion is recommended in addition to maintenance therapy. Answer C: Benzodiazepines may be used for short-term treatment of anxiety and insomnia, but should be avoided for long-term maintenance therapy due to risk of physical dependence and abuse. Answer E: Selective serotonin reuptake inhibitor (SSRI) monotherapy is contraindicated in bipolar disorder type I and during episodes of mania due to possible worsening of mania. An SSRI or bupropion may be used in addition to a mood stabilizer during episodes of depression to augment maintenance therapy. Bottom Line: High quality evidence supports the use of lithium, valproate, quetiapine and olanzapine for maintenance therapy in patients with bipolar disorders. Quetiapine plus lithium or valproate is superior to monotherapy for maintenance treatment and can be used for acute mania, bipolar depression and maintenance therapy.

A 43-year-old female presents with the inability to sleep at night. She complains of difficulty falling asleep due to rumination and notes, "I can't shut my mind off." She sleeps about three hours per night and does not awake in the morning feeling refreshed. She fears she is not a good enough employee, mother, or wife and often contemplates about the household bills, her children's health, and whether she may be fired from her job. History reveals a one-year history of fatigue, muscle tension, and a subjective feeling of restlessness. Her daily medications include levothyroxine and cyanocobalamin, and her most recent labs drawn last week reveal a TSH of 2.2 mIU/L and a vitamin B level that is within normal limits. She is a non-smoker and drinks six ounces of coffee once per week. The most likely diagnosis is A. adjustment disorder with anxious features B. anxiety due to a general medical condition C.caffeine-induced anxiety disorder D. generalized anxiety disorder E. insomnia disorder

The correct answer is: D The question stem presents a case of insomnia. Insomnia can be characterized as a primary sleep disorder or as a secondary sleep disorder resulting from another Axis I mental disorder, a medical condition or substance use. In addition to sleep disturbance, the case presents a female patient with anxiety that she has difficulty controlling, poor concentrating, fatigue, muscle tension and restlessness. These symptoms are impacting her job performance and have been present for one year. This constellation of symptoms is most consistent with a diagnosis of generalized anxiety disorder and her sleep disturbance is part of this condition - a secondary sleep disorder. Generalized anxiety disorder is defined as excessive worry that is difficult to control and which causes significant distress and/or impairment. It must be present for at least 6 months and associated with at least three of the following symptoms: restlessness or psychomotor agitation, fatigue, poor concentration, irritability, muscle tension and sleep disturbance. While thyroid disease (hyperthyroidism) and vitamin B12 deficiency can cause anxiety symptoms, the patient's recent TSH and vitamin B12 level are both within normal limits. Answer A: Adjustment disorder with anxiety is characterized by anxiety symptoms that arise up to three months after a stressful life event and resolve within six months of it. While the question stem describes a female patient having difficulty at work, it is her anxiety and insomnia that interfere with her work performance rather than an acute stressor at work resulting in these symptoms. The question stem clearly describes a woman with excessive worry over various aspects of her life (work, finances, family) which she finds difficult to control and which creates clinically significant somatic symptoms such as muscle tension as well as insomnia and irritability, thus making generalized anxiety disorder the best answer choice. Answer B: This patient has a normal TSH and B12 level without other signs of medical conditions that could be contributing to anxiety at this time. Medical conditions that may be involved in this disorder are hyperthyroidism, hypothyroidism, hypoglycemia, and hyperadrenocorticism. Heart related problems may also underlie this disorder. Some of these conditions are congestive heart failure and arrhythmia. Breathing problems such as COPD, pneumonia, and hyperventilation also can initiate anxiety. Answer C: Both caffeine intoxication and withdrawal can cause anxiety. The question stem mentions that the patient drinks coffee, however her caffeine consumption is minimal (six ounces of coffee once per week) and would not account for her significant and persistent anxiety symptoms and it is not enough caffeine nor frequent enough of use to induce withdrawal anxiety on the days she does not consume it. Answer E: Primary insomnia is a sleep disturbance that is not better accounted for by another Axis I mental disorder, medical condition or substance use. It includes difficulty with initiating or maintaining sleep that results in daytime drowsiness or difficulty completing tasks. It is treated with sleep hygiene measures first and possibly short-term pharmacotherapy if those measures fail. While the patient in the question stem is complaining of difficulty initiating sleep which is resulting in daytime fatigue and difficulty completing tasks at work, her insomnia is present in the context of excessive worry and other significant anxiety symptoms (irritability, difficulty concentrating, muscle tension) which in total meet criteria for generalized anxiety disorder. Thus, her insomnia can be characterized as secondary rather than primary. Bottom Line: Insomnia can be either primary or secondary. With secondary insomnia, the sleep disorder is better accounted for by another Axis I mental disorder, a medical condition or substance use. Mood and anxiety disorders are commonly associated with sleep disturbance. It is important to differentiate primary insomnia from secondary insomnia as their treatments differ. While short-term use of sedative-hypnotics can be useful in the treatment of primary insomnia, the underlying condition must first be treated in cases of secondary insomnia.

Seven weeks after being involved in a fatal motor vehicle accident, where she witnessed the death of her father and uncle, a 26-year-old woman presents with intermittent nightmares and flashbacks of the accident. These symptoms started within 3-4 days of the accident. Due to her new fear of motor vehicles, she refuses to ride in them. She had taken leave from work, and feels like she has lost interest in working, and has still not gone back. Her mother states that, for the past couple of months, her daughter has been avoiding going outside, has become easily irritable, and becomes easily startled if she hears loud noises. Immediately after the accident the patient states that she experienced a feeling of numbness, as if she had just watched the event on television rather than actually experiencing it. She has not been able to sleep well due to nightmares reliving the accident. The most likely diagnosis is A. acute stress disorder B. adjustment disorder C.agoraphobia D. post-traumatic stress disorder E. specific phobia

The correct answer is: D DSM-5 criteria for posttraumatic stress disorder (PTSD) differ significantly from the DSM-IV criteria. The stressor criterion A is more explicit with regard to events that qualify as "traumatic" experiences. Also, DSM-IV Criterion A2 (subjective reaction) has been eliminated. Whereas there were three major symptom clusters in DSM-IV - re-experiencing, avoidance/ numbing, and arousal - there are now four symptom clusters in DSM-5, because the avoidance / numbing cluster is divided into two distinct clusters; avoidance and persistent negative alterations in cognitions and mood. This latter category, which retains most of the DSM-IV numbing symptoms, also includes new or re-conceptualized symptoms, such as persistent negative emotional states. The final cluster - alterations in arousal and reactivity - retains most of the DSM-IV arousal symptoms. It also includes irritable behavior or angry outbursts and reckless or self-destructive behavior. PTSD is now developmentally sensitive in that diagnostic thresholds have been lowered for children and adolescents. Furthermore, separate criteria have been added for children age 6 or younger. The primary difference between acute stress disorder (ASD) and PTSD is the duration of the symptoms and the former's emphasis on dissociative reactions to the trauma. ASD refers to symptoms manifested during the period from 2 days to 4 weeks posttrauma, whereas PTSD can only be diagnosed from 4 weeks. Answer A: Acute stress disorder involves exposure to a traumatic event and a subsequent response of intense fear, helplessness, or horror. The traumatic event involves the threat of death to self or others, or a threat to the physical integrity of self or others. Symptoms occur within 4 weeks of the traumatic event and persist for at least 2 days and up to less than 4 weeks. Either while experiencing or after experiencing the traumatic event, the individual has three or more of the following dissociative symptoms: a sense of numbing or detachment, a reduction in awareness of surroundings, derealization, depersonalization, dissociative amnesia. The traumatic event is re-experienced in the form of dreams or images, marked anxiety or hypervigilance exists, and marked avoidance of stimuli occurs. The impairments cause significant distress in social, occupational, or other important areas of functioning. In this case, the patient was exposed to a traumatic event that threatened the life or physical integrity of self or others, but the diagnosis is post-traumatic stress disorder because the symptoms have been occurring for more than one month's duration. Answer B: Adjustment disorder occurs with the development of emotional or behavioral symptoms in response to an identifiable stressor(s) occurring within 3 months of the onset of the stressor(s). These symptoms or behaviors are clinically significant as evidenced by either of the following: (1) marked distress that is in excess of what would be expected from exposure to the stressor; (2) significant impairment in social or occupational (academic) functioning. Once the stressor has terminated, the symptoms do not persist for more than an additional six months. Answer C: Agoraphobia is when individuals experience anxiety about being in places or situations from which escape might be difficult or embarrassing or a situation in which help may not be available in case the individual experiences a panic attack or panic-like symptoms. Agoraphobic fears typically involve being afraid to go outside of the home alone, being a crowded place or standing in a line, traveling on a bus or a train, or being on a bridge. In this question, the patient is having anxiety of driving or riding in vehicles due being traumatized from the accident. She does not present with fears of being in crowded places. Answer E: This patient would meet the criteria for a specific phobia if the symptom pattern did not develop in response to a stressor that met PTSD criterion A (e.g., spouse leaving, being fired, etc.). Bottom Line: PTSD describes syndrome of post-traumatic emotional distress with avoidance, hypervigilance, detachment, nightmares, flashbacks, etc. at a moderate to severe level which lasts for more than a month. COMBANK Insight : The duration of disease presentation is very important for diagnosing psychiatric disorders. Sometimes it is the only duration of symptoms that leads you to the correct answer.

A 58-year-old male presents with paranoid delusions of being poisoned by his girlfriend and auditory hallucinations commanding him to kill her. History reveals multiple previous inpatient psychiatric hospitalizations and treatment with various psychotropic medications over the past thirty years. Mental status examination reveals a disheveled male with psychomotor agitation, flat affect, and internal preoccupation. When asked why he was brought to the hospital he speaks at length about how the police were rough with him and about the ambulance ride over, but he never articulates the problem for which he is being evaluated. This is representative of A. circumstantiality B. flight of ideas C. loose associations D. tangentiality E. word salad

The correct answer is: D In tangentiality the responses given to questions never reach the goal of answering the question, which is in contrast to circumstantiality where extra and unnecessary details are added but a question is eventually answered. Tangentiality describes the speech of patients who talk past the point. In contrast to loosening of associations, patients with tangential speech have a clearly recognizable link between their ideas. Answer A: In circumstantiality the patient includes extra and unnecessary details to their responses before ultimately answering a question. A clearly recognizable link exists between the ideas expressed although the patient takes a longer time to get to the point. This is in contrast to tangentiality where the patient never reaches the goal of answering the question. Answer B: In flight of ideas the patient speaks rapidly and abruptly changes topics based on links or similarities between associations. Flight of ideas is commonly seen in mania and is often accompanied by pressured speech. Tangentiality is not characterized by these rapid and abrupt shifts in topic but rather by simple inclusion of extra details and ideas which keeps them from answering the question. Answer C: Looseness of associations refers to disruption of the logical connection between the thoughts expressed sequentially, which the patient is unaware of and which makes the thoughts difficult to understand for the interviewer. Loose associations are often a feature of mania. In tangentiality, extra ideas are included to the point of straying from answering a question but logical connections between thoughts remain. Answer E: Word salad is incoherent and illogical speech that lacks meaningful connection between words. It represents an extreme form of loose associations and is commonly seen in schizophrenia. The other answer choices describe thought processes that include some degree of connection between the words and ideas being expressed. Bottom Line: Clinical assessment of the psychiatric patient includes obtaining a comprehensive history and performing mental status examination, which describes the patient's appearance, speech, actions and thoughts. Description of thought is divided into thought processes (or form of thinking) and thought content. Description of thought processes focuses on the way the patient puts together ideas and associations while thought content refers to what they are thinking about including ideas, beliefs, delusions, preoccupations and obsessions.

An 18-year-old male presents to the emergency department with uncontrollable agitation and auditory hallucinations. His friends had to hold him down due to his aggressive state and mentioned that he had "a little marijuana" at a party before demonstrating his uncontrollable behavior. Vitals reveal a heart rate of 130/min. Physical examination displays that he is only oriented to name. His eyes exhibit both vertical and horizontal nystagmus. Which of the following drugs was most likely mixed in the cocktail with the marijuana? A. cocaine B.lysergic acid diethylamide C. methamphetamine D. phencyclidine hydrochloride E. salvia divinorum

The correct answer is: D Phencyclidine hydrochloride (PCP) causes perceptual alterations, mood changes, psychotic symptoms of delusions/hallucinations/paranoia, violence, impulsiveness, fever, tachycardia, horizontal and vertical nystagmus, ataxia, and delirium. Some people experience sensory detachment. Benzodiazapines are typically used to treat severe symptoms. Answer A: Cocaine intoxication (during or immediately after cocaine use) causes clinically significant psychological and behavioral changes. Psychological symptoms include: euphoria, interpersonal sensitivity, anxiety, anger, impaired social and occupational functioning, hypervigilance, and poor judgment. Physiological symptoms include: tachycardia, pupillary dilation, hypertension or hypotension, nausea, vomiting, weight loss, muscle weakness, respiratory depression, possible seizures, confusion, chest pain, arrhythmias, and dystonia. Cocaine withdrawal (within hours or several days after stopping heavy and prolonged use) is manifest by dysphoric mood, fatigue, vivid dreams, sleeping problems, psychomotor agitation, increased appetite. Answer B: LSD intoxication would most likely cause anxiety, depression, pupillary dilation, delusions, visual hallucinations, and flashbacks. The individual develops synesthesias (hearing colors and seeing sounds) and panic symptoms. Answer C: Intoxication with methamphetamines would most likely cause fatigue, apprehension, trouble with concentration, hypertension, pupillary dilation, fever, euphoria, possible symptoms of psychosis, restlessness, irritability, and prolonged wakefulness. Answer E: Salvia divinorum causes hallucinations, euphoria, and perceptual distortions that last for 1-2 hours, but less when it is smoked. Sympathomimetic symptoms are mild, and no deaths or cases of severe toxicity have been reported. Bottom Line: PCP causes psychosis, violent behavior, and is associated with horizontal and vertical nystagmus

A 25-year-old graduate student presents to the university counseling center because she is having trouble maintaining her grades and has been put on academic probation. She says that she knows the material, but her professors are unreasonable and refuse to see that she has mastered their subjects. She failed two classes for not turning in her final projects on time, and the professors refused to grant her extensions, which they did for several other students. On further questioning, she admits that she did not ask for an extension until a week after the projects were due but says: "the professors are just being unreasonable. They knew I would hand it in, and they should have given me a break." She does not smoke and drinks socially on the weekends. Mental status examination reveals no evidence of affective disorder, delusions, hallucinations, or cognitive difficulties. This patient is most likely using which of the following defense mechanisms? A.denial B. displacement C. projection D. rationalization E. suppression

The correct answer is: D Psychological defense mechanisms are unconscious responses, typically employed in response to various life stressors in order to decrease anxiety. They are classified into three types: mature, neurotic, and immature. Mature defense mechanisms are considered as fairly healthy means of coping with reality; these are typically used by adults. Neurotic defense mechanisms are usually seen in obsessive-compulsive patients and adults dealing with large amounts of stress; they usually provide short-term relief, but often result in more complications later on. Immature defense mechanisms are most commonly seen in children and adolescents; they are the most primitive defense mechanisms and result in socially inappropriate behavior. Rationalization is a neurotic defense mechanism whereby the patient unconsciously creates explanations for a situation in order to make the outcome (or their behavior) more acceptable. In this case the student is blaming her professors for not recognizing her work rather than taking responsibility for not having met course requirements. Answer A: Denial is a primitive defense mechanism in which the patient denies the existence of a problem or fact because it is too painful for them to acknowledge it. Answer B: Displacement is a neurotic defense mechanism in which unacceptable emotions in one situation are expressed in a different situation where they are more tolerable to the patient. Answer C: Projection is the attribution of thoughts or feelings to another person who does not, in fact, have those thoughts or feelings. Answer E: Suppression is a mature defense mechanism in which the subject consciously ignores unacceptable impulses or emotions in order to achieve a desired outcome. Bottom Line: Rationalization is a neurotic defense mechanism whereby the patient unconsciously creates explanations for a situation in order to make the outcome (or their behavior) more acceptable.

A 26-month-old female presents with her mother with complaints of developmental speech delay. History reveals the patient has not yet spoken a clear word. The mother reports that the girl does not play with her 4-year-old sister either. The mother reports that the patient had a normal, term birth, and she denies any abnormalities in the first two years of her life stating the child lives in a happy, supportive home. According to the patient's chart, she had a seemingly normal development with a head circumference at the 50th percentile up to 18 months of age. Today, the patient's head growth appears to have decelerated. Physical examination reveals a wobbly gait, poor response to verbal commands, and repetitive wringing of hands. Routine laboratory studies are obtained revealing a normal complete blood count, basic metabolic panel, and thyroid panel. The most likely diagnosis is A. Lennox-Gastaut syndrome B. Patau syndrome C.Prader-Willi syndrome D. Rett's disorder E. Smith-Lemli-Opitz syndrome

The correct answer is: D Rett's disorder is an X-linked dominant disorder due to a MECP2 gene mutation on chromosome Xq28 that occurs almost exclusively in females. In Rett's disorder, individuals have normal prenatal, perinatal, and psychomotor development through the first five months after birth. Head circumference is also within normal limits at birth. There is regression of development after the period of normal development. Rett's disorder encompasses onset of all of the following: (1) deceleration of head growth between ages 5 and 48 months (2) loss of previously acquired purposeful hand skills between ages 5 and 30 months with the subsequent development of stereotyped hand movements like hand wringing, hand washing, etc. (3) loss of social engagement early in the course (4) appearance of poorly coordinated gait or trunk movements (5) severely impaired receptive and expressive language development with severe psychomotor retardation. Cases have been found to have episodic over-breathing, unprovoked laughter, and progressive impaired mobility. This case scenario describes the patient as having wobbly gait, hand wringing movements, deceleration of head growth, loss of social engagement with sister, and impaired receptive and expressive language skills. Rett's disorder is often associated with severe or profound mental retardation. The disease course is progressive and individuals with Rett's disorder are usually in wheelchairs by their late teenage years, and many of them pass away before age 30 years. Answer A: The Lennox-Gastaut syndrome (LGS) is a pediatric syndrome associated with severe seizures in childhood. Patients usually present between babyhood and age seven years old. The criteria defining the syndrome are rather nonspecific and can result from multiple etiologies such as genetic disorders, tuberous sclerosis, encephalopathic disorders following hypoxic-ischemic insults, meningitis, and head injuries. LGS is characterized by multiple types of seizures ranging from tonic seizures, atonic seizures, absence seizures to myoclonic seizures and non-convulsive status epilepticus. EEG usually demonstrates an atypical spike and wave pattern, a slow, less than 2.5 Hz spike-wave pattern that is generalized and the highest voltage is found in the frontal region. LGS manifests with progressive intellectual disabilities with or without other neurologic abnormalities. Psychotic symptoms are also commonly seen. Oftentimes, the child's neurodevelopment is normal before the first seizure occurs. About a quarter of children with LGS have a history of infantile spasms. Up to 25 percent of children with LGS have a history of infantile spasms. Children with LGS are often difficult to manage medically and have poor seizure and neurologic prognoses. Answer B: Patau syndrome, also known as Trisomy 13 syndrome, has three genetic etiologies including: nondisjunction, spontaneous unbalanced Robertsonian translocation involving the long arm of chromosome 13 and another acrocentric chromosome, or trisomy 13 mosaicism. The major phenotypic features of trisomy 13 are found to be due to an early defect in development of the prechordal mesoderm, which is the origin of the midface, eye, and forebrain. These abnormalities include holoprosencephaly (forebrain of the embryo fails to develop into two hemispheres), absence of the olfactory nerve or bulb, sloping forehead, severe eye defects like microphthalmia and coloboma (fissure or cleft of the iris, ciliary body, or choroid), deafness, and cleft lip and/or palate. Other defects that can be found are omphalocele or umbilical hernia, genitourinary abnormalities, superficial hemangiomas, scalp defects, polydactyly (excessive number of digits), narrow convex fingernails, rocker bottom feet, and congenital heart defects. The majority of prenatally diagnosed cases of trisomy 13 die in utero. Severe intellectual disabilities, seizures, and failure to thrive are commonly found in infants who have survived over one year of age. Answer C: Prader-Willi Syndrome (PWS) occurs due to the absence of a critical region on paternally-inherited chromosome 15q11-q13. This genetic area undergoes the process of imprinting, which is the differential expression of the region based on whether the chromosome is of paternal or maternal origin. The facial appearance in PWS is characterized by upslanting, almond-shaped eyes, thin upper lips, and down- turned mouth. Other features include: light skin and hair coloration, small hands and feet, small external genitalia, prominent low tone, and failure to thrive in infancy. PWS is also characterized by hyperphagia, which develops in early childhood, but can be controlled with behavioral techniques. Patients with PWS have high pain thresholds and rarely vomit. Psychiatric symptoms seen in patients with PWS include: obsessions, compulsions, temper tantrums, skin-picking, anxiety, depression, and psychosis. Patients may also demonstrate a decreased IQ or specific learning disabilities accompanied by areas of relative strength (such as completing jigsaw puzzles). On the other hand, deletion of the same region on the maternally inherited chromosome results in Angelman syndrome, which is characterized by severe intellectual disabilities, postnatal microcephaly, seizures, frequent laughter and smiling, excitable personalities, hypermotoric behaviors, attention problems, decreased speech or absence of speech output, ataxia, etc. Answer E: Smith-Lemli-Opitz syndrome is an autosomal recessive disorder of cholesterol biosynthesis caused by genetic mutations in the 7-dehydrocholesterol reductase gene. The enzyme encoded by this gene is required to convert 7-dehydrocholesterol to cholesterol. The syndrome includes: microcephaly, micrognathia (small-sized jaw), low-set posteriorly rotated ears, and syndactyly of the second and third toes (fusion of digits). Affected males have either ambiguous genitalia or nearly normal-appearing female external genitalia. A number of patients with this syndrome have presented with primary adrenal insufficiency symptoms. Bottom Line: In Rett's disorder, individuals have normal development in the first few months of life, but in later months, they have regression of development of physical, social, and language skills.

A 35-year-old woman visits her primary care physician with the chief complaint of, "I just feel sad all the time." She noticed that she started to feel bad about two months ago and now is having trouble getting out of bed in the morning. She has not been eating as much as usual because food does not seem attractive to her. She also says she feels "worthless" and doesn't really enjoy being around other people any more, though she used to enjoy going out with friends often. She denies suicidal ideation. She thinks she often feels sad like this when the weather is cooler, but it is much worse this winter. Her family history is significant for depression in her father and brother. She drinks occasionally and does not smoke. Which of the following is the most likely diagnosis? A. adjustment disorder with depressed mood B. bipolar II disorder C. cyclothymic disorder D. major depressive disorder with seasonal pattern E. persistent depressive disorder

The correct answer is: D Seasonal Affective Disorder, now known as Major Depressive Disorder with Seasonal Pattern, is a subtype of major depressive disorder in which depressive episodes occur and remit in regular seasonal patterns. Most commonly, this occurs in the fall and winter when daylight is less prevalent. Diagnosis is made when patients meet the diagnostic criteria for depression, but when the symptoms follow a seasonal pattern. First-line treatment is with light therapy, beginning with 10 to 15 minute sessions a day, and gradually increasing to 30 to 45 minutes a day. However, in patients with suicidal ideation or significant functional impairment, pharmacotherapy and psychotherapy are usually indicated. Note that in DSM-5 Seasonal Affective Disorder is now listed as a modifier to Major Depressive Disorder. Answer A: There is no specific event to relate symptoms to for this patient. In DSM-5 adjustment disorders have been re-conceptualized as a heterogeneous array of stress-response syndromes that occur after exposure to a distressing (traumatic or non-traumatic) event, rather than a residual category for individuals who exhibit clinically significant distress but whose symptoms do not meet criteria for a more discrete disorder (as in DSM-IV). Answer B: Bipolar II disorder requires a history of hypomania for diagnosis. Seasonal affective disorder can also be seen as a subtype of bipolar I and bipolar II disorders. Answer C: Diagnosis of cyclothymic disorder requires symptoms for at least 2 years' duration. Answer E: What was referred to as dysthymia in DSM-IV now falls under the category of persistent depressive disorder, which includes both chronic major depressive disorder and the previous dysthymic disorder. Persistent depressive disorder is diagnosed with low mood for most or all of 2 years. Bottom Line: Seasonal affective disorder is a type of major depressive disorder in which major depressive episodes occur in regular seasonal patterns.

A 53-year-old male with a history of depression has been on phenelzine for several years. He comes to the physician complaining about worsening of his depression and wants to try a newer medication. Which of the following is recommended when switching to a selective serotonin reuptake inhibitor? A. initiate SSRI adjunctive therapy immediately and taper phenelzine over 2 weeks B. initiate SSRI monotherapy immediately C. taper phenelzine and initiate SSRI therapy D. wait 14 days after discontinuing phenelzine to initiate therapy E. wait 3 weeks after discontinuing phenelzine to initiate therapy

The correct answer is: D Serotonin syndrome can develop with combination use of MAO inhibitors and SSRIs. At least 14 days after MAO discontinuation should pass to avoid possible complication. Clinical presentation of serotonin syndrome includes tremor, fever, diarrhea, restlessness, and diaphoresis. Answers A & B: Switching medications immediately may cause serotonin syndrome. Answer C: Stopping phenelzine immediately may cause headaches, but is not associated with migraine. This patient can stop phenelzine immediately. Answer E: A three-week period after MAO discontinuation is not necessary. Bottom Line: Serotonin syndrome can develop with combination use of MAO inhibitors and SSRI's. At least 14 days after MAO discontinuation should pass to avoid possible complication. COMBANK Insight : Do not confuse serotonin syndrome with neuroleptic malignant syndrome or malignant hyperthermia. Be able to differentiate between these 3 conditions as you are likely to see at least one of them on test day.

A 21-year-old female college student presents for evaluation after an academic decline. History reveals that she recently started a speech class in which several speeches in front of the class are required throughout the semester. The patient reports that she has always had much discomfort with public speaking since her teenage years, and that she avoids public speaking whenever possible because she becomes nervous and trembles while presenting. The class, however, is required for her degree. She is afraid of being embarrassed in front of the class and is afraid that her classmates will ridicule and laugh at her. Presently she is contemplating changing her major to a subject that does not require speech class. Past medical and psychiatric history is unremarkable. The most likely diagnosis is A. acute stress disorder B. agoraphobia C. panic attack D. social anxiety disorder E. specific phobia

The correct answer is: D Social phobia, now known as social anxiety disorder in DSM-5, comprises of marked and persistent fear of one or more social or performance situations in which the person is exposed to unfamiliar people or to possible scrutiny by others. Giving speeches and presentations are common performance situations which provoke social anxiety in people, just like in this patient. The individual fears that he or she will demonstrate anxiety symptoms or act in a way that will be humiliating or embarrassing. Exposure to the feared social situation provokes anxiety. The feared performance or social situations are avoided or they are endured with intense anxiety or distress. The avoidance and distress interferes significantly with the person's normal routine, academic or occupational functioning, or social activities or relationships. Thus, the patient wants to drop out of her class because she feels like she cannot handle the class due to her intense anxiety. Answer A: Acute stress disorder involves exposure to a traumatic event and a subsequent response of intense fear, helplessness, or horror. The traumatic event involves threat of death to self or others, or a threat to the physical integrity of self or others. Symptoms occur within four weeks of the traumatic event and persist for at least two days and up to less than four weeks. Either while experiencing or after experiencing the traumatic event, the individual has three or more of the following dissociative symptoms: a sense of numbing or detachment, a reduction in awareness of surroundings, derealization, depersonalization, dissociative amnesia. The traumatic event is re-experienced in the form of dreams or images, marked anxiety or hypervigilance exists, and marked avoidance of stimuli occurs. The impairments cause significant distress in social, occupational, or other important areas of functioning. In this question, the patient was not exposed to a traumatic event that threatened the life or physical integrity of self or others. Answer B: Agoraphobia is when individuals experience anxiety about being in places or situations from which escape might be difficult or embarrassing or a situation in which help may not be available in case the individual experiences a panic attack or panic-like symptoms. Agoraphobic fears typically involve being afraid to go outside of the home alone, being a crowded place or standing in a line, traveling on a bus or a train, or being on a bridge. In this question, the patient is having performance anxiety with public speaking, thus making social phobia the correct diagnosis. Answer C: A panic attack is a discrete period of intense fear or discomfort, in which four out of twelve criterion symptoms develop abruptly and reached a peak within 10 minutes. The ten criterion symptoms include: palpitations or accelerated heart rate, sweating, trembling or shaking, sensations of shortness of breath or smothering, feeling of choking, chest discomfort or chest pain, nausea, feeling dizzy or light-headed, derealization (feelings of unreality) or depersonalization (being detached from oneself), fear of losing control, fear of dying, paresthesias (numbness or tingling sensations), and chills or hot flushes. Answer E: Specific phobia consists of marked and persistent fear that is excessive or unreasonable, caused by the presence or anticipation of a specific object or situation. Examples of such objects or situations include flying, heights, animals, driving a car, riding rollercoasters, receiving an injection, and seeing blood. Exposure to the phobic stimulus almost invariably provokes an immediate anxiety response. Individuals recognize that the fear is excessive or unreasonable. The phobic situation is avoided or is endured with intense anxiety or distress. The avoidance and distress associated with the feared situation interferes significantly with the person's normal routine, occupational or academic functioning, or social activities or relationships. The diagnosis of social phobia exists specifically for patients experiencing anxiety in social performance situations, thus making specific phobia the incorrect diagnosis for this scenario. Bottom Line: Social phobia occurs in individuals with excessive anxiety or distress in social performance situations such as giving presentations or speeches. COMBANK Insight : The condition known as social phobia in DSM-IV has been renamed social anxiety disorder in the fifth edition of the Diagnostic and Statistical Manual of Mental Disorders (DSM-5).

A 42-year-old male presents to the emergency room on a detention warrant because he was found walking half-naked in the street, yelling, "The demons are coming!" and would not re-direct when police approached him and fought them with his fists, threatening to hurt anyone who came in his way. The patient continues to yell during triage and upon checking the medical record, it is found that the patient has been treated at the hospital for a psychiatric condition and has a history of nonadherence to recommended therapy. Mental status examination reveals an unkempt appearing, guarded male with poor eye contact who is agitated with paranoid delusions. He refuses to cooperate, has erratic thought processes, and is disoriented to reality. The most appropriate pharmacologic therapy is A. acamprosate B. hydroxyzine C. ketamine D. olanzapine E. quetiapine

The correct answer is: D The main indications for the use of psychotropic medications in an emergency room include for violent or assaultive behaviors, paranoid patients, patients in catatonic states, massive anxiety or panic attacks, and extrapyramidal side effects (such as dystonia or akathisia) of anti-psychotic medications. Episodic, violent outbursts that involve psychosis, such as the one described in this case scenario, respond well to anti-psychotic medications such as haloperidol or olanzapine. Olanzapine is an atypical anti-psychotic medication (second generation) used for psychotic disorders, acute agitation with psychosis, bipolar disorder maintenance for mania, treatment-resistant depression in combination with fluoxetine, behavioral disturbances in dementias, and in impulse control disorders. Olanzapine works as a dopamine D2 antagonist, and works by rapidly blocking dopamine D2 receptors, reducing positive symptoms of psychosis, stabilizing mood symptoms, and reducing combative, explosive, violent, hyperactive, manic, and aggressive behaviors. Olanzapine is known to be very effective in the intramuscular formulation or in the form of orally rapidly disintegrating tablets for psychiatric emergencies as a short-term control of acute agitation and psychosis, especially in conjunction with a low dose of a benzodiazepine, often leading to calming and sedation of the agitated and violent patient. The fact that it can be given in an intramuscular formulation is also crucially important as this patient is not oriented with reality and would most likely refuse an oral medication for control of agitation. Answer A: Acamprosate is used as maintenance treatment for abstinence from alcohol dependence. Its mechanism of action involves reducing excitatory glutamate neurotransmission through blocking certain glutamate receptors, and increasing GABA (gamma-aminobutyric acid) neurotransmission. Withdrawal from chronic alcohol use can lead to excessive glutamate activity and deficient GABA activity, acamprosate acts like "artificial alcohol" to diminish these effects. Regular compliance with acamprosate increases abstinence from alcohol. It is NOT to be used in individuals who are not abstinent from alcohol at the time of treatment initiation and it is preferred to avoid this medication in binge drinkers. Answer B: Hydroxyzine is an anti-histaminic medication used in clinical psychiatry as an anxiolytic. It works by blocking histamine 1 receptors. It is used to alleviate anxiety and the oral tablet form takes only 15-20 minutes to work. However, it can take several weeks with daily dosing for maximal therapeutic benefit in chronic anxiety. It can be used on an as needed basis for short-term mild to moderate anxiety attacks. Hydroxyzine has multiple formulations including tablets, capsules, liquid, and an injectable form, but it is not as efficacious for severe anxiety or agitation, as described in this case scenario. Answer C: Ketamine is an anesthetic that may be resorted to when initial treatments (benzodiazepines and antipsychotics) have failed. Ketamine is a noncompetitive NDMA receptor antagonist that produces a cataleptic-like state in which the patient is dissociated from the surrounding environment by direct action of the cortex and limbic system. The onset is 1-2 minutes intravenously and about 5 minutes intramuscularly. Answer E: Quetiapine is an atypical anti-psychotic (second generation) medication used for treating psychotic disorders, mood disorders, behavioral disturbances in dementias and Parkinson's disease, severe treatment-resistant anxiety, impulse control disorders, and behavioral disturbances in children and adolescents. It works as a serotonin-dopamine antagonist through blocking dopamine D2 receptors (reducing positive symptoms of psychosis and stabilizing affective mood symptoms) and through blocking serotonin 2A receptors (enhancing dopamine release in certain brain regions, reducing motor side effects, improving cognitive and affective symptoms). Psychotic and manic symptoms can improve within one week, but it may take several weeks for full effect on behavior and mood stabilization. Thus, it is not indicated immediately in this case scenario as it would not be helpful in cases requiring rapid onset of action. It can help in the long run with chronic control of aggressive symptoms if given over some period of time of at least several weeks to months or more, but would not be useful as acute treatment in a psychiatric emergency. Bottom Line: Severely agitated and violent patients typically require antipsychotics such as haloperidol or olanzapine. Those with undifferentiated agitation should be provided benzodiazepines.

A 25-year-old graduate student comes to see you because she is worried about being dismissed from her graduate program. She has failed her oral examinations twice and her third and last attempt is in two weeks. She denies trouble with the academic material, but says that when she gets up in front of her examination committee her heart begins to race, her hands shake, her palms get sweaty, and she "draws a blank" when asked questions even though she knows the answer. She has always avoided public speaking in the past because she forgets what she wanted to say as soon as she looks at the audience. Which of the following is the most appropriate therapy? A. alprazolam before her exam B.exposure and desensitization C. paroxetine daily D. propanolol before her exam E. psychodynamic therapy

The correct answer is: D This patient admits to a long history of difficulty speaking in public, and this difficulty is so debilitating to her that she has failed her oral boards several times. Her symptoms and fear of speaking are consistent with social phobia. Social phobia refers to a persistent fear of at least one social or performance situation during which there is exposure to strangers or there is risk of scrutiny. These patients are typically afraid that they will act in a manner that will be embarrassing to them, and there is usually a large amount of anxiety involved. When faced with the feared situation, they will develop large amounts of anxiety and may even have a panic attack. People with social phobias usually recognize that their fear is out of proportion to the task, but will remain anxious and this anxiety usually interferes with the persons normal routine. Phobias are the most common mental health disease in the United States, estimated to affect between 5 to 10% of the population, with females being more affected than males. Treatment usually involves psychotherapy, which involves a combination of behavioral and cognitive therapy utilizing desensitization to the feared situation. Psycopharmacotherapy is sometimes indicated for severe social phobias, and selective serotonin uptake inhibitors (SSRIs), benzodiazepines, venlafaxine, and buspirone are used long-term. Beta-blockers can be used to acutely control the anxiety associated with performance situations. While this patient will likely need a combination of these aforementioned treatments, given that her oral boards are in two weeks, they best pharmacologic treatment would be a beta-blocker right before her exam. Answer A: Benzodiazepines (alprazolam) can be used in social phobia, but they are a second-line treatment. Answer B: During exposure and de-sensitization, the patient is slowly desensitized with controlled "doses" of the feared stimulus. While exposure and desensitization is used to treat social phobia, and this patient ought to begin therapy, the results will not likely be significant enough in two weeks. Answer C: Selective serotonin reuptake inhibitors (paroxetine) are useful when the social phobia affects the patient's life regularly, but this patient has trouble only with the specific situation of public speaking. Answer E: Psychodynamic therapy is not indicated for social phobia. Bottom Line: Social phobia refers to fear of performing in public; beta-blockers are used to control the somatic symptoms of social phobia.

A 23-year-old male presents to the emergency department for abdominal pain. He says that he has had this pain for about 8 months. The pain is located in the right lower quadrant, intermittent in nature and radiates throughout his body as well as being associated with vomiting and diarrhea. When further questioned, he states that he believes the pain is due to snake that is crawling around his body. He states that the snake's name is "Viper" that he has to drink extra water each day so that there is enough for both of them. He hears the snake hissing when it is mad and has not had enough water. The patient states that he likes the snake, but has been having difficulty going to work because he has to consistently drink water for the snake. The most likely explanation for his symptoms is A. decreased acetylcholine B. decreased dopamine C. decreased serotonin D. increased dopamine E. increased epinephrine and norepinephrine

The correct answer is: D This patient is presenting with symptoms of schizophrenia. Males and females are equally affected. There is a peak incidence in males from age 18-25 years old whereas females usually present at age 25-35. It is diagnosed by two or more positive symptoms or negative symptoms for six or more months with social occupational dysfunction and is linked to increased levels of dopamine. Examples of positive symptoms include hallucinations, delusions, disorganized speech, bizarre behavior, and thought disorder. Negative symptoms include flat affect, decreased emotional reactivity, poverty of speech, and anhedonia. This disease is part of a spectrum and one must distinguish between schizoid, schizotypal and schizophrenia. Patients who have schizoid personality disorder are isolated and detached from others with no interest in interpersonal relationships. Schizotypal is similar to schizoid but odd thinking is seen. When symptoms are seen for less than 1 month, the term brief psychotic disorder can be used. Symptoms that last from 1-6 months is termed schizophreniform disorder. Symptoms that last beyond 6 months is schizophrenia. Answer A: Decreased acetylcholine has been linked to Alzheimer's disease. Alzheimer's is usually a disease of the elderly. The first presenting sign is often amnesia and can include aphasia, agnosia, and apraxia. Answer B: Parkinson's disease occurs after 50-60 years of age and presents with the tetrad of resting tremor, rigidity, bradykinesia, and postural instability. It is a disease that is caused by decreased levels of dopamine. Answer C: Decreased serotonin is one of the mechanisms of major depressive disorder. Diagnosis requires depressed mood or anhedonia along with five or more signs and symptoms from the mnemonic "SIG E CAPS" (disturbances in sleep, interest, guilt, energy, concentration, appetite, psychomotor agitations or suicidal ideation) for a 2 week period. Answer E: Increased norepinephrine and epinephrine is seen in pheochromocytoma. Patients present with sudden and episodic hypertension, headaches, diaphoresis, tachycardia. Bottom Line: Schizophrenia is associated with increased levels of dopamine. COMBANK Insight : Two changes were made to Criterion A for schizophrenia: 1) the elimination of the specific attribution of bizarre delusions and Schneiderian first-rank auditory hallucinations (e.g., two or more voices conversing), leading to the requirement of at least two Criterion A symptoms for any diagnosis of schizophrenia, and 2) the addition of the requirement that at least one of the Criterion A symptoms must be delusions, hallucinations, or disorganized speech. The DSM-IV subtypes of schizophrenia were eliminated due to their limited diagnostic stability, low reliability, and poor validity. Instead, a dimensional approach to rating severity for the core symptoms of schizophrenia

A 32-year-old woman presents to the emergency room after attempting suicide by cutting her wrists. She has a several year history of depression and wrist cutting, which she states she does whenever "things seem hopeless." She has been in 5 relationships in 2 years, all of which have ended badly with restraining orders or police interventions. According to the emergency room nurse, one minute she is crying and the next minute she is laughing. She complains that the nurses made her wait too long in the waiting room and because of that, they should be fired. She tells the physician, "you are a very good doctor because you listened to me. You are my favorite." This patient is exhibiting which of the following defense mechanisms? A. altruism B. displacement C. regression D. splitting E. suppression

The correct answer is: D This patient most likely has borderline personality disorder (BPD), which is one of the cluster B personality disorders, along with antisocial, histrionic, and narcissistic personality disorders. Patients with BPD have very unstable moods, unpredictable personalities, and unstable interpersonal relationships. They are very impulsive, attention-seeking, have very poor self-esteem, and will often have recurrent suicidal threats/attempts and frequently resort to self-mutilation. BPD has a prevalence of 1 to 2% in the population, with women two times as likely to have BPD. While suicidal threats and attempts are often employed by patients with BPD to gain attention, they are still at high risk for suicide, with approximately a 10% suicide rate. Patients with borderline personality disorder often display splitting. Splitting is a defense mechanism in where a situation is black and white, there is no shade of grey. Patients with borderline personality disorder will often see people as being very good or very bad, as this patient has done in this scenario - hating her nurses, but loving her doctor. Answer A: Altruism is an advanced defense mechanism in which a person helps others to deal with their problem. A good example would be a person who struggles with drug addiction volunteering to speak to troubled teens with drug problems. Answer B: Displacement is a defense mechanism in which a person shifts a negative feeling from one thing to another. For example, a person who is angry at his boss at work may go home and yell at his children. Answer C: Regression is a primitive defense mechanism in which a person deals with a problem by reverting to activities which would be more appropriate for an earlier stage of development. An example would be a person who returns to bedwetting after an emotional trauma. Answer E: Suppression is a defense mechanism in which one consciously makes an effort to not think about or remember an emotional trauma. The conscious nature of this separates it from repression, which is unconscious. Bottom Line: Splitting is a defense mechanism seen in borderline personality disorder in where the patient sees everything in black and white, good or bad. It is a key finding in borderline personality disorder.

A 35-year-old man with a previous history of multiple psychiatric hospitalizations for attempted suicide is admitted after he swallows all of his antidepressants at once. After he is medically stabilized, he is transferred to the psychiatric ward where he begins individual and group therapy in addition to having his medications adjusted. His treatment team consists of a resident, a social worker, and the attending psychiatrist. One week into his stay he tells the resident, "You are the best doctor I've ever had, but the rest of them are incompetent. I only want you to do my therapy from now on." He has a 17 pack-year smoking history and drinks approximately three beers daily. He has used marijuana and MDMA in the past but is not currently using any illicit drugs. Mental status exam reveals depressed mood and restricted affect without disorders of thought content of process. Which of the following defense mechanisms is this patient most likely using? A. acting out B. projection C.repression D. splitting E. suppression

The correct answer is: D This patient most likely has borderline personality disorder (BPD), which is one of the cluster B personality disorders, along with antisocial, histrionic, and narcissistic personality disorders. Patients with BPD have very unstable moods, unpredictable personalities, and unstable interpersonal relationships. They are very impulsive, attention-seeking, have very poor self-esteem, and will often have recurrent suicidal threats/attempts and frequently resort to self-mutilation. BPD has a prevalence of 1 to 2% in the population, with women two times as likely to have BPD. While suicidal threats and attempts are often employed by patients with BPD to gain attention, they are still at high risk for suicide, with approximately a 10% suicide rate. Patients with borderline personality disorder often display splitting. Splitting is a defense mechanism in where a situation is black and white, there is no shade of grey. Patients with borderline personality disorder will often see people as being very good or very bad, as this patient has done in this scenario- loving only her resident while calling everyone else on the team incompetent. Answer A: Acting out is an immature defense mechanism in which the patient gives into an unacceptable impulse rather than face the anxiety of suppressing it. Answer B: Projection is the attribution of thoughts or feelings to another person who does not, in fact, have those thoughts or feelings. An example would be a man who feels guilty for cheating on his wife accusing her of cheating on him. Answer C: Repression is the unconscious prevention of an undesirable thought or feeling from reaching consciousness. Answer E: Suppression is a mature defense mechanism in which the subject consciously ignores unacceptable impulses or emotions in order to achieve a desired outcome. Bottom Line: Splitting is the labeling of people as either all good or all bad, often used by patients with borderline personality disorder. It may have the effect of causing conflict within the treatment team if not recognized.

A 30-year-old female is brought to her primary care physician by her husband for strange behavior. He states that for the past 3 years she has been different than she was at the beginning of their marriage. He feels that her moods have been constantly shifting. She is often down, fatigued, and has low self-esteem. However, her husband will notice periods of a few days during which she has increased energy and productivity, and will engage in behavior that is atypical for her, such as excessive shopping and drinking. During those times, she will often sleep very little, and stay up through most of the night working on home improvements. When asked about her behavior, she feels that her husband is exaggerating, stating that everyone has "good days and bad days," and feels that it is ridiculous that he even brought her to the office. She feels that her periods of increased energy are great because she is able to get so much done. Both her and her husband deny any hallucinations or delusions or suicidal ideation during this time. She has no significant past medical history and takes no medications. Physical examination is normal. Which of the following is the most likely diagnosis? A. bipolar I disorder with rapid cycling B. bipolar II disorder with rapid cycling C.borderline personality disorder D. cyclothymic disorder E. drug intoxication

The correct answer is: D This patient's symptoms are most consistent with cyclothymic disorder, a chronic, fluctuating mood disturbance involving numerous periods of hypomanic symptoms and periods of depressive symptoms. The DSM-5 criteria includes the following: A. For at least 2 years (at least 1 year in children and adolescents) there have been numerous periods of hypomanic symptoms that do NOT meet criteria for a hypomanic episode and numerous periods with depressive symptoms that do NOT meet criteria for a major depressive episode. B. During the above 2 year period, the hypomanic and depressive periods have been present for at least half the time and the individual has not been without the symptoms for more than 2 months at a time. The DSM-5 criteria for a manic episode requires the following: A. A distinct period of abnormally and persistently elevated, expansive, or irritable mood and abnormally and persistently increased activity or energy, lasting at least 1 week and present for most of the day, nearly every day. B. During this period three of the following symptoms have persisted representing a noticeable change from usual behavior: inflated self-esteem or grandiosity, decreased need for sleep, more talkative, flight of ideas, distractibility, increased goal-directed activity, or excessive involvement in activities that have a high potential for painful consequences. C. The episode is sufficiently severe to cause marked impairment in social or occupational functioning or to necessitate hospitalization to prevent harm to self or others, or there are psychotic features. The DSM-5 criteria for a hypomanic episode requires the following: A. A distinct period of abnormally and persistently elevated, expansive, or irritable mood and abnormally and persistently increased activity or energy, lasting at least 4 consecutive days and present for most of the day, nearly every day. B. During this period three of the following symptoms have persisted representing a noticeable change from usual behavior: inflated self-esteem or grandiosity, decreased need for sleep, more talkative, flight of ideas, distractibility, increased goal-directed activity, or excessive involvement in activities that have a high potential for painful consequences. E. The episode is NOT severe enough to cause marked impairment in social or occupational functioning. Answer A: Both disorders may resemble cyclothymic disorder by virtue of the frequent marked shifts in mood. By definition, in cyclothymic disorder the criteria for a major depressive, manic, or hypomanic episode has never been met, whereas the bipolar I and bipolar II specifier "with rapid cycling" requires that full mood episodes be present. Answer B: A diagnosis of bipolar II disorder requires a major depressive episode. Major depressive episodes require five of the following symptoms for at least a 2-week period: depressed mood, anhedonia, change in appetite, feelings of worthlessness or excessive guilt, insomnia or hypersomnia, diminished concentration, psychomotor agitation or retardation, recurrent thoughts of death or suicide. From the question stem the patient does not meet qualifications of major depressive disorder. Answer C: Borderline personality disorder is associated with marked shifts in mood that may suggest cyclothymic disorder. Those who are borderline exhibit a pervasive pattern of instability of interpersonal relationships, self-image, and affects, and marked impulsivity, beginning in early adulthood and present in a variety of contexts. Answer E: There is no evidence of drug intoxication in this patient, but drugs must always be considered as a cause of psychiatric disease. Bottom Line: Cyclothymic disorder is the combination or hypomania and dysthymia. Hypomania is less severe than true mania and does not cause significant impairments in occupational or social functioning and never requires psychiatric hospitalization. COMBANK Insight : Diagnostic criteria for bipolar disorders now include both changes in mood and changes in activity or energy. The DSM-IV diagnosis of bipolar I disorder, mixed episodes - requiring that the individual simultaneously meet full criteria for both mania and major depressive episode - is replaced with a new specifier "with mixed features." Particular conditions can now be diagnosed under other specified bipolar and related disorder, including categorization for individuals with a past history of a major depressive disorder whose symptoms meet all criteria for hypomania except the duration criterion is not met (i.e., the episode lasts only 2 or 3 days instead of the required 4 consecutive days or more). A second condition constituting another specified bipolar and related disorder variant is that too few symptoms of hypomania are present to meet criteria for the full bipolar II syndrome, although the duration, at least 4 consecutive days, is sufficient.

A 45-year-old Caucasian male presents to the emergency department following attempted suicide by hanging. He has had one prior suicide attempt and lives alone. The patient has a history significant for antisocial personality disorder and has relapsed several times from therapy for alcohol abuse. The single most important factor that places this patient at highest risk for future suicide attempts is A. alcoholism B. antisocial personality disorder C. ethnicity D. living alone E. prior suicide attempts

The correct answer is: E Nearly 30% of all individuals who attempt suicide will try again at some point, making it the single most important predictor of future suicide attempts. Of this group of patients, 10% will be successful. The patient in the above case has many factors that place him at risk for future attempts. He is a male who lives alone and has a longstanding history of alcohol abuse and antisocial personality disorder. None of these factors are as favorable for recurrent attempt, however, as the fact that this patient has had prior suicide attempts. Answer A: Although alcoholism may predispose a patient to depression, this is not the highest risk factor for future attempts. Answer B: There is no association between antisocial personality disorder and suicide attempts. Patients with major depressive disorder are at increased risk of attempting suicide. Answer C: Ethnicity is not a risk factor for suicide attempts. Answer D: Living alone is not the highest risk factor for future suicide attempts. Bottom Line: The single most important predictor of future suicide attempts is a history of prior attempts.

A 52-year-old male presents for evaluation of unusual behavior. History reveals that he is employed as a laborer for a property management company. The patient does not typically come into contact with tenants when he is working on a property, but on one occasion he encountered a couple while they were moving into their apartment. When he learns that one of the two were about to start working for the nearby national laboratory he warns him about the "aliens" who also work there. He reports that they are actually lizards underneath their human skin and warned the individual to "be careful up there." Observation reveals that he is reluctant to make eye contact but did not seem to be responding to internal stimuli. Which of the following is the most likely diagnosis? A. antisocial personality disorder B. delusional disorder C. schizoid personality disorder D. schizophrenia E. schizotypal personality disorder

The correct answer is: E Schizotypal personality disorder (SPD) is one of the cluster A personality disorders, along with schizoid and paranoid personality types. Cluster A personality types are often perceived as being eccentric or "weird." Patients with schizotypal personality disorder have a pervasive pattern of both eccentric behavior and unusual thoughts. These are people who are often thought of as "weird" or "strange." They usually have deficits in their ability to have close relationships, and typically have peculiar ideations, as well as unusual appearance and behavior. Under large amounts of stress, these patients can become transiently psychotic. The personality disorder usually begins to emerge in early adulthood. Diagnosis requires five or more of the following: (1) odd beliefs or magical thinking, (2) ideas of reference, (3) unusual perceptual experiences, (4) excessive social anxiety, (5) suspiciousness of others, (6) odd or strange appearance and behavior, (7) few close personal relationships, (8) odd thinking or speech, (9) inappropriate or restricted affect. SPD affects approximately 3% of the population. Psychotherapy is the treatment of choice for these patients, and some may eventually go on to develop schizophrenia. Answer A: Antisocial personality disorder is one of the cluster B personality disorders, in which patients tend to be emotional, impulsive, and dramatic. Patients with this disorder refuse to conform to social norms and tend to show no remorse for their actions. They are impulsive, frequently violate laws, and show a blatant disregard for the rights of others. Answer B: There is overlap with delusional disorder and brief psychotic disorder but the odd beliefs in schizotypal personality disorder are not necessarily fixed, as are delusions. Also, the delusions in delusional disorder are non-bizarre, while many odd beliefs of schizotypal personality disorder are quite bizarre. Answer C: Schizoid personality disorder is also one of the cluster A personality disorders, and is characterized by a lifelong pattern of social withdrawal. These people are also usually perceived as weird or unusual, but do not display magical thinking like schizotypal personality types. Answer D: The absence of any true paranoid delusions or hallucinations makes schizophrenia unlikely. It would also be unusual for a patient to present with schizophrenia at the age of 50. Bottom Line: Schizotypal personality disorder is one of the cluster A personality types, and is characterized mainly by odd beliefs and magical thinking. COMBANK Insight : Two changes were made to Criterion A for schizophrenia: 1) the elimination of the specific attribution of bizarre delusions and Schneiderian first-rank auditory hallucinations (e.g., two or more voices conversing), leading to the requirement of at least two Criterion A symptoms for any diagnosis of schizophrenia, and 2) the addition of the requirement that at least one of the Criterion A symptoms must be delusions, hallucinations, or disorganized speech. The DSM-IV subtypes of schizophrenia were eliminated due to their limited diagnostic stability, low reliability, and poor validity. Instead, a dimensional approach to rating severity for the core symptoms of schizophrenia is included.

A 52-year-old male presents for a routine wellness examination. Past medical history is pertinent for hypertension, psoriasis, and asthma. He is a non-smoker, denies illicit drug use, and reports being laid off four months ago for company financial reasons. He drinks about 5 beers per night on a daily basis. He says he has been trying to cut down on his drinking and often becomes annoyed when asked about his drinking. He denies any feelings of guilt about his alcohol use, and he denies needing an "eye opener" to start his day. This patient's history is A. diagnostic for alcohol dependency B. diagnostic for alcohol use disorder C. insufficient to determine whether alcohol abuse is present D.negative for alcohol abuse E. suspicious for alcohol abuse

The correct answer is: E The CAGE Questionnaire is a standard tool for assessing alcohol abuse in patients. The CAGE Questionnaire is a quick and reliable screening, even for patients who to try to conceal their alcohol abuse. It found to be a more reliable indicator than elevated liver function tests. The acronym of CAGE stands for: Have you ever felt you should Cut down on your drinking? Have people Annoyed you by criticizing your drinking? Have you ever felt bad or Guilty about your drinking? Have you ever had a drink first thing in the morning to steady your nerves or to get rid of a hangover (Eye opener)? Each question that is answered as a "yes" scores 1 point. In this scenario, the patient answers "yes" to 2 out of 4 questions, scoring 2 points in total. A score of 2-3 points is highly suggestive of alcohol abuse, and a score of 4 is suggestive of alcohol dependency. Answer A: Alcohol dependency is a physiologic addiction that results in tolerance and withdrawal effects, which this patient is not exhibiting at this time. Answer B: The DSM-5 criteria for alcohol use disorder requires a pattern of alcohol use leading to clinically significant impairment or distress, as manifested by at least two of the following within a 12-month period: 1. Alcohol is taken in larger amounts or over a longer period than intended. 2. Persistent desire or unsuccessful efforts to cut down or control alcohol use. 3. A great deal of time is spent in activities necessary to obtain or use alcohol, or recover from the effects. 4. Craving, or a strong desire or urge to use alcohol. 5. Recurrent use resulting in failure to fulfill major role obligations. 6. Continued use despite having persistent or recurrent social or interpersonal problems caused by alcohol. 7. Important social, occupational, or recreational activities are given up or reduced because of alcohol use. 8. Recurrent use in situations which it is physically hazardous. 9. Use is continued despite knowledge of having a persistent or recurrent physical or psychological problem that is likely to have been caused or exacerbated by alcohol. 10. Meets diagnostic criteria for tolerance. 11. Meets diagnostic criteria for withdrawal. Answer C: Based on the CAGE Questionnaire, there is sufficient information in the patient's history to determine some level of alcohol abuse. Answer D: The number of drinks per week is concerning when males have more than 20 drinks per week and when females have more than 10 drinks per week. Thus, this patient is at risk for alcohol dependence, which is a physiologic addiction resulting in withdrawal symptoms. Bottom Line: CAGE scoring: 1 point - At risk for alcohol dependency; 2-3 points - Highly suspicious for alcohol dependency; 4 points - diagnostic for alcohol dependency. COMBANK Insight : One would not expect a CAGE questionnaire item to be difficult, but often times the COMLEX is able to extrapolate the difficult parts of seemingly simple topics, such as screening for alcohol abuse. Of note, substance use disorders comprise 11-17% of the Psychiatry COMAT.

A 55-year-old male presents for evaluation of altered mental status of two months' duration. History reveals the patient is having problems with his memory, although he cannot describe any symptomatology. His past medical history is negative for any psychiatric outpatient visits or hospitalizations, but he does report depression intermittently for several years. Social history reveals that he has been drinking alcohol heavily at least 3-4 times per week over the past ten years since a divorce. He denies concurrent tobacco or illicit drug use. He said he lives in a fifteen bedroom home with twelve cats and five dogs with his brother. He denies any auditory or visual hallucinations. Vital signs reveal the following: Temperature 36.6ºC (98ºF) Blood pressure 145/86 mmHg Respiratory rate 18/min Heart rate 99/min Physical examination reveals an unkempt male who appears older than his age. He has an ataxic gait, horizontal nystagmus, and slight wheezing upon auscultation. Mental status examination reveals illogical thought processes and loose associations. He is confused regarding the month and believes the current president to be Bill Clinton. Laboratory studies reveal an ammonia level that is within normal limits and an EEG does not demonstrate any abnormalities. The most likely diagnosis is A. acute alcoholic hallucinosis B. delirium tremens C. hepatic encephalopathy D. Wernicke's encephalopathy E. Wernicke-Korsakoff's syndrome

The correct answer is: E The classic names for alcohol-induced persisting amnestic disorder are Wernicke's encephalopathy (set of acute symptoms) and Korsakoff's syndrome (chronic condition). Wernicke's encephalopathy, also known as alcoholic encephalopathy, is an acute neurological disorder seen in alcoholics characterized by vestibular dysfunction, ataxia, ophthalmoplegia (ocular abnormalities such as horizontal nystagmus, lateral orbital palsy, gaze palsy), and/or confusion. Wernicke's encephalopathy occurs due to thiamine deficiency or malabsorption, which occurs frequently in those who abuse alcohol. Thiamine is a co-factor for several important enzymes and may also be in nerve conduction along the axon and in synaptic transmission. The neuropathological lesions are paraventricular and symmetrical, involving the mammillary bodies, the thalamus, the hypothalamus, the midbrain, the pons, the medulla, the fornix, and the cerebellum. It can be fatal but is most often reversible with thiamine. It is important to give thiamine before administering glucose because if given vice versa, the encephalopathy can rapidly worsen. Wernicke's encephalopathy may resolve after large doses of parenteral thiamine treatment as it is reversible within a few days or weeks, or it may progress into the more severe, often irreversible condition of Korsakoff's syndrome. The cardinal features of Korsakoff's syndrome are impaired recent memory and anterograde amnesia (loss of the ability to create new memories after the event that caused amnesia) in an alert and responsive patient. He exhibits anterograde amnesia in that he cannot describe any of his symptomatology over the last two months. Patients may or may not have the symptom of confabulation, which is a memory disturbance characterized by the production of fabricated and distorted memories. In this case scenario, the patient has symptoms of Wernicke-Korsakoff's syndrome: nystagmus, ataxia, confusion, anterograde amnesia, confabulation. He said he lives in a fifteen-bedroom home with four cats and two dogs. This is an example of confabulation, where the patient is not purposefully lying, but there is production of distorted or fabricated memories about the self or the world. Only about 20 percent of individuals with Korsakoff's syndrome recover fully, but some will have improvement in cognition with IV thiamine and nutritional support. Answer A: Acute alcoholic hallucinosis occurs in an alcohol-dependent individual after the cessation of drinking, and occurs during alcohol withdrawal. No delirium, tremors, or autonomic hyperactivity develops. The individual experiences auditory and paranoid hallucinations. Symptoms can become chronic. In this case scenario, the patient does not report any hallucinations, and withdrawal symptoms occur on an acute basis. This case scenario describes the patient last drinking about two months ago. Answer B: Delirium tremens (DTs) describes late, acute withdrawal symptoms with an onset of 48-96 hours after the last drink and cessation from heavy drinking. DTs are otherwise known as alcohol withdrawal delirium. Symptoms of DTs include the following: tremors, an increase in psychomotor activity, vivid hallucinations, illusions, delusions, profound disorientation and confusion, increased autonomic activity (tachycardia, hyperhidrosis, fever, dilated pupils). Patients may also experience formication (tactile hallucinations that feel like insects crawling all over the skin). In this case scenario, the patient last drank about two months ago, and DTs describe a more acute withdrawal that occurs within 48-96 hours after the last drink, and in DTs, patients often present with abnormal vital signs of fever and tachycardia. Answer C: Metabolic encephalopathy should be considered whenever sudden or abrupt changes in thinking, orientation, behavior, or consciousness occur. Hepatic encephalopathy may result from acute, subacute, or chronic hepatocellular failure, often precipitated by hyperammonemia, with a range of neuropsychiatric symptoms from mild personality changes to coma. Early signs include the following: mild intellectual difficulties, often covered by intact verbal ability; problems with recent memory, impaired orientation, objective mental slowing, mild confusion, decreased concentration, depressed or labile mood, irritability, sleep-wake reversal, decreased personal grooming. Some patients become anxious, agitated, and hyperactive while other patients often become inactive and withdrawn. There may be periods of intermittent disorientation, inappropriate behaviors, and outbursts of rage before the progressive deterioration in consciousness, speech, cognition, and memory leave the patient somnolent, incoherent, disoriented, confused, and amnestic. Severe hepatic encephalopathy manifests with physical signs of asterixis (flapping tremor of the hands), hyperventilation, EEG abnormalities. As hepatic encephalopathy progresses, confusion or delirium progresses to decreased responsiveness or stupor, and the final stage is coma. In this case scenario, the patient's ammonia level is within normal limits, which is not the case in most patients with hepatic encephalopathy. The patient in this case scenario does not present with intermittent disorientation, agitation, incoherence, asterixis, abnormal EEG changes, as would be evident in someone with hepatic encephalopathy. Answer D: Wernicke's encephalopathy occurs due to thiamine deficiency or malabsorption, which occurs frequently in those who abuse alcohol. Thiamine is a co-factor for several important enzymes and may also be in nerve conduction along the axon and in synaptic transmission. The neuropathological lesions are paraventricular and symmetrical, involving the mammillary bodies, the thalamus, the hypothalamus, the midbrain, the pons, the medulla, the fornix, and the cerebellum. It can be fatal but is most often reversible with thiamine. It is important to give thiamine before administering glucose because if given vice versa, the encephalopathy can rapidly worsen. Wernicke's encephalopathy may resolve after large doses of parenteral thiamine treatment as it is reversible within a few days or weeks, or it may progress into the more severe, often irreversible condition of Korsakoff's syndrome. Bottom Line: Wernicke-Korsakoff's syndrome occurs in individuals with chronic alcohol use and consists of confusion, ataxia, ophthalmoplegia, vestibular problems, confabulation, and anterograde amnesia. COMBANK Insight : Anywhere from 4-10% of Psychiatric COMAT questions will be about a psychiatric illness secondary to a general medical condition. This is often an aspect of psychiatry that is unrecognized, thus questions of this caliber can take one off-guard unless studied for. Please review medical causes of depression, dementia, and anxiety.

A 47-year-old woman presents to her primary care physician with the chief complaint of insomnia. She says that she goes to bed around 8 o'clock every night and gets up at 7 o'clock the next morning, but she does not usually fall asleep until midnight or later and often wakes early in the morning and tries to go back to sleep until she eventually gets up. She states her mood is "okay," and that she has not felt like eating much recently; however, she does not think she has lost any weight. She sometimes has trouble concentrating at work and has generally felt that her energy has been low. Family history is significant for heart disease in her father and asthma in her mother. She smokes half a pack of cigarettes daily and drinks socially on weekends. She has never used illicit drugs. On physical examination, she is a well-developed woman who appears tired. Mood and affect are appropriate. Neurological examination is unremarkable. Which of the following is the next best step in the treatment of this patient? A. order a sleep study B.prescribe alprazolam C. prescribe hormone replacement therapy D. prescribe trazodone E. recommend sleep hygiene

The correct answer is: E This patient has classic signs and symptoms of insomnia disorder. The diagnosis of insomnia requires difficulty initiating or maintaining sleep and consequent daytime sleepiness and/or difficulty completing tasks. The disturbance must occur three or more times a week for at least one month. Insomnia has been estimated to affect up to 30% of the general population, and is more prevalent in females, middle-aged and older adults, and patients with general medical and psychiatric conditions. Insomnia has also been found to be exacerbated by anxiety about not being able to get adequate amounts of sleep. First line treatment for insomnia consists of educating patients well on proper sleep hygiene. Sleep hygiene measures consist of the following: Regular sleeping schedule: going to bed at the same time each night and awakening at around the same time Not spending more time in bed than needed and avoiding staying in bed in the morning to catch up on sleep Avoiding daytime naps Avoid exercising and other vigorous activities before bedtime Use the bed only for sleep and sex Limit caffeine intake, ideally not consuming any caffeine after lunch Soaking in hot tub or warm bath prior to bedtime Avoid watching TV in the bedroom or falling asleep on sofa then going to bed Keeping the bedroom peaceful, dark, quiet and at a comfortable temperature Avoid sleeping pills or over-the-counter drug remedies. If proper sleep hygiene is not adequate enough, pharmcotherapy can be used short-term, which options such as ambien, benadryl, and trazodone. Answer A: A sleep study would be appropriate after other avenues have been exhausted or if sleep apnea were suspected. Answer B: Benzodiazepines can be useful for sleep after behavioral modification and non-habit forming medications have been tried. Alprazolam is still not the best choice because of their rapid onset and short half-life make dependence more likely. Answer C: Hormone replacement therapy is useful for patients with severe menopausal symptoms that have not responded to other therapy. This patient's symptoms are more likely to resolve with improvement of sleep hygiene rather than hormonal therapy. Answer D: Trazodone is useful for sleep but behavioral modifications should be tried first. Bottom Line: The first treatment for insomnia is behavioral change to encourage proper sleep hygiene. This includes going to bed when tired, not staying in bed after trying to sleep for more than 20 minutes, arising at the same time each day, moderate exercise during the day, and limitation of caffeine and other stimulants.

A 31-year-old woman is found unresponsive in her car by the police and is transported to the emergency department. On arrival to the emergency department her speech is garbled, and she is unable to give a history. No family members or medical records are available. Vitals reveal her temperature is 39.2°C (102.6°F), heart rate is 110/min, blood pressure is 125/85 mm Hg, and respirations are 20/min. She reacts purposefully to jaw thrusts but is unable to respond to questioning. Her face appears flushed and mucous membranes and underarms are dry. Pupils are 4 mm and sluggishly reactive. Reflexes are 1+ globally. There is no clonus with ankle jerk. An overdose of which of the following medications could be responsible for her condition? A. acetaminophen B. aspirin C. haloperidol D.lithium E. olanzapine

The correct answer is: E This patient is presenting with signs and symptoms consistent with anticholinergic toxicity, which are particularly seen in low-potency, atypical antipsychotics, such as olanzapine. The following is the classic description of anticholinergic toxicity: "Red as a beet" - caused by cutaneous vasodilation "Dry as a bone" - referring to the anhidrosis that results from anticholinergic toxicity "Hot as a hare" - secondary to anhidrotic hyperthermia "Blind as a bat" - referring to nonreactive mydriasis "Mad as a hatter" - patients will typically present with delirium and hallucinations "Full as a flask" - referring to urinary retention Patients presenting with a history and symptoms consistent with anticholinergic toxicity first need to be ruled out for other medical or toxicology causes of their symptoms. Patients who are extremely agitated and/or are seizing should first receive benzodiazepines to control symptoms. Management of hyperthermia is also extremely important. Physostigmine is often used as an antidote. Answer A: Acetaminophen overdose leads to abdominal pain, nausea, vomiting, and possibly severe liver toxicity. Answer B: Salicylate overdose causes tinnitus, nausea, vomiting, metabolic acidosis, respiratory alkalosis, seizure, coma, and death. Answer C: Haloperidol overdose causes lethargy, tremors, and dystonia. The typical antipsychotics may cause anticholinergic toxicity as do the atypical antipsychotics, however, they cause this at a much lower frequency than do atypicals. Answer D: Lithium overdose causes nephrogenic diabetes insipidus, hyperreflexia, tremor, ataxia, nystagmus, and seizure. Bottom Line: Olanzapine is an atypical antipsychotic; therefore, it can carry anticholinergic properties. Overdose may lead to anticholinergic toxicity, characterized by hyperthermia, facial flushing, dry mucus membranes and delirium.

A 24-year-old female presents to the primary care office with multiple complaints that began one month ago. She states that she is concerned because she has been gaining a lot of weight in the last month. Despite sleeping about twelve hours a day, she has been feeling very fatigued during the day. Her grades in her classes have also been declining as she states that her memory and focus have deteriorated. She states that she has not been feeling like herself around her friends. The most likely explanation for her findings is an imbalance of A. acetylcholine B. cortisol C. dopamine D.norepinephrine E. serotonin

The correct answer is: E This patient is presenting with signs and symptoms of depression. Diagnosis of depression requires depressed mood or anhedonia in addition to five or more signs/symptoms from the mnemonic 'SIG E CAPS' which stands for sleep (increased or decreased), interest (decreased), guilt (increased), energy (decreased), concentration, appetite/weight changes (can be weight loss or weight gain), psychomotor agitation/retardation, or suicidal ideation occurring for at least a two-week period. For proper diagnosis, organic causes must be ruled out such as hypothyroidism, brain neoplasm, dementias, etc. Remember to distinguish this diagnosis from normal bereavement after the death of a loved one as long as there is no severe impairment/suicidality. Normal bereavement can occur up to one year and may also include illusions and hallucinations as long as the patient is aware that they are not real. Major depressive disorder is considered dysthymia when mild symptoms last for more than two years. Answer A: Decreased acetylcholine has been linked to Alzheimer's disease, which is usually a disease of the elderly. The first presenting sign is often amnesia and can include aphasia, agnosia, and apraxia. Answer B: Abnormal levels of cortisol may occur in Cushing's syndrome. Cushing's syndrome patients classically present with hypertension, obesity, purple striae, hirsutism, moon facies, and a "buffalo hump". Answer C: Disturbances in dopamine are the underlying cause of diseases such as Parkinson's and schizophrenia. Parkinson's disease occurs after 50-60 years of age and present with the tetrad of resting tremor, rigidity, bradykinesia, and postural instability. Schizophrenia patients must have two or more "positive" or "negative" symptoms for six months or more with social or occupational dysfunction. Positive symptoms include hallucinations, delusions, disorganized speech, bizarre behavior, and thought disorder. Negative symptoms include flat affect, poverty of speech, and lack of purposeful actions. Answer D: Dysregulation of norepinephrine may be caused by a pheochromocytoma, a tumor of the adrenal medulla that secretes catecholamines. Patients present with intermittent tachycardia, palpitations, hypertension, and anxiety. Dysregulation of norepinephrine may also occur with depression, though not to the extent as it is correlated with serotonin levels. Bottom Line: Diagnosis of depression requires depressed mood or anhedonia in addition to five or more signs/symptoms from the mnemonic 'SIG E CAPS' (sleep, interest, guilt, energy, concentration, appetite, psychomotor agitation, suicidal ideation) occurring for at least two weeks. Dysregulation of serotonin is associated with depression.

A 35-year-old male is brought to the emergency room after being found unconscious in his bathroom. Vitals reveal a temperature of 36.2°C (97.2°F), respiratory rate of 7/min, pulse of 45/min, and blood pressure of 85/45 mmHg. The patient is only minimally responsive to painful stimuli. Pupils are pinpoint and there is mild perioral cyanosis. Which of the following pharmacologic agents should be administered next? A. atropine B. ceftriaxone C. clonidine D.flumazenil E. naloxone

The correct answer is: E This patient presents with clinical signs of opioid intoxication and should be administered naloxone immediately. Patients with opioid toxicity can present with pupillary constriction, itching, nausea, constipation, respiratory depression, slurred speech, hypotension, bradycardia, and hypothermia. While people often look for pupillary constriction as a sign of opiate intoxication, it is important to remember that it is not always present and that respiratory depression is the most sensitive sign. Severe overdose will result in respiratory suppression, coma, and sometimes death. This patient with severe signs of intoxication and respiratory depression, requires expedient endotracheal intubation to protect his airway. Naloxone is an opiate antagonist with a short half-life, and should also be administered intravenously as soon as possible to reduce the degree of respiratory depression. Answer A: Atropine would produce a sympathomimetic effect due to its inhibition of the action of acetylcholine at muscarinic receptors. Atropine is used for asystole and bradycardia. Answer B: This patient is not meeting sepsis criteria due to lack of an infection source, thus antibiotic therapy is not warranted. Answer C: Clonidine is used for signs of opiate withdrawal, not intoxication. Signs of withdrawal include lacrimation, dilated pupils, piloerection, mild fever, and mild tachycardia. Answer D: Flumazenil is used to treat respiratory depression from benzodiazepine overdose. Benzodiazepine overdoses can also present with respiratory depression but rarely with miotic pupils. Bottom Line: Naloxone antagonizes opioid receptors and should be given in suspected opioid overdose. The most life-threatening and sensitive symptom of opioid toxicity is respiratory depression.

Using a Biopsychosocial model....outline a treatment plan for a Manic Patient with narcissistic personality disorder that's unemployeed and isolated completely from their family.

Biological lithium (order TSH and creatine levels) depakote- (lfts, HCG, cbc/platelets) SGA- olanzipine (A1c/ lipids, fasting glucose) FGA- haldol Psychological introspection encouragement talk-therapy Social vocational rehabilitation encourage family reconnection legal aid

Which Cluster B personality disorder is characterized by a pervasive pattern of instability in interpersonal relationships, self-image, and emotions, and of marked impulsivity beginning by early adulthood and present in a variety of contexts

Borderline Personality Disorder "stable instability"

What is the AIMS scale? How is it used?

Before starting pt on first generation anti-psychotic abnormal involuntary movement scale baseline and repeat every 6 months

Klonopin (Clonazepam)

Benzodiazepine:Antianxiety; C-IV - panic disorder, seizures, MOA:

What mechanism of action explain the side effects of first generation anti-psychotics?

Besides D2 antagonism, first-generation agents have effects on other receptors, such as muscarinic, adrenergic alpha 1 and histamine-1. Blockade of these receptors is related with their side effects profile.

PTSD (Post Traumatic Stress Disorder)

First line tx: SSRI --increase risk of suicide and substance abuse

Define a manic episode

Lasts for 1 week. ● DISTRACTIBILITY (easier to lose attention) ● INSOMNIA (didn't feel like they needed sleep) ● GRANDIOSITY (on-top of the world feeling) ● FLIGHT OF IDEAS ● ACTIVITY (increased goal oriented activity, sex, spending) ● SPEECH (faster, more talkative) ●THOUGHTLESSNESS (impulsive decisions, out of character

A 35-year-old pregnant woman at 12 weeks' gestation, appearing disheveled and tired, presents to your clinic complaining of extreme fatigue, decreased energy, lack of interest in her usual hobbies of reading and sewing, decreased appetite, lack of concentration, and intermittent thoughts of suicidal ideation. She tells you about a plan she has devised to commit suicide which involves stabbing herself in the neck. She also tells you that she will do anything to make these feelings go away, but that she does not want to put her baby at risk. Which of the following treatments is most appropriate for this patient? A. electroconvulsive therapy B. fluoxetine C.group therapy D. paroxetine E. phenelzine

The correct answer is: A The most likely diagnosis in this case scenario is severe depression. Using the SIGECAPS mnemonic and DSM-5 criteria, this patient has depression. When a patient has devised a plan to commit suicide, immediate and emergent treatment is indicated, especially in a pregnant patient since there is risk to the fetus along with risk to the mother. This patient has recognized her depression and is willing to undergo treatment, however, she wants to avoid any potential risk to the fetus. Electroconvulsive therapy (ECT) has been shown to be very helpful and effective in the acute treatment of severe depression in patients who are not able to take antidepressant medications. Giving anti-depressant medication that may be used in certain terms of pregnancy (usually not the first term) may be allowed but would take 6-8 weeks for notable therapeutic effects and would not be immediate, emergent treatment. Thus, in this case, ECT would be the best option. In preparation for ECT in pregnant patients, a pelvic exam should be done, any non-essential anticholingeric medications should be discontinued, and uterine tocodynamometry should be available. Answer B: Fluoxetine (Prozac) is an anti-depressant and belongs to the SSRI class of medications (selective serotonin reuptake inhibitors). It is commonly prescribed for major depressive disorder, obsessive-compulsive disorder, panic disorder, bulimia nervosa, PTSD, pre-menstrual dysphoric disorder, etc. It blocks the serotonin reuptake pump (serotonin transporter) and increases serotonergic neurotransmission, boosting serotonin neurotransmission. Fluoxetine also has antagonist properties at 5HT2C receptors, which could increase norepinephrine and dopamine neurotransmission. Fluoxetine has been known to be effective in the treatment of anxiety. Onset of therapeutic action is usually not immediate, but is often delayed 2-4 weeks for depression. If it is not working adequately within 6-8 weeks, dosage can be increased. Some notable side effects include: nausea, diarrhea, decreased appetite, dry mouth, constipation, agitation, insomnia, sexual dysfunction, sweating, urinary retention, etc. The long half-life of fluoxetine provides for decreased withdrawal symptoms following any abrupt discontinuation of medication. All SSRIs are pregnancy category C, some animal studies show adverse effects but there are no controlled studies in humans. It is not generally recommended for use during pregnancy, especially in the first trimester. Continuous treatment in second and/or third terms may be needed and has not proven to be harmful to the fetus. One has to weigh the risks and benefits of treatment vs no treatment for mother and child. Exposure to SSRIs early in pregnancy may be associated with increased risk of septal cardiac defects. SSRI use beyond the 20th week of gestation may be associated with increased risk of pulmonary hypertension in newborns. This answer is incorrect as it is not indicated for acute treatment in severe depression as the patient in this case scenario needs. Answer C: While group therapy is a conservative treatment for depression that poses no risk to the mother or fetus, this patient's severe depression is evidenced by her plan to commit suicide. Group therapy is unlikely to be beneficial at this point in the short-term period of her depression and immediate treatment should be sought. Answer D: Paroxetine (Paxil) is an anti-depressant and belongs to the SSRI class of medications (selective serotonin reuptake inhibitors). It is commonly prescribed for major depressive disorder, obsessive-compulsive disorder, panic disorder, social anxiety disorder, post-traumatic stress disorder, generalized anxiety disorder, and pre-menstrual dysphoric disorder. It blocks the serotonin reuptake pump and increases serotonergic neurotransmission, boosting serotonin neurotransmission. Paroxetine also has mild anticholinergic actions. Paroxetine has been known to be very effective in the treatment of anxiety. Onset of therapeutic action is usually not immediate, but is often delayed 2-4 weeks for depression. If it is not working adequately within 6-8 weeks, dosage can be increased. Some notable side effects include: nausea, diarrhea, decreased appetite, dry mouth, constipation, agitation, insomnia, sexual dysfunction, urinary retention, etc. Paroxetine produces the highest incidence of discontinuation syndrome out of the SSRIs due to its relatively short half-life and anticholinergic activity. Paroxetine is an SSRI that should not be used in pregnant patients due to an increased risk of persistent pulmonary hypertension in the newborn and anencephaly. Epidemiological data have shown an increased risk of cardiovascular malformations in infants born to mothers who took the medication in first trimester. Paroxetine usage later in pregnancy may be associated with higher risk of neonatal complications, including respiratory distress. Answer E: Phenelzine (Nardil) belongs to the MAOI class (monoamine oxidase inhibitor) and is FDA approved for atypical depression, and is often used for treatment-resistant depression and treatment-resistant panic disorder. It works by irreversibly blocking MAO from breaking down norepinephrine, serotonin, and dopamine. This presumably boosts neurotransmission of these neurotransmitters. Onset of depression. If it is not working adequately within 6-8 weeks, dosage can be increased. Notable side effects include dizziness, sedation, headache, sleep disturbance, fatigue, weakness, tremor, constipation, dry mouth, nausea, and weight gain. If a patient is on a MAOI, he/she has to avoid tyramine containing foods to prevent occurrence of hypertensive crisis. Due to their propensity to cause a hypertensive crisis that can lead to strokes in pregnant and non-pregnant patients, MAOIs should not be used during pregnancy. MAOIs can also cause a possibly fatal serotonin syndrome when combined with medications that block serotonin reuptake, so it is advisable to wait 5 weeks after discontinuing an SSRI before starting the MAOI in a non-pregnant patient. Bottom Line: ECT has been found to be very effective for the acute treatment of depressive episode and would greatly help this patient.

A 55-year-old woman presents to her primary care physician with the chief complaint of "itching all over because of bugs on my skin." She says the itching started about a month ago, and she has not seen a rash but she knows she has "bugs living on me." During the interview she produces a small plastic bag with lint and skin debris that she says also contains the insects. She has no complaints other than the itching and parasite infestation. She has a past medical history significant for hypertension and depression. Medications include sertraline and lisinopril. She has not traveled recently and has not been exposed to animals. She does sometimes work in her back yard where there is some tall grass. Physical examination reveals that she is alert and oriented. Her skin has excoriations on her forearms and back without evidence of a primary rash. Her eyes are anicteric. Laboratory studies, including skin biopsy and microscopic examination of the contents of the patient's plastic bag, yield no evidence of infestation or systemic disease. Which of the following is the most likely diagnosis? A. delusional disorder B. functional neurological symptom disorder C.illness anxiety disorder D. major depressive disorder with psychotic features E. somatic symptom disorder

The correct answer is: A This patient most likely has delusional disorder of the somatic subtype. DSM-5 criteria (abridged) include the following: A. The presence of one (or more) delusions with a duration of 1 month or longer. B. Criterion A for schizophrenia has never been met. Hallucinations, if present, are not prominent and are related to the delusional theme (e.g., the sensation of being infested with insects associated with delusions of infestation). C. Apart from the impact of the delusion(s) or its ramifications, functioning is not markedly impaired, and behavior is not obviously bizarre or odd. Answer B: Conversion disorder is now known as functional neurologic symptom disorder and includes those with altered voluntary motor or sensory function. It was re-named in DSM-5 to emphasize the neurologic examination. Answer C: Hypochondriasis is now known as illness anxiety disorder in DSM-5. Those with illness anxiety disorder have a preoccupation with having or acquiring a serious illness. Somatic symptoms are not present or, if present, are only mild in severity. Answer D: This patient has minimal features of major depressive disorder. Answer E: Somatization disorder is now known as somatic symptom disorder in DSM-5. In somatic symptom disorder, the individual's beliefs that somatic symptoms might reflect serious underlying physical illness are not held with delusional intensity. Nonetheless, the individuals beliefs concerning the somatic symptoms can be firmly held. In contrast, in delusional disorder, somatic subtype, the somatic symptom beliefs and behavior are stronger than those found in somatic symptom disorder. Bottom Line: Patients with delusions of parasitosis have a fixed belief that they have pruritus due to a parasitic infestation. Antipsychotic medications are the accepted treatment.

A 35-year-old female presents to the emergency department for fatigue. She reports that she has been feeling increasing weakness since starting an antidepressant for depression and anxiety a few months ago. She states that the medication has not helped and actually made her feel worse. Physical examination reveals pale conjunctiva and oral mucosa. A guaiac test is performed and is positive. Laboratory studies reveal the following: Hemoglobin 7.0 g/dL Mean corpuscular volume 65 fL Mean Corpuscular Hemoglobin Concentration 25 g/dL Which of the following medications should be discontinued? A. doxepin B. fluoxetine C. levothyroxine D. lithium E. propranolol

The correct answer is: B This patient is presenting with a case of acute microcytic hypochromic anemia. Serotonin reuptake inhibitors, especially fluoxetine and paroxetine, can increase risk of abnormal bleeding. This is thought to be caused by two mechanisms. First, serotonin is known to directly promote platelet aggregation. Serotonin reuptake inhibitors are thought to limit the uptake of blood serotonin by platelets. This decreased amount of serotonin taken up by platelets may increase risk of abnormal bleeding. Second, serotonin reuptake inhibitors inhibits liver microsomal enzymes, specifically cytochrome P450 2D6. As a result, elevations of other drugs metabolized through the same hepatic enzyme (warfarin) can significantly elevate. Because of the above mechanisms, caution must be used when prescribing concomitantly with other anticoagulant medications especially aspirin, nonsteroidal anti-inflammatory drugs, warfarin, etc. Other common side effects of serotonin reuptake inhibitors include sexual disturbances, gastrointestinal effects, agitations, insomnia, and tremor. Serotonin syndrome can also occur if used in conjunction with monoamine oxidase inhibitors. Answer A: Doxepin is a tricyclic antidepressant. It may enhance the antiplatelet effect of NSAIDs, but this class of medications does not have a major bleeding risk. Common adverse effects of TCAs include: overdose (mental status changes, anticholinergic toxicity, seizures), orthostatic hypotension, arrhythmia, sedation, weight gain, and sexual dysfunction. Answer C: There is no associated risk of bleeding or anemia with levothyroxine. Answer D: Lithium is used as a first line mood stabilizer. The most common side effects can be remembered by the mnemonic "LMNOP" (Lithium, Movement (tremor), Nephrogenic diabetes insipidus, hypOthyroidism, Pregnancy problems (Ebstein's anomaly). Answer E: There is no associated risk of bleeding or anemia with propranolol. Bottom Line: Serotonin reuptake inhibitors can increase risk of abnormal bleeding.

A 10-year-old male was sent home from school due to inappropriate behaviors towards his classmates. He is in 5th grade and has been maintaining a C average in class. He often starts physical altercations and bullies 2 or 3 students per day. He has caused other students to cry or fight back with him. His mother reports that he had been cruel to rabbits in the backyard and would try to trap and beat them up. He also has gotten in trouble in the past for lying and for defying authority figures at home and at school, and was also suspended twice in the past for stealing something from the classroom. Most recently, he was found touching the genital area of several girls and he was trying to coerce them into the bathroom against their will at school. He has a history of attention-deficit hyperactivity disorder for which he takes medication. Approximately two years ago, he developed motor tics which included involuntary shoulder shrugging, blinking, and head thrusting. He has also been known to grunt and bark in the middle of class. Which of the following best accounts for his recent inappropriate behavior towards girls? A. antisocial personality disorder B.borderline personality disorder C. conduct disorder D. oppositional defiant disorder E. Tourette's disorder

The correct answer is: C Children with ADHD often have other co-morbidities including Tourette's disorder, oppositional defiant disorder, and conduct disorder. Conduct disorder presents in children or adolescents with a repetitive and persistent pattern of behavior in which the basic rights of others or major age-appropriate societal norms or rules are violated. Behaviors in conduct disorder include: aggression to animals and people, destruction of property, deceitfulness, theft, and serious violation of rules. Criteria for conduct disorder are met when at least three or more of the following are met: (1) often bullies or threatens others; (2) often initiates fights; (3) has used a weapon or object that can cause serious harm to others; (4) has been physically cruel to others; (5) has been physically cruel to animals; (6) has stolen while confronting a victim (example: mugging); (7) has forced someone into sexual activity; (8) has deliberately engaged in fire setting with the intention of causing serious harm; (9) has deliberately destroyed others' property; (10) has broken into someone else's house or car; (11) often lies to avoid obligations or to obtain favors; (12) has stolen items of non-trivial value without confronting a victim; (13) often stays out at night despite parental prohibitions, beginning before age 13 years; (14) has run away from home at least twice; (15) is often truant from school, beginning before age 13 years. The behavioral disturbances cause clinically significant impairments in social, occupational, or academic functioning. There is an earlier average of onset for boys (age 10-12 years) compared to girls (age 16 years). Prevalence is higher amongst boys compared to girls. The patient in this case scenario demonstrates at least 6 of the above-mentioned criteria for conduct disorder, making this choice the best answer. Conduct disorder is also more commonly found in children with ADHD. Answer A: Antisocial personality disorder is diagnosed at or above 18 years of age. Individuals with anti-social personality disorder have long criminal records of repetitive unlawful acts and socially irresponsible behaviors that began prior to the age of 15 years (otherwise known as conduct disorder in children and adolescents). These individuals lack remorse for the harm they cause and are generally unconcerned for the feelings and rights of others. Most of these individuals are male. Frequent co-morbid disorders include ADHD, somatization disorder, and substance abuse disorders. The patient in the case scenario is below the age of 18 years. Answer B: Borderline personality disorder involves a pervasive pattern of unstable interpersonal relationships, self-image, affects, and marked impulsivity beginning by early adulthood, and present in a variety of contexts, as indicated by at least five or more of the following: (1) frantic efforts to avoid real or imagined abandonment; (2) a pattern of unstable interpersonal relationships characterized by attenuating between the extremes of idealization and devaluation (splitting); (3) self-identity disturbance; (4) impulsivity in at least two areas that are potentially self-damaging (such as spending, sexual behaviors, reckless driving, binge eating, etc); (5) recurrent suicidal behaviors or gestures; (6) affective instability due to a marked reactivity of mood; (7) chronic feelings of emptiness; (8) inappropriate, intense anger and difficulty with controlling the anger; (9) transient, stress-related paranoid ideation. Borderline personality disorder is seen as occurring over a chronic course throughout the lifetime. Treatment options include extensive psychotherapy (such as dialectical behavioral therapy) combined with low-doses of antipsychotics, SSRIs, or mood stabilizers. Answer D: Oppositional defiant disorder (ODD) comprises of a pattern of negativistic, hostile, and defiant behavior lasting at least 6 months, during which at least 4 or more of the following criteria are present: (1) often loses temper; (2) often argues with adults; (3) often actively defies to comply with adults' requests or rules; (4) often deliberately annoys people; (5) often blames others for his or her mistakes or misbehavior; (6) is often touchy or easily annoyed by others; (7) is often angry and resentful; (8) is often spiteful or vindictive. The behaviors are usually directed at authority figures. The behavioral disturbances cause clinically significant impairments in social, occupational, or academic function. ODD cannot be diagnosed if conduct disorder is present. There is a gradual onset of symptoms, usually before 8 years of age. Answer E: Tourette's disorder manifests with an onset before the age of 18 years and presents with both involuntary motor and vocal tics at some time during the illness, although not necessarily at the same time. A tic is a sudden, rapid, recurrent, non-rhythmic, stereotypic motor movement or vocalization. The tics occur many times during the day nearly every day or intermittently throughout a period of more than one year. During this period, there is never a tic-free period of more than three consecutive months. The patient in this case scenario has Tourette's disorder which accounts for his symptoms of involuntary motor and vocal tics but does not account for his inappropriate aggressive and sexual behaviors. Bottom Line: Conduct disorder is a childhood disorder characterized by antisocial behaviors including stealing, fighting, destroying property, forcing sexual activity on others, truancy, cruelty towards people and/or animals, etc.

A 34-year-old male is arrested for rubbing himself on others in public. He admits to experiencing gratification from rubbing his penis against young strangers in crowded places since he was in his twenties. The most likely diagnosis is A. exhibitionistic disorder B. fetishistic disorder C. frotteuristic disorder D. sexual masochism disorder E.voyeuristic disorder

The correct answer is: C Frotteurism is defined as recurrent and intense sexual fantasies, urges or behaviors that involve rubbing against or touching a non-consenting person. The condition must cause marked distress or disturbance in functioning and be present for at least 6 months. The sexual acts often occur in crowded places such as subways and buses and these patients tend to be passive and isolated. Answer A: An intense and recurrent sexual urge to expose the genitals to unsuspecting strangers lasting six months or more characterizes exhibitionism. The anticipation of the exposure produces sexual excitement. This condition occurs nearly always in men. Answer B: In fetishism, sexual fantasies, urges or behaviors are focused on nonliving objects such as shoes, stockings or gloves. It is often a chronic disorder that usually begins in adolescence and may have been established in childhood. When the behavior involves a heterosexual male cross-dressing and the behavior causes significant distress or impairment it is referred to as transvestic fetishism. Fetishism is much more common in men, as are all the paraphilias. Answer D: Those with masochism disorder have intense sexual arousal from the act of being humiliated, beaten, bound, or otherwise made to suffer. Answer E: Voyeurism is recurrent and intense preoccupation with observing an unsuspecting person who is disrobing, naked or engaged in sexual activity. It is also known as scopophilia. Bottom Line: Paraphilia is defined as specific deviant sexual behaviors or stimuli necessary for arousal and orgasm, with suffers being unable to respond to typical, non-deviant erotic stimuli. The Diagnostic and Statistical Manual of Mental Disorders (DSM-IV-TR) identifies several paraphilias (exhibitionism, fetishism, transvestic fetishism, pedophilia, sexual masochism, sadism and voyeurism) under the category of sexual and gender identity disorders. These divergent behaviors exclude or harm others and do not involve the bonding between persons and procreation that define normal sexual behavior. COMBANK Insight : Most paraphilic disorders are unchanged in DSM-5 with the addition of course specifiers in a controlled environment" and "in remission." There is a distinction between paraphilias and paraphilic disorders. A paraphilic disorder is a paraphilia that is currently causing distress or impairment to the individual or a paraphilia whose satisfaction has entailed personal harm, or risk of harm, to others. A paraphilia is necessary but not a sufficient condition for having a paraphilic disorder, and a paraphilia by itself does not automatically justify or require clinical intervention.

A 25-year-old man with a history of type 1 diabetes, renal insufficiency, and seizure disorder presents for management of depression. His medications include insulin, lisinopril, and valproic acid. He previously was on fluoxetine but did not like the sexual side effects. He wants to try bupropion. Which of the following is an absolute contraindication to use of this medication in this man? A. insulin dependent diabetes B. lisinopril use C. renal insufficiency D. seizure disorder E. valproic acid use

The correct answer is: D Bupropion is an anti-depressant and is usually a good alternative for patients who have struggled with sexual side effects on other anti-depressants, as well as for patients who wish to attempt smoking cessation. It weakly inhibits the neuronal uptake of norepinephrine and dopamine. However, it can lower the seizure threshold and is absolutely contraindicated in patients with a seizure disorder, as this patient does. Other absolute contraindications to use include monoamine oxidase use within 14 days, those on linezolid or IV methylene blue, anorexia nervosa, and bulimia nervosa (bulimic patients also often present with seizures on bupropion), use of MAO inhibitors or MAO inhibitors intended to treat psychiatric disorders (concurrently or within 14 days of discontinuing either bupropion or the MAO inhibitor). Common adverse effects of bupropion include tachycardia, insomnia, and headaches. Answer A: Hyperglycemia and hypoglycemia are listed as adverse reactions to bupropion with an occurrence of <1%. However, this is not a contraindication. Answer B: There are no known interactions between lisinopril and bupropion. Answer C: The manufacturer's labeling suggests a reduction in dose and/or frequency be considered in those with chronic kidney disease, but it does not provide specific dosing recommendations. Answer E: There are no known interactions between valproic acid and bupropion. Bottom Line: Seizure disorder is an absolute contraindication to use of bupropion.

A 55-year-old female presents with her daughter who is concerned that the patient has become increasingly withdrawn over the past four weeks. History reveals a six-week period of difficulty sleeping, decreased appetite, anhedonia, difficulty concentrating, and increased agitation. She no longer takes part in her hobbies or interacts with her grandchildren minimally. The patient admits to feeling hopeless and worthless but she states that this is because "the devil has taken her soul." The daughter states that her mother was hospitalized four months ago for hearing voices and having delusions but was not having any mood symptoms at the time. Two months ago, the patient was found talking to the walls about a plot to save her soul, and last night the patient stated that she can no longer fight the devil and that she has surrendered her soul. The most likely diagnosis is A. delusional disorder B. depressive pseudodementia C.major depression with psychotic features D. schizoaffective disorder E. schizophreniform disorder

The correct answer is: D Schizoaffective disorder comprises an uninterrupted period of illness during, which at some time, there is either a major depressive episode, a manic episode, or a mixed episode concurrent with symptoms that meet Criterion A for schizophrenia. During the same period of illness, there have been delusions or hallucinations for at least two weeks in the ABSENCE of prominent mood symptoms. Symptoms that meet criteria for a mood episode are present for a substantial portion of the total duration of the active and residual periods of the illness. The disturbance is not due to the direct physiological effects of a substance or a general medical condition. Criterion A for schizophrenia would be met by the presence of at least two or more of the following, each present for a significant portion of time, continuously for more than six months: (1) delusions; (2) hallucinations; (3) disorganized speech; (4) grossly disorganized or catatonic behaviors; (5) negative symptoms. Delusions, disorganized speech, hallucinations, and agitation are considered to be positive symptoms, associated with dopamine receptors. Negative symptoms of schizophrenia are associated with muscarinic receptors and include characteristics such as flattened affect, social withdrawal, anhedonia (lack of interest), apathy, and poverty of thought. A major depressive episode comprises of five or more of the following symptoms have been present during the same two-week period and represents a change from previous functioning: (1) depressed mood most of the day, nearly every day; (2) markedly decreased interest or pleasure in most activities; (3) significant weight loss when not dieting and due to decreased appetite or weight gain from increased appetite; (4) insomnia or hypersomnia nearly every day; (5) psychomotor agitation or retardation nearly every day; (6) fatigue or loss of energy nearly every day; (7) feelings of worthlessness or excessive or inappropriate guilt nearly every day; (8) decreased ability to think or concentrate; (9) recurrent thoughts of death or suicidal ideation. At least one of the symptoms has to be either (1) depressed mood or (2) loss of interest or pleasure. These symptoms cause clinically significant distress or impairment in social, occupational, or other important areas of functioning. This patient's mood symptoms have been present for at least two weeks (and she meets criteria for having a major depressive episode) but her psychotic symptoms have been present for at least more than two months, and thus, the diagnosis is appropriate. Management of schizoaffective disorder typically involves the use of cognitive behavioral therapy in addition to anti-psychotic medications, usually atypical anti-psychotic medications. Answer A: Delusional disorder involves delusions of at least one month's duration and NOT meeting the criteria for schizophrenia (e.g., hallucinations). The DSM-5 criterion A for delusional disorder no longer has the requirement that the delusions must be non-bizarre; a specifier is now included for bizarre type delusions to provide continuity with DSM-IV. Those with delusional disorder may have olfactory or tactile hallucinations if they are directly related to the delusion. Answer B: Depressive pseudodementia is a syndrome seen in older patients in which they exhibit symptoms consistent with dementia but the cause is actually depression. Answer C: Major depression may be classified as with psychotic features if there is the presence of delusions or hallucinations during the major depressive episode. This diagnosis would be appropriate if the psychotic symptoms occurred ONLY when the patient was having mood symptoms. Answer E: Schizophreniform disorder occurs when symptoms of schizophrenia are present for more than one month but less than six months. It cannot account for this patient's mood symptoms. Bottom Line: To make the diagnosis of schizoaffective disorder, the psychotic symptoms must be present in the absence of the mood symptoms for at least two weeks. COMBANK Insight : In DSM-5 Schizoaffective disorder is reconceptualized as a longitudinal instead of a cross-sectional diagnosis - more comparable to schizophrenia, bipolar disorder, and major depressive disorder, which are bridged by this condition - and requires that a major mood episode be present for a majority of the total disorders duration after Criterion A has been met.

A 40-year-old female presents to the clinic complaining of fatigue. History reveals that she has been unemployed for the past few months and often stays at home worrying about her financial situation. She states she is always anxious and restless and has been having difficulty falling asleep. She becomes agitated very easily which often results in a fight with her boyfriend as well. Physical examination and vitals are unremarkable. What is the most appropriate pharmacologic therapy for this patient's symptoms? A. alprazolam B. amitriptyline C. diazepam D. paroxetine E. pregabalin

The correct answer is: D This patient is suffering from generalized anxiety disorder (GAD), characterized by persistent worrying along with psychological and physical symptoms that interfere with daily functioning of an individual as demonstrated by fatigue and insomnia in this patient. First line treatment for GAD is based on pharmacologic and/or psychological therapy with cognitive behavioral therapy depending on availability of resources and patient preference. Regarding pharmacologic therapy, guidelines support selective serotonin receptor inhibitor SSRI (paroxetine) or selective norepinephrine reuptake inhibitor SNRI (venlafaxine, duloxetine) as first line therapies for GAD. Benzodiazepines and tricycle anti-depressants (TCA) like amitriptyline are used as second line therapy for patients who do not respond to SSRI or SNRI or in addition to them. . Diazepam is also a benzodiazepine which helps curb anxiety by potentiating the effects of endogenous GABA. Benzodiazepines are more useful for acute symptom relief during the period before an SSRI takes effect or in addition to an SSRI for partial responders; however, it should not be started as a single agent for long-term therapy due to side effect profile. Side effects include dependence especially in someone with a history of drug or alcohol abuse. Other side effects include amnesia, drowsiness and impaired psychomotor performance. Benzodiazepines like alprazolam and diazepam are second line therapies for GAD, therefore, making it a less likely answer choice. Bottom Line: SSRI like paroxetine is first line therapy for patients with generalized anxiety disorder (GAD).

A 53-year-old man with a history of hypertension, hyperlipidemia, type 2 diabetes mellitus, and insomnia presents to his primary care physician with the chief complaint of weight gain. His medications include hydrochlorothiazide, simvastatin, metformin, glyburide, and zolpidem. He has been feeling generally well recently and does not understand why he has gained 4.5 kg (10 lb) in the last month without any changes in eating pattern. He has noticed crumbs in his bed when he wakes up some mornings. He has a 60 pack-year smoking history and drinks 1-2 glasses of wine nightly. He weighs 85 kg (187 lbs). Physical examination is unremarkable. Laboratory studies reveal a hemoglobin A1c of 7.5% and a total cholesterol of 250 mg/dL. Which of the following is the most likely cause of his weight gain? A. depression B. excessive alcohol intake C. metformin D.normal consequence of aging E. zolpidem

The correct answer is: E This patient most likely has sleep-related eating disorder as evidenced by weight gain, crumbs in the bed, and recent increase in zolpidem use. Patients with sleep-related eating disorder eat while asleep and often only recognize the cause of their weight gain because food is missing or left out in the kitchen, or there are crumbs in the bed. Answer A: Though this patient has changes in his sleep patterns and increased weight, there is no report of a depressed mood or loss of interests, consistent with a mood disorder. Answer B: Excessive alcohol intake can cause weight gain but this patient has not changed the amount that he drinks recently. Answer C: Metformin would be expected to cause weight loss, not weight gain. Answer D: Normal aging alone does not cause weight gain. Bottom Line: Sleep related-eating disorder involved eating while asleep. It can be caused by hypnotics used for insomnia, such as zolpidem.

Pharmacological Treatment of Delirium

treat underlying conditions Reorientation techniques (clocks, photos, quiet, lighting) eliminate sensory deficits (hearing aids/glasses) regulate sleep rhythms -Haloperidol CF


Set pelajaran terkait

GEO 309: Sedimentation Stratigraphy - Test 1

View Set

ap lang - ai chatbots, chatgpt, etc.

View Set

HIM 298 Sayles CHAPTER 10 DATA SECURITY

View Set

Life Insurance and Health Insurance

View Set